312
Pediatrics Tests (текстові тестові завдання) 1. Name the main way of the infection entering at pneumonia: A. *bronchogenic B. hematogenic C. lymphogenic D. mixed E. urogenic 2. In pneumonia etiology prevails: A. candida B. klebsiella C. *pneumococci D. staphylococci E. streptococci 3. At the early childhood an acute pneumonia most often is: A. interstitial B. croupous (lobar) C. *microfocal D. segmental E. polysegmental 4. Typical physical data at pneumonia are: A. diffuse dry wheezes B. local small moist wheezes C. diffuse small moist wheezes D. *diminished breath sounds E. local dry wheezes 5. The chest X-ray sign typical for acute pneumonia is: A. strengthening of pulmonary picture (lung pattern) B. *infiltrative shadows C. emphysema D. dilation of lungs' roots E. pneumosclerosis 6. What medicine is not used for improvement of the rheologic blood properties and microcirculation during treatment of acute pneumonia? A. rheopoliglucin B. heparin C. trental D. *droperidol E. curantil 7. Duration of an acute pneumonia is: A. up to 4 weeks B. *up to 8 weeks C. up to 10 weeks

Pediatrics - Тернопільський державний медичний ...intranet.tdmu.edu.ua/.../med/lik/Pediatrics/6/Pediatrics.doc · Web viewSpirography ECG Allergic tests

  • Upload
    dominh

  • View
    458

  • Download
    130

Embed Size (px)

Citation preview

PediatricsTests (текстові тестові завдання)

1. Name the main way of the infection entering at pneumonia:A. *bronchogenicB. hematogenicC. lymphogenicD. mixedE. urogenic

2. In pneumonia etiology prevails:A. candidaB. klebsiellaC. *pneumococciD. staphylococciE. streptococci

3. At the early childhood an acute pneumonia most often is:A. interstitial B. croupous (lobar)C. *microfocalD. segmentalE. polysegmental

4. Typical physical data at pneumonia are:A. diffuse dry wheezes B. local small moist wheezesC. diffuse small moist wheezesD. *diminished breath soundsE. local dry wheezes

5. The chest X-ray sign typical for acute pneumonia is:A. strengthening of pulmonary picture (lung pattern)B. *infiltrative shadowsC. emphysemaD. dilation of lungs' roots E. pneumosclerosis

6. What medicine is not used for improvement of the rheologic blood properties and microcirculation during treatment of acute pneumonia?A. rheopoliglucin B. heparinC. trentalD. *droperidolE. curantil

7. Duration of an acute pneumonia is:A. up to 4 weeksB. *up to 8 weeksC. up to 10 weeksD. up to 2 weeksE. up to 6 weeks

8. In most cases an acute pneumonia at children of the early age develops as a result of:A. overheat B. super coolingC. violation of the regimeD. *upper respiratory tract viral infectionE. upper respiratory tract microbial infection

9. To the pathogenetic links of an acute pneumonia does not belong:A. hypoxemia

B. *alcalosisC. acidosisD. hypoxiaE. bacteriemia

10. To bronchopneumonia are not typical such laboratory changes:A. *reticulocitosisB. leucocytosisC. neutrophylosisD. elevated ESR E. anemia

11. The clinical features of an acute pneumonia at children of the early age do not depend on the following:A. age B. sexC. premorbid stateD. *nationalityE. weight

12. To the criteria of an acute pneumonia diagnosis do not belong:A. intoxication B. *epidemiological historyC. local physical changesD. respiratory acidosisE. percussion changes

13. Correlation of pulse and breathing 2 - 1,5 : 1 is typical for:A. Respiratory ins. 0 st.B. Respiratory ins. I st.C. Respiratory ins. II st.D. *Respiratory ins. III st.E. Respiratory ins. IVst.

14. Oxygen saturation of the blood 90 % is typical for:A. Respiratory ins. 0 st.B. *Respiratory ins. I st.C. Respiratory ins. II st.D. Respiratory ins. III st.E. Respiratory ins. IVst.

15. Inconstant perioral cyanosis is typical for:A. Respiratory ins. 0 st.B. *Respiratory ins. I st.C. Respiratory ins. II st.D. Respiratory ins. III st.E. Respiratory ins. IVst.

16. Level РСО2 40 mm Hg. is typical for:A. Respiratory ins. 0 st.B. Respiratory ins. I st.C. Respiratory ins. II st.D. *Respiratory ins. III st.E. Respiratory ins. IVst.

17. Mostly destructive pneumonia is caused by:A. pneumococciB. *staphylococciC. proteusD. klebsiellaE. streptococci

18. What type of oxygen therapy is the best for a child with the Respiratory ins. III st.?A. serve of oxygen through an oxygen pillowB. serve of oxygen in an oxygen tentC. serve of oxygen through a nasal catheterD. *serve of oxygen through an intubation tubeE. serve of humidified oxygen through a mask

19. What type of oxygen therapy is the best for a child with the Respiratory ins. II st.?A. serve of oxygen through an oxygen pillowB. serve of oxygen in an oxygen tentC. serve of oxygen through a nasal catheterD. serve of oxygen through an intubation tubeE. *serve of oxygen through a mask

20. What group of antibiotics is not used for pneumonia treatment at children of the early age?A. penicyllinesB. *tetracyclinC. aminoglicosydesD. cefalosporinesE. macrolides

21. At mild pneumonia antibacterial therapy lasts:A. 5 - 7 daysB. 3 - 5 days C. *7 - 10 daysD. 10 - 14 daysE. 15 - 18 days

22. At moderate pneumonia antibacterial therapy lasts:A. 5 - 7 daysB. 7 - 10 daysC. *10 - 14 dayD. 14 - 20 daysE. 15 - 18 days

23. At severe pneumonia antibacterial therapy lasts:A. 7 - 10 days B. 10 - 14 daysC. *14 - 21 dayD. 21 - 24 daysE. 24 - 28 days

24. For treatment of mild pneumonia semisynthetic penicillin’s are used in a dose:A. 30 - 50 mg/kg/dailyB. *50 - 80 mg/kg/dailyC. 80 - 100 mg/kg/dailyD. 100 - 150 mg/kg/dailyE. 150 - 200 mg/kg/daily

25. For treatment of moderate pneumonia semisynthetic penicillin’s are used in a dose:A. 30 - 50 mg/kg/dailyB. 50 - 80 mg/kg/dailyC. *80 - 100 mg/kg/dailyD. 100 - 150 mg/kg/dailyE. 150 - 200 mg/kg/daily

26. For treatment of severe pneumonia semisynthetic penicillin’s are used in a dose:A. 30 - 50 mg/kg/dailyB. 50 - 80 mg/kg/dailyC. 80 - 100 mg/kg/dailyD. 100 - 150 mg/kg/daily

E. *150 - 200 mg/kg/daily27. In definition of pneumonia is absent the following position:

A. *it is an noninfectious disease of pulmonary parenchymaB. pneumonia is diagnosed at presence of respiratory disordersC. pneumonia is diagnosed at presence of tachypneaD. pneumonia is diagnosed at presence of infiltrative changes on x-rayE. pneumonia is diagnosed at presence of respiratory insufficiency

28. Among the forms of pneumonia is absent:A. community acquiredB. hospitalC. *teenagers' pneumoniaD. ventilator associatedE. bronchopneumonia

29. In classification of pneumonia forms is absent:A. pneumonia in children who have an immune deficitB. ventilator associated C. congenital pneumonia D. *extraschoolE. hospital

30. Hospital pneumonia is one that developed:A. during 6 hours being in the hospitalB. during 12 hours being in the hospitalC. during a 24 hours being in the hospitalD. *during 48 hours being in the hospitalE. during 56 hours being in the hospital

31. Hospital pneumonia is one that developed:A. during 6 hours after discharge from the hospitalB. during 12 hours after discharge from the hospitalC. during a 24 hours after discharge from the hospitalD. *during 48 hours after discharge from the hospitalE. during 56 hours after discharge from the hospital

32. Pulmonary complications of pneumonia don’t belong to:A. pleurisy B. pneumothoraxC. pyopneumothoraxD. *DIC syndrome E. abscess

33. More often the etiology of community acquired pneumonia in children from 6 months to 6 years is:A. chlamydia B. mycoplasmC. *pneumococcusD. staphylococcusE. E. coli

34. More often the etiology of community acquired pneumonia in children from 7 to 15 years is:A. streptococcus B. Listeria monocytogenesC. *pneumococcusD. hemophilus influenzaE. E. coli

35. What is the definition of tachypnea in children 2-12 mo: A. *>50 breaths/minB. >80

C. >60D. >40E. >35

36. What is the definition of tachypnea in children 1-5 ya:A. >50 breaths/minB. >80C. >60D. *>40E. >35

37. To the most frequent etiological agents of hospital pneumonia does not belong:A. E. coli B. *pneumococcusC. proteusD. enterobacterE. virus

38. Ventilator associated pneumonias (with less than four days of artificial ventilation), usually, are conditioned by:A. pneumococcusB. B. cereusC. *klebsiellaD. enterobacterE. virus

39. Pneumococci are absolutely non-sensitive to:A. penicillinesB. *aminoglicozidesC. macrolydesD. cephalosporin’sE. tetracycline’s

40. Medicine of choice to treat typical community acquired pneumonias is:A. *aminopenicillinsB. carbapenemsC. fluorquinolonesD. antibiotics of other groupsE. tetracycline’s

41. To the antibiotics of the first choice to treat typical community acquired pneumonias do not belong:A. amoxycillinB. macrolidesC. *fluorquinolonesD. inhibitor-protected penicillinsE. carbapenems

42. Positive effect of pneumonia treatment by antibiotic is all, exceptA. improvement of the general conditionB. decrease of temperature below 38°C in 24 - 48 hours from the beginning of treatmentC. *increase of dyspneaD. roentgenologic changes do not increase or even decrease E. decrease of dyspnea

43. An absent effect of pneumonia treatment by antibiotic is all, except:A. fever remainsB. *improvement of the general conditionC. increase of pathological changes in the lungsD. increase of dyspnea and hypoxemiaE. decrease of dyspnea and hypoxemia

44. For the beginning of lobar pneumonia is not typical:A. headacheB. normal or subfebrile temperature C. *poor consciousnessD. cough with «ferruginous» sputumE. febrile temperature

45. What symptom of pneumonia is typical only for one month old children?A. CoughB. Foamy excretions from a nose and mouthC. Dulling of percussion soundD. *Nod motions by a head E. Dissipated moist wheezes

46. At what respiratory insufficiency stage is observed inconstant perioral cyanosis ?A. *At the firsB. At the secondC. At the thirdD. At all stagesE. Does not have the diagnostic value

47. Beginning of lobar pneumonia reminds sometimes:A. attack of bronchial asthmaB. rheumatismC. *acute appendicitisD. pyelonephritisE. gastritis

48. ?More often the reason of obstructive bronchitis is: A. *Viral – bacterial flora B. ParasitesC. Fungi’s D. Bacterial - fungi’s flora E. Viral – fungi’s flora

49. The beginning of obstructive bronchitis is:A. *Catarrhal syndromeB. Allergic reactionsC. Inspiration dyspneaD. Expiration dyspneaE. Tonsillitis

50. At the acute phase of obstructive bronchitis is prevailing:A. IntoxicationB. *CoughC. DyspneaD. WheezingE. Tonsillitis

51. Percussion during the acute phase of obstructive bronchitis is:A. Clear lung soundB. *Box soundC. Dullness of lung soundD. Dullness of lung sound in lower partsE. Dullness of lung sound in upper parts

52. Auscultation during the acute phase of obstructive bronchitis is:A. Prolonged inspirationB. Decrease of breathingC. *Dry rales and moist diffuse ralesD. Crepitation

E. Local moist rales53. During the acute period of obstructive bronchitis on X-ray is present:

A. *Perivascular and peribronhial infiltration, increasing of lung pattern and enlarged roots of lungs

B. Particularly clear lung fieldC. Hyperinflation D. Occasional scattered areas of consolidationE. Local infiltration of lung tissue

54. The main treatment of obstructive bronchitis is:A. Liquidation of virusesB. Desintoxication therapyC. * Normalization of drainage function of bronchusD. Rehydratation therapyE. Antibiotics

55. The chest falls on inspiration and rises on expiration. What type of respiration is it?A. Kussmaul’s respirationB. *Paradoxical respirationC. Normal respirationD. Biot’s respirationE. Cheyne-Stokes respiration

56. What type of respiratory movements is in children after 7 years?A. *ThoracicB. AbdominalC. CostalD. SternlyE. Diaphragmatic

57. What is tachypnea?A. *The increase of the respiratory rateB. The distress during breathingC. The decrease of the respiratory rateD. The cessation of breathingE. The increase of the respiratory depth

58. What is an average respiratory rate in 1-year old child?A. 20B. *30C. 18D. 50E. 60

59. What is apnea?A. The increase of the respiratory rateB. The distress during breathingC. The decrease of the respiratory rateD. *The cessation of breathingE. The increase of the respiratory depth

60. What is bradypnea?A. The increase of the respiratory rateB. The distress during breathingC. *The decrease of the respiratory rateD. The cessation of breathingE. The decrease of the respiratory depth

61. What is an average respiratory rate in children after 12 years?A. 22B. 32

C. *18D. 25E. 45

62. What is usual ratio of breaths to heartbeats?A. 1:1B. 1:2C. 1:3D. *1:4E. 1:5

63. What is an average respiratory rate in newborn?A. 22B. 30C. 18D. *45E. 64

64. What main clinical features are useful in the diagnosis of bronchiolitisA. Paroxysmal coughB. *WheezingC. TachypneaD. DyspneaE. Tension and flaring of the nostris

65. What main X-ray features are useful in the diagnosis of acute bronchitis?A. *Perivascular and peribronhial infiltrationB. Particularly clear lung fieldC. Hyperinflation D. Occasional scattered areas of consolidationE. Local infiltration of lung tissue

66. What examination is most important in the case of bronchitis?A. Complete blood countB. Culture of sputumC. Culture of alveolar fluidD. Biochemical examination of the blood E. *Chest X-ray

67. Bronchitis is caused most often by:A. FungiB. *VirusesC. BacteriaD. ParasitesE. Mixed flora

68. All factors can lead to bronchitis, except:A. CoolingB. *Poor feedingC. AllergiesD. Genetically predilectionE. Bad ecology

69. What sound is typical for obstructive bronchitis? A. *Clear pulmonary soundB. Box soundC. Local shortness of pulmonary sound D. Total shortness of pulmonary soundE. Mosaic sound

70. Such auscultation picture may be determined at obstructive bronchitis, except:A. *Local decrease of vesicular breathing

B. Increase vesicular breathingC. Rough breathingD. Puerile breathingE. Bronchus breathing

71. What is the main feature of bronchiolitis?A. Puerile breathingB. Dry wrestling ralesC. Course bubbling rales in lower parts of lungsD. *Diffuse fine rales E. Decreased vesicular breathing

72. The criteria of acute obstructive bronchitis are all, except:A. *Severe intoxicationB. The great quantity of dry ralesC. Nonproductive coughD. Box sound during percussionE. Perivascular infiltration of lung tissue

73. The X-ray criteria of acute bronchitis is all, except:A. *AtelectasisB. Increase of lung patternC. Perivascular infiltrationD. Peribronchial infiltrationE. Infiltration of roots

74. According classification bronchitis cannot be:A. AcuteB. *LatentC. RelapseD. ChronicE. Bronchiolitis

75. The main symptom of acute bronchitis is:A. WheezingB. Pain in throatC. *CoughD. DyspneaE. Hyperthermia

76. The first sign of acute bronchitis is everything, except:A. HyperthermiaB. WheezingC. *VomitingD. PharyngitisE. Cough

77. There is not typical such type of rales during bronchitis:A. Diffuse B. Symmetrical C. *Local D. Decreasing or disappearing of rales after cougE. Dry rales

78. There is not typical such auscultation dates during bronchitis:A. Rough breathingB. *Decrease breathingC. Dry rales D. Moist diffuse ralesE. Decreasing or disappearing of rales after cough

79. The auscultation date during bronchitis is:

A. Puerile breathingB. Decrease breathingC. *Rough breathingD. Moist local ralesE. Crepitus rales

80. During acute bronchitis respiratory insufficiency is most often of:A. *0 stageB. 1 stageC. 2 stageD. 3 stageE. 2-3 stage

81. X-ray criterion of acute bronchitis isA. *Symmetrical increasing of lung patternB. Symmetrical decreasing of lung patternC. Infiltration of lung tissueD. Infiltration of lung tissue near rootsE. Symmetrical decreasing of lung pattern and infiltration of lung tissue

82. Complex of treatment of acute bronchitis does not include:A. MucoliticsB. PhysiotherapyC. *Antacids drugsD. Antipiretics drugsE. Antibiotics

83. Bronchoobstructive syndrome is characterized by:A. *Noisy breathing B. Paroxysmal breathingC. Stridor breathingD. Silent breathingE. Diminished breathing

84. What main clinical features are useful in the diagnosis of bronchial asthma?A. Chest pain B. *DispnoeC. TahycardiaD. VomitingE. Wet cough

85. What examination is the most important at bronchial asthma?A. Stools examinationB. Culture of sputumC. *Immunoglobulin E D. Biochemical examination of the blood E. Chest X-ray

86. What special substances may be found in sputum in case of bronchial asthma?A. *EosinophylsB. ErythrocytesC. NeuthrophylsD. LeucocytesE. Monocytes

87. What will be the therapeutic management of acute attacks of asthma?A. *BronchodilatorsB. Anti-inflammatory agentsC. Commonly sodiumD. Inhalation of corticosteroidsE. Prednizolon orally

88. There is necessary to use for control the effect of treatment of bronchial asthma:A. SpirographyB. ECGC. Allergic testsD. *Peak Flow MeterE. X-ray

89. More often the reason of bronchial asthma is theA. *Dust B. DrugsC. FoodD. BacteriaE. Viruses

92. What symptom is not specific for atrial septal defect?A. Difficulty breathing (dyspnea)B. *CyanosisC. Frequent respiratory infections in childrenD. Sensation of feeling the heart beat (palpitations)E. Shortness of breath with activity

93. Children with such pathology have a high risk of the congenital heart disease, except for:A. Down syndrome B. Genetic disorders (chromosomal abnormalities)C. Premature babies D. Children with multiple extracardiac (other than the heart defect) malformationsE. *Newborns with chronic intrauterine hypoxia

94. The most common symptoms of patent ductus arteriosus are:A. Tachycardia or other arrhythmiaB. Shortness of breath and other respiratory problemsC. *Continuous machine-like murmurD. Enlarged heartE. All mentioned above

95. What heart murmur is characteristic for patent ductus arteriosus?A. SystolicB. DiastolicC. Systolic and diastolicD. *Continuous machine-like murmurE. May be variants

96. What symptom is not typical for hypoplastic left heart?A. cyanosisB. cold extremitiesC. *hepatosplenomegalyD. LethargyE. Poor pulse

97. What symptoms are typical for aortic coarctation (aortarctia)?A. Dizziness or fainting B. Shortness of breathC. headacheD. Chest painE. *all that is mentioned

98. Attack of rheumatic fever is more frequent after the:A. fluB. sinusitisC. *scarlet feverD. measles

E. rhinitis99. Etiology of rheumatic fever is:

A. *streptococcusB. pneumococcusC. the virus of influenzaD. fungiE. staphylococcus

100. What does confirm streptococcal etiology of rheumatic fever?A. previous measles B. *previous acute tonsillitisC. previous influenzaD. previous sepsisE. previous pneumonia

101. What heart defect is most often formed on a background of rheumatic fever?A. *mitral valve insufficiencyB. stenosis of aorta valvesC. insufficiency of aortic valvesD. tricuspid valve insufficiencyE. stenosis of mitral valve

102. What is the main criterion of rheumatic fever?A. *carditisB. hepatitisC. nephritisD. pneumonia E. gastritis

103. What is the main criterion of rheumatic fever?A. nephritisB. hepatitisC. *carditisD. pneumonia E. gastritis

104. What is the main criterion of rheumatic fever?A. *polyarthritisB. hepatitisC. dermatitisD. pneumoniaE. gastritis

105. What is the main criterion of rheumatic fever?A. *rheumatic nodulesB. dermatitisC. nephritisD. pneumoniaE. gastritis

106. What is the criterion of carditis?A. *damage of myocardium and endocardiumB. cardialgiasC. damage of epicardiumD. only pericarditis E. hypothermia

107. What is typical for myocarditis?A. heart borders narrowedB. strengthening of the heart tonesC. syndrome WPW

D. СLС syndrome E. *heart borders dilatation

108. What is typical for myocarditis?A. *decrease of arterial pressureB. increase of arterial pressureC. normal arterial pressure D. high pulse pressure E. СLС syndrome

109. For rheumatic polyartritis is typical:A. morning stiffnessB. constant deformations of the jointsC. *inconstant damage of jointsD. damage of spineE. damage of neck

110. Small chorea is characterized for..A. violation of consciousnessB. central paralysesC. damage of the cranial-cerebral nervesD. *muscular hypotoniaE. muscular hypertonia

111. Rheumatic endocarditis is characterized by:A. *rough systolic murmur on the apexB. accent of 2 tone above the aortaC. soft systolic murmur on the apexD. systolic murmur above the pulmonary arteryE. soft systolic murmur on the aorta

112. ECG in case of rheumatism characteristically is characterized by:A. lengthening of the QT interval B. *lengthening of the PQ intervalC. deformation of the QRS complexD. increase of voltageE. decrease of voltage

113. In the case of the rheumatism's acute duration is better to use:A. plaquenilB. delagylC. ibuprophenD. *aspirinE. analgin

114. In the case of the rheumatism's prolonged duration is better to use:A. voltaren B. *plaquenilC. ibuprophenD. aspirinE. analgin

115. What is prescribed in case of penicillin allergy for rheumatism etiological treatment?A. gentamycinB. claforanC. chloramphenicolD. kanamycin E. *erythromycin

116. What is the dose of prednisolone in case of severe rheumatic carditis?A. 1 mg/kg;B. *2 mg/kg;

C. 3 mg/kg;D. 4 mg/kg;E. 5 mg/kg

117. What is the complication of rheumatic fever?A. *mitral valve insufficiency B. tricuspid valve insufficiency C. stenosis pulmonary arteryD. pulmonary artery valves insufficiency E. pulmonary artery valves stenosis

118. What is typical for myocardiosclerosis? A. *arrhythmia B. low blood pressure C. high blood pressure D. bradypneaE. tachypnea

119. What is typical for rheumatic arthritis?A. Morning stiffnessB. Permanent courseC. *Symptoms subside within 2 weekD. Pale skin over jointsE. All transffered

120. Arthralgia associated with rheumatic fever differs from arthralgia associated with rheumatoid arthritis by:A. Permanent courseB. *Absence of tenderness during passive movement of the affected jointC. More expressed in the morningD. More expressed in the eveningE. Increases day by day

121. Sydenham’s chorea is characterized by involuntary movements, specially:A. On the legsB. On the handsC. On the faceD. *On the face and limbsE. All over the body

122. Clinic of Sydenham’s chorea includes:A. Concomitant psychological dysfunctionB. Increased emotional labialityC. HyperactivityD. Age-regressed behaviourE. *All transferred

123. Name wrong assertion about annular erythema at rheumatic feverA. Pale pink rash in the form of a thin ring-shaped rimB. Does not rise above the skin surfaceC. Disappears when pressingD. *Appears in 30-40% of children with rheumatismE. Appears mostly at an altitude of disease and usually is fragile

124. Choose WRONG assertion about diagnostic of rheumatic feverA. No exact diagnostic test has been developed to test for its presenceB. *Only blood tests to check for the presence of a strep infection are helpfulC. A careful exam by a qualified medical practitioner is necessaryD. Just as there is no particular laboratory test to diagnose Rheumatic fever E. Throat swab for culture, Antistreptolysin O titre (ASOT) and blood for acute phase

reactants must be done

125. The duration of salicylates treatment at rheumatic fever is:A. 1 weekB. 2 weeksC. *3-4 weeksD. 6 weeksE. 2 months

126. Name the main treatment of rheumatic carditisA. CortiscoteroidsB. SalicylatesC. VasodilatorsD. DigoxinE. *All that is mentioned

127. Name the main treatment of Sydenham’s chorea.A. CorticosteroidsB. SalicylatesC. VasodilatorsD. *HaloperidolE. Diuretics

128. The acute phase of rheumatic fever lasts in 75% of cases:A. 2 weeksB. 4 weeksC. *6 weeksD. 2 monthsE. 3 months

129. What is primary prevention of rheumatic fever?A. Periodic examination of people who have a family history of Rheumatic feverB. Good nutritionC. Obligatory antibiotic therapy after dental operationsD. *Treatment of the streptococcal upper respiratory infection with antibioticsE. All that is mentioned

130. Patients without rheumatic carditis need a secondary prophylaxis:A. 2 yrsB. 3 yrsC. *5 yrsD. 10 yrsE. Whole life

131. What is not acquired heart diseases in children?A. Mitral insufficiency B. *Coarctation of aortaC. Tricuspid insufficiencyD. Mitral stenosisE. Aortic stenosis

132. Pathogenesis of mitral insufficiency includes:A. Constant retrograde flow of blood in the left atrium during systole of the lef ventricleB. Expansion and hypertrophy of the left ventricleC. Stagnation in the pulmonary veinsD. Decompositions of right ventricular stagnationE. *All that is mentioned

133. Clinic of mitral insufficiency includes:A. WeaknessB. Poor appetiteC. Pale skinD. *Shortness of breath

E. Nothing that is mentioned134. Clinic of mitral insufficiency includes:

A. WeaknessB. PalpitationsC. Pasty legs and feetD. *All that is mentionedE. Nothing that is mentioned

135. Name the most common rheumatic heart diseaseA. Mitral insufficiency B. Coarctation of aortaC. Tricuspid insufficiencyD. *Mitral stenosisE. Aortic stenosis

136. Name auscultation data at mitral stenosisA. *The first heart sound is unusually loudB. The second heart sound is unusually loudC. The first heart sound is unusually dullD. The second heart sound is unusually dullE. Heart sounds are normal

137. Name auscultation data at mitral stenosis.A. Mid-diastolic rumbling murmurB. *Mid-systolic rumbling murmurC. Mid-systolodiastolic rumbling murmurD. Blowing systolic murmurE. Blowing diastolic murmur

138. Clinic at mitral stenosis includes:A. Butterfly rashB. ArthralgiaC. *Ankle/sacral edemaD. AscitesE. All transferred

139. Name changes of arterial pressure at aortic insufficiencyA. The maximum arterial pressure is dramatically reduced with high minimumB. *The minimum arterial pressure is dramatically reduced with high maximum C. High arterial pressure on hands and low on legsD. Arterial hypertension E. May be different variants

140. The most characteristic feature in tricuspid insufficiency is:A. Systolic murmurB. Diastolic murmurC. Loud heart tonesD. *Pulsation of the neck veins and the liverE. Peripheral oedema

141. Cardiac catheterization provides in tricuspid insufficiency an opportunity to identify:A. Size of defectB. DiagnoseC. Presence of hypertrophyD. *High pressure in the right atrium and the portal veinsE. All that is mentioned

142. On the first place as ethiologic factor at nonrheumatic carditises are:A. BacteriaB. *VirusesC. Viruses + bacteria

D. FungiE. Alergic reactions

143. What is the main clinical feature of early congenital carditis?A. *Progressive left-heart cardiac insufficiency, refraction to the therapyB. Physical and psychomotor retardationC. TachycardiaD. Occurs under influence of harmful factorsE. ECG: high R

144. Fibroelastosis is a result of: A. Viral carditisB. Bacterial carditisC. Fungi carditisD. *Early congenital carditisE. Late congenital carditis

145. What is used for treatment of acute nonrheumatic carditis?A. *Glucocorticoids+ nonsteroid anti-inflammatory drugsB. Chinoline derivatives+ nonsteroid anti-inflammatory drugs C. AntibioticsD. VitaminsE. All transferred

146. What dose of Heparin is used at treatment of nonrheumatic carditis?A. 50 U/kgB. *100 U/kgC. 150 U/kgD. 200 U/kgE. 250 U/kg

147. Name the wrong assertion about cardiomyopathy.A. Cardiomyopathy is a chronic and progressive disease in which the heart muscleB. *abnormally enlarged, thickened and stiffenedC. Cardiomyopathy affects only children at any stage of their lifeD. It is not gender, geographic, race or age specificE. It is a rare disease when diagnosed in infants and young childrenF. Cardiomyopathy continues to be the leading reason for heart transplants inG. ldren

148. What type of cardiomyopathy does not exist?A. Restrictive cardiomyopathy B. Dilated C. HypertrophicD. *Hypotrophic E. Ischemic cardiomyopathy

149. What is the main symptom of cardiomyopathy?A. *ArrhythmiaB. FatigueC. Difficulty breathingD. Poor appetiteE. All transferred

150. What investigation can differentiate between hypertrophic, restrictive or dilated cardiomyopathy in most cases?A. 24-hour Holter monitorB. ECGC. *EchoCGD. X-ray of chest E. All that is mentioned

151. What is the most common form of cardiomyopathy?A. *Dilated B. Restrictive C. HypertrophicD. InfiltrativeE. Ischemic

152. Basis of therapy of hypertrophic cardiomyopathy is:A. Calcium antagonists of Verapamilum groupB. Metoprolol succinateC. PropranololD. AtenololE. *All that is mentioned

153. Basis of therapy of hypertrophic cardiomyopathy is:A. *Calcium antagonists of Verapamilum groupB. Antiunrhythmical preparations  C. inhibitors of ACE (angiotensin converting enzyme)D. Cardiac glycozidesE. All that is mentioned

154. Surgical treatment of hypertrophic cardiomyopathy is indicated at:A. Immediately after diagnoseB. In early ageC. *Absence of clinical effect from active medicinal therapyD. Appearance of heart failureE. All that is mentioned

155. Specify one of symptoms of acute left heart insufficiency.A. Swelling of neck veinsB. Skin cyanosisC. *Foamy excretions from mouth D. HepatomegaliaE. Edema on extremities

156. Specify one of symptoms of acute left heart insufficiency.A. OlyguriaB. Expansion of right border of heart C. Decline of arterial pressure D. *Forced sitting position of bodyE. Edema on extremities

157. At what degree of cardiac insufficiency does ascites appear?A. I A B. I B C. II A D. II BE. *III

158. Specify indications for prescription of cardiac glycosides.A. *Cardiac insufficiency B. Atrioventricular blockade C. Expressed bradycardiaD. Group extrasystolesE. Expressed hypokaliemiaF. 159. At what type of collapse is necessary to include mezatoni and adrenalini in

urgent therapy?A. ToxicB. SympathotonicC. *Paralytic

D. OrtostaticE. Hypoxic159. At what type of collapse is necessarily to include aminasini in urgent therapy?

A. ToxicB. ParalyticC. HypoxicD. *SympathotonicE. Ortostatic

160. Ratio of indirect heart massage and artificial respiration for children of all age groups is:A. 2:1B. 3:1C. *4:1D. 5:1E. 6:1

161. Name one of symptoms of acute left heart insufficiency.A. Edema of extremitiesB. Accent of 2nd tone on a pulmonary artery C. Swelling of neck veinsD. HepatomegalyE. *Weak pulse or undetermined

162. Position of patient at syncope must be:A. Horizontal with the low position of feetB. *Horizontal with high position of feet C. Half sitting with the inclined head to the rightD. Half sitting with the inclined head to the leftE. Raised with the low position of feet

163. The best medicines at symphatotic collapse are:A. Cordiamini, caffeiniB. Eyphyllini, salbutamoliC. *Droperidoli,aminasiniD. Adrenalini, mezatoniE. Reopolyglucini, polyglucini

164. The best medicines at vagotonic collapse are:A. *Adrenalini, mezatoniB. Droperidoli,aminasini C. Eyphyllini, salbutamoliD. Cordiamini, caffeine E. Reopolyglucini, polyglucini

165. Paroxismal tachycardia is the attack of sudden tachycardia:A. More than 90-100 per minuteB. More than 110-130 per minuteC. More than 130-140 per minute D. *More than 150-180 per minuteE. More than 200 per minute

166. The general electrocardiography criteria of paroxysmal tachycardia are: A. Presence 3 and anymore groups of extrasystolesB. Absence of compensating pause C. C. Cardiac frequency more than 150 per 1 minuteD. *Outbreak and sudden endE. All that is mentioned

167. The first medicine at treatment of ventricular paroxysmal tachycardia is:A. Cordaroni B. Novocainomidi

C. CorgluconiD. *LidocainiE. Seduxeni

168. Symptomatic hypertension in children is conditioned:A. By illnesses of kidneysB. By the anomalies of vesselsC. By illnesses of adrenal glands D. By nothing of theseE. *By all of these

169. Basic in determination of clinical death is:A. Absence of the independent breathingB. Absence of photoreaction of pupils on lightC. *Permanent expansion of pupilsD. Absence of pulse on carotid and femoral arteriesE. All of these

170. What drug is possible to enter only intracardiac?A. LidocainiB. Atropini C. *Adrenalini D. All that is mentioned E. sodium hydrocarbonatum

171. What is the best method for diagnostic of arrhythmias?A. ECGB. *Holter-monitoringC. EchoCGD. Heart catheterizationE. All transferred

172. Chronic cardio-vascular insufficiency (heart failure) is classified as:A. Left heart, right heart, arythmogenes, total B. Systolic, diastolic, mixed C. *On stages: I-A, I-B, II-A, II-B,IIID. Left heart, right heart, systolic, diastolicE. All of these

173. Ventricular tachycardia is indeed the most dangerous of the cardiac arrhythmias with a real risk of:A. Infarction B. *Sudden cardiac deathC. MyocardiosclerosisD. Heart failureE. All transferred

174. What type of tachycardia does not exit?A. Supraventricular tachycardiaB. Ectopic atrial tachycardiaC. *Restrictive tachycardiaD. Ventricular tachycardiaE. All exit

175. ?What systemic connective tissue disease is on the first place of prevalence in children?A. Systemic lupus erythematosusB. *Rheumatoid arthritis C. DermatomyositisD. Periarteritis nodosaE. Scleroderma

176. How many criteria out of ten (according criteria) are enough to put the diagnosis of SLE?A. 3B. *4C. 5D. 6E. 7

177. Affection of the eyes is specific for:A. Infectious arthritis B. Acute rheumatic leverC. systemic lupus erythematosusD. *Juvenile rheumathoid arthritis E. Osteomyelitis

178. The dose of aspirin for the treatment of juvenile rheumatoid arthritis is:A. 10-20 mg/kg/day B. 25-50 mg/kg/dayC. *50-120 mg/kg/day D. 100-120 mg/kg/day E. 150-200 mg/kg/day

179. The dose of ibuprofen for the treatment of juvenile rheumatoid arthritis is:A. *10-30 mg/kg/day B. 30-50 mg/kg/dayC. 50-120 mg/kg/day D. 100-120 mg/kg/day E. 150-200 mg/kg/day

180. The “gold standard” of JRA treatment considered to be:A. AspirinB. CyclophosphamideC. AzathioprineD. *MethotrexateE. Hydroxychloroquine

181. How long should be present arthritis in patient to diagnose JRA (according WHO criteria)?A. More than 1 monthB. More than 2 monthsC. *More than 3 monthsD. More than 6 monthsE. More than 1 year

182. The main clinical criterion of juvenile rheumatoid arthritis is:A. *Arthritis more than 3 months B. Morning stiffness of jointsC. Symmetrical affection of small jointsD. Effusion in joint cavity (under the capsule)E. Joint contracture.

183. Among clinical criteria of juvenile rheumatoid arthritis is NOT present:A. Arthritis more than 3 months B. Morning stiffness of jointsC. Symmetrical affection of small jointsD. Effusion in joint cavity (under the capsule)E. *Subfebrile temperature

184. What joints are the most frequently damaged in JRA?A. *KneeB. AnklesC. WristsD. Elbow

E. Hip185. Choose the most typical sign of rheumatoid arthritis in children in difference from adults:

A. Symmetrical involvement of small jointsB. Rheumatoid nodulesC. *Damage of neck and mandible jointsD. Resistance to the treatmentE. Rare mono- or pauciarticular types development

186. What is NOT characteristic for the damage of mandible joint at JRA?A. Limited possibility to open the mouseB. Further lower jaw dysplasiaC. Pain during opening the mouseD. Crepitating at palpationE. *Edema of parotid area

187. What form is absent in the JRA classification?A. SystemicB. PauciarticularC. Still’s syndromeD. PolyarticularE. *Waterhouse-Friderichsen syndrome

188. What is Still's syndrome?A. *Systemic form JRAB. Pauciarticular form JRAC. Subclinical form JRAD. Damage of spine in JRAE. Damage of eyes in JRA

189. Choose the most typical sign of Still's syndrome?A. Muscular atrophyB. *Salmon pink rush on the bodyC. Symmetrical involvement of small jointsD. Eyes involvementE. Presence of rheumatoid nodules

190. Choose the most typical sign of Still's syndrome?A. Muscular atrophyB. Iritis, uveitisC. Symmetrical involvement of small jointsD. *FeverE. Presence of rheumatoid nodules

191. For polyarticilar form of JRA typical is damage of more than:A. 1 jointB. 3 jointsC. *5 jointsD. 10 jointsE. 4 joints

192. What factors contribute to the immune system disturbances observed in lupus?A. Genetic predispositionB. Hormonal imbalanceC. Allergens (eggs, drugs)D. Sunlight exposureE. *All mentioned above

193. What is the daily dose of prednisolone (mg) for pulse-therapy?A. 50B. 100C. 250

D. 500E. *1000

194. What daily dose of prednisolone (mg/kg) should be given for treatment of polyarteritis nodosa with abdominal syndrome?A. 0,5 - 0,75B. 1 - 2C. 2,5 - 5D. *5 - 7E. 10

195. What average daily dose of prednizolne (mg/kg) shoild be given for polyarteritis nodosa treatment?A. 0,5 - 0,75B. *1 - 2C. 2,5 - 3D. 5-7E. 10

196. What is the most frequent cardio-vascular system involvement symptom at polyarteritis nodosa?A. *Ischemia of myocardiumB. FibroelastosisC. MyocarditisD. MyocardiosclerosisE. Pancarditis

197. The characteristic skin changes at polyarteritis nodosa are namedA. *Livedo reticularisB. Nodular erythemaC. Annular erythemaD. «Butterfly»-rashE. Heliotrope erythema

198. What is NOT the diagnostic criterion of polyarteritis nodosa?A. *Annular (ring-form) erythemaB. Loss more than 4 kg of body weight during the monthC. Diffuse myalgiaD. PolyneuropathyE. Increased level of urea in the blood

199. What from enumerated is NOT typical for dermatomyositis?A. LeucocytosisB. CreatininuriaC. Increased activity ALT, АSТD. Presence of MSAE. *Lymphocytosis

200. What is NOT included in Peter’s and Bohan’s criteria of juvenile dermatomiositis?A. Symmetrical proximal muscle weaknessB. Elevated serum enzymes (CK,CPK, LDH, and/or aldolase)C. Abnormal EMG (abnormal activity and muscle movements)D. Inflammation or necrosis on muscle biopsyE. *Hypertermia and intoxication syndrome

201. What is NOT included in Peter’s and Bohan’s criteria of juvenile dermatomiositis?A. Symmetrical proximal muscle weaknessB. Characteristic skin eruptionC. Abnormal EMG (abnormal activity and muscle movements)D. Inflammation or necrosis on muscle biopsyE. *All signs mentioned above are among criteria

202. Heart pain at dermatomyositis is more frequent caused by all enumerated, EXEPT:

A. Intercostal myositisB. *StenocardiaC. CoronaritisD. Necrosis of myofibrilsE. Pericarditis

203. Limitation of motions at patients with dermatomyositis is due to:A. Flexors hypertoniaB. Atrophy of musclesC. *Pain in musclesD. Pain in jointsE. Damage of innervation

204. The characteristic skin changes at dermatomyositis are named:A. Livedo reticularisB. Nodular erythemaC. Annular erythemaD. «Butterfly»-rashE. *Heliotrope erythema

205. Calcinosis at dermatomyositis can develop mainly in:A. KidneysB. Cardiac musclC. LungD. *Perypheral muscleE. Joints

206. Dermatomyositis is more frequently seen in:A. *GirlsB. BoysC. NewbornsD. PreschoolersE. Teenagers

207. What of the following is NOT the systemic scleroderma symptom?A. Raynaud's phenomenon B. *Weakness of the proximal muscles C. Thickening, hardening, and discoloration of the skin D. Ulcers in the oral cavityE. Swelling of the fingers, hands, forearms

208. What of the following is the systemic scleroderma symptom?A. Symmetrical proximal muscle weaknessB. *Thickening, hardening, and discoloration of the skinC. Abnormal EMG (abnormal activity and muscle movements)D. Inflammation or necrosis on muscle biopsyE. Leukocytosis and eosynophilia in peripheral blood

209. What medicine is used for the basic therapy of systemic scleroderma?A. DelagylB. *PenicillamineC. MethotrexateD. IbuprofenE. Prednisolone

210. ?Etiology of chronic gastroduodenitis is:A. StreptococcusB. StaphylococcusC. *Helicobacter pyloriD. E. coliE. Candida

211. Name aggressive factor of the stomach mucus membrane damage:A. *HClB. Mucus productionC. Prostaglandin Е2D. BicarbonatesE. NaCl

212. Name aggressive factor of the stomach mucus membrane damage:A. *PepsinB. Mucus productionC. Prostaglandin Е2D. BicarbonatesE. KCl

213. The regulator of the hydrochloric acid secretion is:A. SomastatinB. GlucagonC. *GastrinD. InsulinE. Bicarbonates

214. Name the factor of the stomach mucus membrane protection:A. PepsinB. HClC. *Mucus productionD. GastrinE. NaCl

215. Pathogenesis of the gastritis development is:A. Increase of mucus productionB. *Increase of the salt acid secretionC. Increase of bicarbonates secretion D. Physiological blood stream in a mucus membraneE. Decrease of the salt acid secretion

216. The secret of cells of the stomach mucus membrane consists of:A. * HClB. BicarbonatesC. Mucus D. EndorphinsE. Blood

217. Helicobacter pylori is the cause of:A. EsophagitisB. *GastritisC. ColitisD. HepatitisE. Uretritis

218. Helicobacter pylori is colonized at the:A. Cardial part of the stomachB. Fundal part of the stomachC. Pyloric part of the stomachD. *Antral part of the stomachE. Esophagus

219. What enzyme is produced by Helicobacter pylori?A. LactaseB. *UreaseC. PeptidaseD. LipaseE. Amylase

220. Product of urea hydrolyses in the gastric juice is:A. OxygenB. *AmmoniumC. NitrogenD. ChlorineE. HCl

221. What neutralizes the ammonia in the stomach juice?A. PepsinB. *HClC. BicarbonatesD. MucusE. Ammonium

222. Name main syndromes of chronic gastritis:A. Pain, dyspepsic, hemorrhagicB. *Pain, dyspepsic, intoxicationC. Disuric, dyspepsic, intoxicationD. Pain, hemorrhagic, dysuricE. Pain, dyspepsic, dysuric

223. At patients with hyperacidic gastritis pain arises up through:A. * 15-30 min. after a mealB. 30-45 min. after a mealC. 45-60 min. after a mealD. 1-1.5 hours after a mealE. 2-2.5 hours after a meal

224. In the case of increased secretion in patients with gastritis more prominent is:A. *PainB. DyspepsiaC. IntoxicationD. DysuriaE. Hemorrhagic syndrome

225. In case of increased secretion in patients with gastritis pain is:A. MildB. DullC. ModerateD. *IntensiveE. No intensive

226. In case of decreased secretion in patients with gastritis pain is:A. intensiveB. *mildC. acuteD. absentE. moderate

227. At patients with duodenitis pain arises up more frequent through:A. 15-30 min. after a mealB. 30-45 min. after a mealC. 45-60 min. after a mealD. *1-1.5 hours. after a mealE. 2-2.5 hours. after a meal

228. Most frequent localization of pain at gastritis is: A. In mesogastrium B. Right hypochondriaC. Left hypochondriaD. *Epigastria

E. Hypogastria229. Most frequent localization of pain at duodenitis is:

A. In mesogastrium B. Right hypohondriumC. *Pyloroduodenal areaD. Left hypochondriaE. Hypogastria

230. Pain on empty stomach is characteristic for patients with: A. *Increased acid production B. Normal acid productionC. Decreased acid productionD. AchlorhydriaE. Hypochlorhydria

231. Nightly pains are characteristic for patients with: A. *Increased acid production B. Normal acid productionC. Decreased acid productionD. AchlorhydriaE. Hypochlorhydria

232. Predominance of pain above a dyspepsia syndrome is characteristic for gastroduodenitis with:A. *Increased acid production B. Normal acid productionC. Decreased acid productionD. AchlorhydriaE. Hypochlorhydria

233. Predominance of dyspepsia above a pain syndrome is characteristic for gastroduodenitis with:A. Increased acid production B. Normal acid productioC. *Decreased acid productionD. AchlorchydriaE. Hypochlorhydria

234. Feeling of overweight in epigastria is characteristic for patients with: A. Increased acid production B. Normal acid productionC. *Decreased acid productionD. AchlorhydriaE. Hypochlorhydria

235. Meteorism is characteristic for gastritis with: A. Increased acid production B. Normal acid productionC. *Decreased acid productionD. AchlorhydriaE. Hypochlorhydria

236. Constipation is characteristic for gastritis with: A. *Increased acid production B. Normal acid productionC. Decreased acid productionD. AchlorhydriaE. Hypochlorhydria

237. Secondary gastroduodenitis develops more frequently on the background of:A. Food poisoningB. Helicobacter pylori persistencyC. Parasites invasion

D. *Other chronic diseases of digestive organsE. Gastroenteral enzymopathy

238. Chronic gastroduodenitis more frequently begins at children in:A. The first month of lifeB. The first year of lifeC. *Preschool ageD. First three years of lifeE. School age

239. Chronic gastroduodenitis more frequently combines with diseases of:A. CNSB. *Hepatobiliary systemC. Respiratory systemD. Heart and vesselsE. Urinary system

240. What syndrome is the most constant in case of chronic gastroduodenitis at children?A. *PainB. DyspepsicC. IntoxicationD. EpithelialE. Hemrrhagic

241. The intensity of dyspepsia at chronic gastroduodenitis in children depends on:A. *Secretory functionB. Age of childC. Peculiarities of feedingD. Intervals between the receptions of mealE. Balanced food ingredients

242. In case of the decreased stomach juice acidity more frequently is observed:A. ConstipationB. *DiarrheaC. Intensive painD. “Hungry” painE. Vomiting

243. In case of the decreased stomach juice acidity is not characteristic:A. ConstipationB. *DiarrheaC. Pain in epigastriumD. Belch (regurgitationE. Nausea

244. Classification of the chronic gastroduodenitis by the etiology is:A. *Primary and secondaryB. Inborn and acquiredC. Postinfectious and alimentaryD. Widespread and limitedE. Allergic and toxic

245. What secretory function in gastritis is the most characteristic for children?A. DecreasedB. Decreased or normalC. *Increased or normalD. DecreasedE. Served

246. What from the instrumental methods is most informative for the diagnostics of chronic gastroduodenitisA. *Esophagogastroduodenoscopy

B. рН-metryC. Breath testD. ColonoscopE. X-ray of abdomen

247. In children most rare chronic gastroduodenitis according the endoscopic description is:A. SuperficialB. HemorrhagicC. *AtrophicD. MixedE. Erosive

248. What from the helicobacteriosis diagnostics methods belongs to invasive?A. *Biopsy of the stomach mucus membraine with its bacteriological investigationB. PCR of feces and salivaC. Respiratory testsD. Determination of specific immunoglobulinesE. "Aerotest"

249. Treatment of children with chronic gastroduodenitis must be (choose the most perfect answer):A. Prolonged and intensiveB. PermanentC. *Complex and phasedD. StationaryE. Ambulatory

250. What does determine duration of the bed regime in case of chronic gastroduodenitis in children?A. Severity of dyspepsiaB. Type of secretionC. *Severity of pain syndromeD. Age of childE. Endoscopic changes

251. What diet is appointed at the exacerbation of chronic gastroduodenitis?A. № 1B. № 5C. № 10D. № 9E. *№ 1A, 1B

252. What from antacids is the most reasonable to appoint to the children?A. Sodium hydrocarbonateB. *Aluminium hydrate + Magnesium hydrochlorideC. Calcium carbonateD. Magnesium hydrochloride + Carbonic acidE. Carbonic acid

253. What from the medicine belongs to Н2 histamine-blockers?A. *CimetidinB. MaaloxC. VicalinD. MethacinE. Gastropharm

254. What from medicine belongs to procinetics?A. *DomperidonB. PanzynormC. RenegastD. BellasponE. Ranitidin

255. What physical therapy procedure is reasonable to apply to the children with chronic gastroduodenitis with the expressed pain syndrome?A. Ozocerite appliquesB. Probbing with xylitolC. *Electrophoresis with novocaineD. DiathermyE. Balneotherapy

256. What from the named medicine is used for Helicobacter pylori eradication?A. *De-nolB. MaaloxC. MotiliumD. CimetidinE. Benzylpenicilliun

257. The leading symptom of the 1st stage of peptic ulcer disease is:A. *PainB. VomitingC. Belch (regurgitation)D. HeartburnE. Local tension of muscles

258. The sequence of symptoms at the Moinighan rhythm is the following:A. The intake of meal - pain - hunger - reliefB. *Hunger - pain - the intake of meal - reliefC. Pain - hunger - the intake of meal - reliefD. Hunger - relief - the intake of meal - painE. The intake of meal - relief - pain - hunger

259. ?Name the forms of chronic hepatitis at children:A. Persistent, active, autoimmuneB. *Viral, autoimmune, medicinal-induced, toxicC. Medicinal, autoimmune, alcoholicD. Cryptogenic, viral, toxicE. Cryptogenic, autoimmune, toxic, alcoholic

260. Phases of chronic hepatitis activity are:A. Active (minimum, moderate, expressed)B. Active (mild, moderate, severe) and nonactivC. *Active (minimum, moderate, expressed) and nonactivD. Active, incomplete clinic-laboratory remission, complete clinic-laboratory remissionE. Incomplete clinic-laboratory remission, complete clinic-laboratory remission

261. Amylase level of blood is:A. 10-15 g/hour/lB. *16-32 g/hour/lC. 33-45 g/hour/lD. 46-60 g/hour/lE. 60-90 g/hour/l

262. To the necessary biochemical researches for patients with the diseases of hepatobiliary system belong:A. General albumen, CRP, seromucoid, bilirubinB. *General protein and fractions, transaminases, bilirubin, cholesterolC. General protein and fractions, urea, creatinin, cholesterolD. General protein and fractions, CRP, seromucoid, ureaE. General protein and fractions, CRP, cholesterol

263. The syndrome of "cytolysis" includes:A. *Increase of the АsАТ level, АlАТ, LDG, iron, bilirubinB. Increase level of cholesterol, iron, LDG

C. Decrease level of proteins, cholesterol, bilirubin, СRPD. Decrease level of iron, СRP, remaining nitrogen, АsАТ, АlАТE. Decrease of the АsАТ level, АlАТ, LDG, iron, bilirubin

264. Name basic clinical signs of cholestasis syndrome: A. *Jaundice, skin itching B. Jaundice, hepatosplenomegalyC. Pallor, jaundice, hepatosplenomegalyD. Stomach-aches, jaundiceE. Pallor, skin itching

265. Markers of the chronic hepatitis С are:A. HBe Ag, HBs Ag, HBV-DNA, anti-HBe Ig G, anti-HBs Ig GB. HAV-RNA, anti-HAV Ig M, anti-HAV Ig GC. *HСV-RNA, anti-HСV Ig MD. HBs Ag, HDV-RNA, anti-HDV Ig M, anti-HBe Ig ME. HGV-RNA, anti-E2 HGV

266. Markers of the chronic hepatitis D are:A. HBe Ag, HBs Ag, HBV-DNA, anti-HBe Ig G, anti-HBs Ig GB. HAV-RNA, anti-HAV Ig M, anti-HAV Ig GC. HСV-RNA, anti-HСV Ig MD. *HBs Ag, HDV-RNA, anti-HDV Ig M, anti-HBe Ig ME. HGV-RNA, anti-E2 HGV

267. Markers of the chronic hepatitis G are:A. HBe Ag, HBs Ag, HBV-DNA, anti-HBe Ig G, anti-HBs Ig GB. HAV-RNA, anti-HAV Ig M, anti-HAV Ig GC. HСV-RNA, anti-HСV Ig MD. HBs Ag, HDV-RNA, anti-HDV Ig M, anti-HBe Ig ME. *HGV-RNA, anti-E2 HGV

268. What color of feces is in case of viral hepatitis?A. Bright-yellowB. Dark-brownC. *Clays colorD. Mud-color E. Black

269. What medicine does belong to interferons?A. EssentialeB. CholenzymC. *Intron AD. CarsylE. Prednisolon

270. Viferon is the medicine of group:A. GlucocorticoidsB. AntibioticsC. AntihystamineD. *InterferonE. Vitamins

271. Main treatment of autoimmune hepatitis is: A. Antiviral therapyB. *Immunesuppressive therapyC. Antibiotic therapyD. ImmunomodulatorsE. Antihystamines

272. What medicine with immune suppressive effect will you use for treatment of chronic hepatitis?A. *Prednisolon

B. VoltarenC. CycloferonD. EssentialE. Vitamins

273. What medicine does belong to immune suppressive therapy of chronic hepatitis?A. IndomethacinB. *AzatioprinC. PenicillinD. CholenzymE. Vitamins

274. Primary dyskinesia of bile ducts is connected with:A. Pathology of stomachB. Pathology of duodenumC. Pathology of large intestineD. *Neurohumoral regulation dysfunctionE. Pathology of esophagus

275. Secondary dyskinesia of bile ducts is connected with:A. Neurohumoral regulation dysfunctionB. Disorder of the vegetative nervous systemC. *Pathology of stomach and duodenumD. Pathology of hypophysisE. Pathology of esophagus

276. At the decreased synthetic function of liver is observed:A. *Hypoalbuminemia, decreased levels of fibrinogen and protrombinB. Hyperbilirubinemia, decreased levels of fibrinogen and protrombinC. Hypoalbuminemia, hypercholesterolemia, hyperazotemiaD. Hyperbilirubinemia, hypercholesterolemia, hyperazotemiaE. Hyperalbuminemia, decreased levels of fibrinogen and protrombin

277. About the decreased detoxication function of liver testifies:A. Hyperbilirubinemia, hypoproteinemia, hypoazotemiaB. Hyperbilirubinemia, hypoproteinemia, hypophenolemiaC. *Hyperazotemia, hyperphenolemia, hyperamiakemiaD. Hypoazotemia, hyperphenolemia, hypoamiakemiaE. Hypoazotemia, hypophenolemia, hyperamiakemia

278. Ultrasonography signs of hypotonic form of bile ducts dyskinesia are:A. Increase of liverB. *Enlarged goal bladderC. Goal bladder is diminished in sizesD. Diminishment of liverE. Decrease of liver

279. Ultrasonography signs of hypertonic form of bile ducts dyskinesia are:A. Increase of liverB. Diminishment of liverC. *Goal bladder is diminished in sizesD. Enlarged goal bladderE. Decrease of liver

280. In treatment of hypotonic form of bile ducts dyskinesias apply:A. AnalgeticsB. SpasmoliticsC. *Choleretics and cholekineticsD. HepatoprotectorsE. Vitamins

281. In treatment of hypertonic form of bile ducts dyskinesias apply:

A. AnalgeticsB. *Spasmolitics and sedativesC. AntibioticsD. HepatoprotectorsE. Vitamins

282. Etiology of chronic cholecystocholangitis is:A. StreptococcusB. VirusesC. *E. coliD. MicopolasmsE. Staphylococcus

283. Chronic cholecystocholangitis is the chronic inflammation of:A. Stomach and duodenuB. Stomach and goal bladderC. *Goal bladder and bile ductsD. Goal bladder and pancreasE. Stomach and bile ducts

284. Leading syndromes in case of chronic cholecystocholangitis are:A. Pain, dysuriaB. *Pain, dyspepsiaC. Toxic, hemorrhagicD. Dyspepsia, dysuriaE. Pain, toxic

285. Presence of vascular asterisks is characteristic for:A. GastritisB. DuodenitisC. *Cholecystocholangitis D. Bile ducts dyskinesiaE. Colitis

286. Toxic syndrome is more characteristic for: A. Bile ducts dyskinesiaB. *Cholecystocholangitis C. GastritisD. DuodenitisE. Colitis

287. Enlargement of liver is typical for:A. Bile ducts dyskinesia B. *Cholecystocholangitis C. GastritisD. PancreatitisE. Colitis

288. Medicine of choice at the cytolitic syndrome is:A. *EssentialeB. CholenzymC. InterferonD. No-spaE. Papaverin

289. Medicine of choice at cholestasis is:A. EssentialeB. CarsilC. No-spaD. *CholenzymE. Papaverin

290. Medicine of choice at the low synthetic function of liver is:A. EssentialeB. *CarsylC. CholenzymD. InterferonE. Papaverin

291. Distension of the skin capillaries on the back is characteristic for:A. Bile ducts dyskinesiaB. *Cholecystocholangitis C. GastritisD. DuodenitisE. Colitis

292. Medicine of choice in case of lambliosis (giardiasis) is:A. GentamycinB. PenicillinC. AspirinD. *FurasolidonE. Papaverin

293. Causes of secondary chronic pancreatitis are:A. Viral-bacterial infectionsB. *Liver pathology C. Allergic factors D. Medicinal damagesE. Traumas of stomach

294. Localization of pain at chronic pancreatitis:A. In the right subcostal areaB. In the left subcostal areaC. *In epigastriumD. In the left inguinal areaE. In the right inguinal area

295. Typical point tenderness at chronic pancreatitis is:A. MendehlB. *Mayo-RobsonC. KehrD. OrthnerE. Maslov

296. What symptom is a positive at chronic pancreatitis?A. *ShoffarB. MendehlC. OrthnerD. Musse-GeorgievskyE. Kehr

297. Feces at chronic pancreatitis are:A. SolidB. ThickC. *Porridge-like or liquidD. Putty-like E. Normal

298. Chronic pancreatitis is characterized by:A. HyperbilirubinemiaB. *HyperamylasemiaC. HypoamylasemiaD. Hypercholesterolemia

E. Hypobilirubinemia299. Steatorrhea is characteristic for:

A. Gastritis B. HepatitisC. ColitisD. *PancreatitisE. Esophagatis

300. Kreatorrhea is characteristic for: A. Gastritis B. HepatitisC. ColitisD. *PancreatitisE. Esophagatis

301. Diastasuria is characteristic for: A. HepatitisB. Cholecystitis C. *Pancreatitis D. ColitisE. Esophagatis

302. For correction of endotoxic shock in the patients with pancreatitis is used:A. AlbumenB. *RheopoliglucinC. CrioprecipitateD. HaemodesE. Heparin

303. What parasites may not be present in bile?A. GiardiaB. OpistarchosisC. StroingyloidesD. *AscaridosisE. All may be present

304. ?All of the following conditions are cause of glomemlonephritis, except:A. *AllergiesB. Bacterial infectionC. ViralD. Fungal E. Vaccination

305. Which symptom is the criterion of nephritic variant of acute glomerulonephritis?A. *А. Hematuria B. В. LeucocyturiaC. С. Bacteruria D. AnasarcaE. Proteinuria more than 3g per day

306. What investigation must be performed to confirm the kidneys dysfunction during chronic glomerulonephritis?A. General analysis of urineB. *Endogenous creatinin clearanceC. Nechiporenko testD. Estimation of daily proteinuriaE. Estimation of cholesterol

307. What dose of heparin is necessary to prescribe for treatment of acute glomerulonephritis?A. 50-100 IU/kgB. *100-300 IU/kg

C. 10-20 IU/kgD. 500-600 IU/kgE. 250-500 IU/kg

308. Which syndrome is contraindication for prescribing of heparin?A. Nephrotic syndrome of acute glomerulonephritisB. Mixed form of chronic glomerulonephritisC. Sub acute malignant glomerulonephritisD. *Aplastic anemiaE. Nephrotic syndrome of chronic glomerulonephritis

309. Which position is incorrect in relation to treatment of nephrotic syndrome:A. Corticosteroid therapy is appointedB. The dose of diuretics is depending from an effectC. *To all patients with nephrotic syndrome it is necessary to prescribe cytostaticsD. During the expressed edema and hyper coagulation it is necessary to prescribe heparinE. It is necessary to prescribe cytostatics according the needing

310. During the hormone depending nephrotic form of chronic glomerulonephritis it is necessary to prescribe with the prednisolone:A. Non-steroid antiinflamation drugsB. Preparates of aminohinoline groupC. *CytostaticsD. HeparinE. Curantil

311. Which method of research is most informing at the syndrome of malignant hypertension?A. X-ray of kidneysB. Excretory X- ray of kidneysC. *AngiographyD. Ultrasound investigation of kidneysE. Biochemical analysis of blood

312. Which position is incorrect relatively to nephritic variant of edema syndrome?A. Edema syndrome more frequent is moderately expressedB. In. the basic of mechanism of such edema is violation of the vascular penetrating C. *In the basic of mechanism of such edema is hypoproteinemiaD. In the basic of mechanism of such edema hypoproteinemia is not importantE. In. the basic of mechanism of such edema is increase of pressure

313. Excretory urography allows estimate:A. Anatomical state of urinary tract and urodinamics B. State of nephronsC. Functional state of urinary tract D. State of urinary bladderE. Functional state of urodinamics

314. Which variant of therapy is most expedient during the mixed form of chronic glomerulonephritis?A. Monotherapy of prednizoloneB. *Prednisolone and cytostaticsC. Prednisolone and heparinD. Prednisolone and heparin with curantilE. Prednisolone and heparin with curantil and cytostatics

315. Which position is incorrect in relation to treatment of nephrotic syndrome?A. Corticosteroid therapy is appointedB. The dose of diuretics depends of effectC. *Cytostatics are prescribing to all patientsD. During edema syndrome and hypercoagulation it is necessary to prescribe heparinE. Cytostatics are prescribing only according indications

316. Violation of desintoxication function of kidneys in the beginning of glomerulonephritis is

characteristic for:A. Isolated urinary syndromeB. *Nephritic syndromeC. Nephrotic syndromeD. Mixed formE. Nephrotic syndrome with arterial hypertension

317. Which biochemical test can’t demonstrate activity of inflammatory process of glomerulonephritis?A. Protein and its fractionsB. C – reactive proteinC. SeromucoidD. ESRE. *Ca and P of blood

318. The children of school age have normal content of urea in the blood:A. 21,44-32,76 mmol/lB. 3,33-8,32 mmol/lC. 15,66-17,45 mmol/lD. 1,76-2,87 mmol/lE. 2,43-3,76mmol/l

319. The children of school age have normal content of creatinine in the blood:A. *Up to 0,1 mmol/lB. 2,1-3,2 mmol/lC. 4,3-7,3 mmol/lD. 15,0-17,3 mmol/lE. 4,2- 1,1 mmol/l

320. Which formula is necessary for the calculation of day's amount of urine for children?A. *600+100(n-1), where n-amount of years B. 500х(n+5), where n-amount of years C. mass +10D. 600х(n+5), where n-amount of years E. 600х(n+10), where n-amount of years

321. Excretory urography does not allow discover:A. Position, sizes, form of kidneysB. State of nephronsC. Functional state of urinary tract D. Anomaly of kidney vesselsE. State of urinary bladder

322. What is the main feature of kidney edema?A. Appears at morning, warm, paleB. Appears in the second half of day, cold, cyanoticC. DenseD. Disposed mainly on trunkE. Disposed mainly on place or sex organs

323. Kidney angiography is the main investigation for:A. Suspicion on the defects of development or disease of vessels B. Arterial hypertensionC. NephroptosisD. NephritisE. Glomerulonephritis

324. What is the normal correlation between daily and nightly dieresis?A. 1:1B. 2:1C. 3:1

D. 4:1E. 1:2

325. Which drug do we use for pathogenetical treatment of nephrotic syndrome?A. *PrednisoloneB. PenicillinC. CurantilD. SuprastinE. Ascorutin

326. Which diet is necessary prescribes during acute glomerulonephritis?A. 1B. 2C. 3D. *7E. 5

327. What reason of glomerulonephritis is the more often?A. *StreptococcusB. StaphylococcusC. E.СоlіD. Virus of hepatitis AE. Virus of flu

328. What is the main pathogenetical mechanism of development of glomerulonephritis?A. Bacterial inflammationB. Allergy reactionC. *Immunocomplex damaging D. Violation of passage of urineE. Reflux

329. Which symptom is not characteristic for glomerulonephritis?A. Pain in backB. *Pain during the act of urinationC. HypertensionD. EdemaE. Oliguria

330. For nephrotic variant of acute glomerulonephritis is characteristic:A. HypoglycemiaB. HyperbilirubinemiaC. *HyperlipidemiaD. AzotemiaE. Hypocalcaemia

331. Which level of proteinuria is the criterion of nephrotic variant of acute glomerulonephritis?A. *3 g/l and moreB. 2-3 g/l C. 1-3 g/l D. Up to 1 g/lE. 0,5-2 g/l

332. Which symptom isn’t characteristic for isolated urine syndrome during acute glomerulonephritis: A. ProteinuriaB. ErythrocyturiaC. CylindruriaD. Absence of extra renal signsE. *Leucocyturia

333. Which investigation is demonstrates the concentration function of kidneys?A. General analysis of urineB. Analysis of urine by Nechiporenko

C. *Analysis of urine by ZimnitskiyD. Ultrasound investigation of kidneysE. Biochemical analysis of blood

334. Which sign does give information about the damaging of function of kidneys?A. HypoproteinemiaB. ProteinuriaC. *AsotemiaD. HyperlipidemiaE. Hyperglycemia

335. Recovering after acute glomerulonephritis is the period of clinical and laboratory remission during:A. 1 yearB. *3 yearsC. 5 yearsD. 2 yearsE. 6 months

336. The main criterion of hematuric form of chronic glomerulonephritis is:A. Arterial hypertensionB. ProteinuriaC. *HematuriaD. CylindruriaE. Leucocyturia

337. Severe edema syndrome is most characteristic for:A. *Nephrotic syndromeB. Nephritic syndromeC. Mixed form of chronic glomerulonephritisD. Hematuric formE. Isolated urinary syndrome

338. What is the reason of pain during glomerulonephritis:A. Inflammation syndromeB. Stagnation of urineC. *Enlargement of capsules of kidneysD. Physical exertionE. Hematuria

339. Which medicine do we use during pathogenetical treatment of nephritic variant of glomerulonephritis?A. Reserpin B. LasixC. TavegilD. DelagilE. *Prednisolone

340. Which dose of indometacin is use for treatment of nephritic variant of glomerulonephritis? A. *2-3 mg/kg/dayB. 1-2 mg/kg/dayC. 3 – 5 mg/kg/dayD. 0,5 – 1 mg/kg/dayE. 0,8 – 1 mg/kg/day

341. It is nessesary to prescribe imunosupressors during acute glomerulonephritis in the case of:A. Suddenly beginning of diseaseB. Severe proteinuriaC. *The absence of effect from the treatment of prednisolone during 3-4 weeks in the case of

nephrotic variantD. The absence of effect from the treatment of prednisolone during 3-4 weeks in the case of

nephritic variantE. The damaging of functions of kidneys at the beginning of disease

342. Severe edema syndrome is more characteristic for:A. MyocarditisB. *GlomerulonephritisC. PyelonephritisD. Kidney stones diseaseE. Cystitis

343. The Zimnitski test of urine gives information about:A. *Concentration functionB. BacteruriaC. LeucocyturiaD. ErithrocyturiaE. Glucosuria

344. Which from these products are eliminated in a diet 7?A. RiceB. Porridge C. Vegetable pureeD. EggE. *Meat

345. What from diseases more frequent can result in development of chronic kidney insufficiency?A. Kidney stones diseaseB. Metabolic nephropathiesC. RefluxD. *Glomerulonephritis chronicE. Oxalaturia

346. The middle age dose of lasix is:A. *1-2 mg/kg/dayB. 4-5 mg/kg/dayC. 10 mg/kg/dayD. 0,5-1 mg/kg/dayE. 0,5mg/kg/day

347. What test is used to confirm filtration function of kidneys?A. UrinanalysisB. NechiporencoC. Zimnitski testD. *Creatinine clerance test E. Rebergs test

348. The most frequent complication of hereditary nephropathy is: A. ARFB. Pyelonephrosis C. *CRF D. Nephrolithiasis E. Amiloidosis of kidneys

349. The development of hereditary nephropathy is often associated with: A. Sex chromosome abnormality B. *Gene mutations C. Chronic inflammatory diseases of the mother D. Measles of pregnant women E. Change of the number of chromosomes

350. What is Alport syndrome?A. Cystic kidneys B. *Hereditary nephritis with deafness

C. Hereditary nephritis without deafness D. Renal diabetes E. Embryonic kidney tumor

351. What kidney disease is disease with chromosomal aberrations?A. Hereditary nephritisB. Primary tubulopathyC. Secondary tutulopathyD. Renal amyloidosisE. *Cystic kidneys

352. Disease caused by gene mutations is:A. *Hereditary nephritisB. Primary tubulopathyC. Secondary tubulopathyD. Renal amyloidosisE. Cystic kidneys

353. Secondary tubulopathy is observed at:A. Phosphate-diabetesB. Tubulyar renal acidosisC. Disease de Toni-Debrae-FanconiD. Hereditary nephritisE. *Galaktozemia

354. Hereditary nephritis is inherited by:A. *Dominant type, connected with the X chromosomeB. Recessive type, connected with the X chromosomeC. Dominant type, connected with Y chromosomeD. Recessive type, connected with Y chromosomeE. Autosomal recessive type

355. The basis of the pathogenesis of hereditary nephritis is:A. Inflammatory process in glomerularsB. *Dysembriogenesis of connective tissueC. Violation of intravascular coagulation in glomerular capillariesD. Failure of proximal tubules in reabsorbtion of bicarbonatesE. Violation of the process in tubular transport

356. Most distinctive feature of the initial period of hereditary nephritis is:A. LeukocyturiaB. BacteriuriaC. *MicrohematuriaD. OxaluriaE. Uraturia

357. What symptom is observed at initial period of hereditary nephritis?A. *Vascular hypotensionB. HypertensionC. PolyuriaD. OliguriaE. Edema

358. Hereditary nephritis with deafness is called:A. Disease-de Toni- Debrae- FanconiB. Berger's diseaseC. *Alport syndromeD. Harnupa diseaseE. Leu syndrome

359. Which extrarenal signs do point the possibility of hereditary nephritis at presence of urinary syndrome?

A. *Stigmes of dyzembriogenesisB. Edema of legsC. Skin edemaD. Increased APE. Haemorrhagic rash

360. The diagnostic criterion of hereditary nephritis does not include:A. Urinary SyndromeB. DeafnessC. DysmorphiesD. *Prior tonsillatiesE. Similar illness in family

361. For urinary syndrome at hereditary nephritis is not typical:A. *BacteriuriaB. MicrohematuriaC. PoteinuriaD. MacrohematuriaE. Cylindruria

362. Hereditary nephritis typically ends with:A. RecoveringB. Formation of pyelonephrosisC. *CRFD. Development of urolithiasisE. Acute renal failure

363. Rickets like changes among hereditary nephropathy are often found at:A. Cystic kidneysB. *Phosphate-diabetesC. Renal without hereditary deafnessD. Renal with hereditary deafnessE. Renal amyloidosis

364. What is not characteristic for hereditary nephritis?A. *EdemaB. Arterial hypotensionC. HematuriaD. ProteinuriaE. Dyzmorphies

365. The main criterion of differential diagnose of hereditary nephritis is:A. *Genetic anamnesisB. HematuriaC. ProteinuriaD. Previous renal colic attacksE. Hypertension

366. At hereditary nephritis connection of bacterial infection in children leads to:A. AzotemiaB. CylindruriaC. *PiuriaD. HypoproteinemiaE. Crystaluria

367. The main role in etiology of pielonephritis plays:A. *BacteriaB. VirusesC. MicoplasmaD. ParasitesE. Fungi

368. What from such drugs is not used at treatment of acute pyelonephritis?A. AntibioticsB. UrosepticsC. *HormonesD. Antiinflamation drugsE. Diet

369. What from such drugs is ineffective at treatment of pyelonephritis?A. Ampicillin B. AmicinC. CefatoxinD. *LevomicetinE. Loracin

370. What from these signs do not present at pyelonephritis?A. LeucocytosisB. LeucocyturiaC. Little proteinuria D. *HematuriaE. Bacteriuria

371. What sign gives information about the damaging of kidneys function?A. HypoproteinemiaB. ProteinuriaC. *AzotemiaD. HyperlipidemiaE. Hyperglycemia

372. In oxalaturia it is necessary to eliminate from ration: A. PotatoesB. *SorrelC. CabbageD. PeaE. Mushrooms

373. What diet it is necessary prescribe during acute pyelonephritis?A. 1B. 2C. 3D. 7E. *5

374. The main sign of pielonephritis is:A. LeukocyturiaB. HematuriaC. ErythrocyturiaD. ProteinuriaE. Cylindruria

375. Excretory urographia allows to estimate:A. *Anatomical state of urinary tract and urodinamicsB. State of nephronsC. Functional state of urinary tract D. State of urinary bladderE. Functional state of urodinamics

376. At what pathology ultrasound research is most informing?A. *Kidney tumorB. Hydronephrotic transformationC. GlomerulonephritisD. Pielonephritis

E. Cystitis377. What is predispositive factor of the development bladder-urether-pelvis reflux in children?

A. *Anatomic immaturity of urether orificeB. Intravesicular obstructionC. Neurogenic dysfunction of urine bladderD. Recanalization in violation of urethers in embriogenesisE. All transferred

378. What is the etiologic factor of primary tubulopathy?A. MicrobsB. *Violation of membrane substances inrenal tubulesC. Reflux D. Anatomic anomaly of kidneyE. Insufficiency of podotcytes

379. Which of the following pathogenic mechanisms occur at chronic obstructive pyelonephritis in children?A. Immune damage of the glomerular capillaries of the kidneysB. *Malformations of the urinary system, impeding outflow of urineC. Fatty degeneration of the epithelium of tubulesD. RefluxE. All transferred

380. Which of the following symptoms are typical for acute pyelonephritis in children?A. Proteinuria more than 3g / dayB. Macrohematuria and bacteriuriaC. Microhematuria and bacteriuriaD. Microhematuria and proteinuria E. *Pyuria and bacteriuria

381. What system of organism is damaged first of all at renal failure?A. CardiovascularB. CNSC. DigestiveD. RespiratoryE. *All equally

382. What symptom is uncommon at cystitis in children?A. *Blood in urineB. FeverC. Vomiting or diarrheaD. Crying, going off feeds and generally unwellE. Appear to be in pain

383. What is the main feature of urine infection in children?A. LeucocyturiaB. *BacteriuriaC. ErythrocyturiaD. CylindruriaE. All transferred

384. The main task of therapy of acute cystitis in children should be directed to:A. The elimination of painB. Normalization of urination disordersC. Elimination of microbial-inflammatory process in the bladderD. Liquidation of spasmsE. *All transferred

385. Urethritis almost always occurs in children:A. First year oldB. Preschoolers

C. *Before pubertyD. 14-17 years oldE. Of all age groups

386. What is not characteristic for urethritis in children?A. Discomfort, stinging, or burning when urinating B. Feeling an urgent and frequent need to urinate C. *FeverD. Itching in the genital area E. Pain in the genital area

387. What are the main causes of urethritis in children?A. Irritation by chemicals in bubble bathB. Shampoo left on the genital areaC. Soap left on the genital areaD. Urinary tract infectionsE. *All transferred

388. Pyelonephritis is distinguishing from low urinary infection by:A. Fever more than 38,5°СB. Leukocytosis and increasing of ESRC. Increasing of concentration function of kidneysD. Proteinuria and aminoaciduriaE. *All transferred

389. ?The main symptom of chronic bronchitis is:A. HyperthermiaB. WheezingC. *Prolonged coughD. DyspneaE. Intoxication

390. What is the most important reason of all organs and systems dysfunction in chronic lung disease in children? A. Surfactant deficiencyB. *Hypoxia C. IntoxicationD. Hemodynamic disordersE. Immunity impairment

391. What symptom is the sign of the continuous hypoxemia? A. Cyanosis of nasolabial triangleB. Acrocyanosis C. Wet cough in the morning D. Total cyanosisE. *Deformation of the terminal phalanges as "drumsticks" and "watch glasses"

392. What are the auscultation signs of chronic lung disease in children? A. A variety of wheezes B. A variety of dry and moist rales C. *Permanent local fine rales D. Amphoric breathingE. Crepitation

393. What X-ray changes are characteristic for chronic lung disease in children? A. Infiltration of lung tissue in basal areas B. Lung tissue infiltration on the tops of the lungs C. Availability of cavitiesD. *Increased and deformation of the lung pattern E. Scanty lung pattern

394. What is the leading component of complex treatment of chronic lung disease in children?

A. Antibacterial therapy B. *Restoration of bronchial drainage function C. General stimulation therapy D. Therapeutic feedingE. Immune therapy

395. Final diagnose of pulmonary hypoplasia is based on:A. *BronchographyB. BronchoscopyC. X-ray examinationD. US observation of lungsE. CT-scan

396. What is NOT typical for lung agenesia?A. CyanosisB. The heart is shifted towards the lesionC. On the lesion side thorax is flattened, and the healthy half is convexD. *On the lesion side thorax is convex, and the healthy half is flattenedE. Nail "drumsticks" on phalanges

397. The incidence of lung hypoplasia is higher in:A. Big birth weight infantsB. Low birth weight infantsC. Newborns after pathological pregnancyD. Newborns after pathological deliveryE. *Preterm infants

398. Pulmonary hypoplasia occurs as a result of:A. Oligohydramnios B. Potter’s syndromeC. Abnormalities of the thoracic cageD. Diaphragmatic herniaE. *All mentioned above

399. What does predominate in the clinical presentation of cystic adenomatous malformation?A. HypoxiaB. Respiratory acidosisC. *Disturbance of the bronchial drainage functionD. Arrhythmic respirationE. Respiratory failure

400. What examinations are the most important in cystic adenomatous malformationA. Chest X-ray and CT-scanB. Bronchoscopy and chest X-ray C. Chest X-ray and bronchographyD. *CT-scan and bronchographyE. Bronchoscopy and bronchography

401. What complication may occur at cystic adenomatous malformation?A. InfectionB. HemorrhageC. Acute respiratory failureD. Neoplastic transformationE. *All mentioned above

402. Choose the wrong statement about cystic adenomatous malformation?A. Cysts may be filled with air or fluidB. *Cysts can not be asymptomaticC. Cysts arise from an abnormal budding of the ventral foregutD. Such complications as infection, hemorrhage, and malignancy can occurE. Dysphagia and epigastric discomfort can occur

403. What is not characteristic for cystic adenomatous malformation?A. Dullness of percussion soundsB. Respiratory failureC. Auscultation with the weakening of breathD. Wheezing of various sizesE. *Dry rales

404. What is pulmonary sequestration?A. Increasing of the number of lobesB. Increasing of the mass of lobesC. Histological changes of lung tissueD. *Benign mass of non-functioning lung tissueE. Intralobar malformation

405. What is the predominant localization of pulmonary sequestration?A. *Left lower lobeB. Left upper lobeC. Right lower lobeD. Right upper lobeE. Right middle lobe

406. What is NOT typical for pulmonary sequestration?A. Chronic coughB. Recurrent pneumoniasC. Poor exercise resistanceD. *Pain in the chestE. Congestive cardiac failure

407. What is the reason of chronic cough in children with pulmonary sequestration?A. Compression of the lung tissueB. Decreased mucociliary clearanC. *Recurrent respiratory tract infectionsD. Disturbance of the drainage functionE. All mentioned above

408. Choose the gastrointestinal sign of pulmonary sequestration?A. VomitingB. Failure to thriveC. Abdominal painD. Poor appetiteE. *All mentioned above

409. What is the main distinguishing feature of sequestration of the lung?A. Progressive respiratory failureB. Chronic coughC. Recurrent respiratory tract infectionsD. *Additional large vesselE. Small bubbling (moist) rales

410. The main diagnostic value at sequestration of the lung has:A. *AngiographyB. BronchoscopyC. X-ray D. BronchographyE. CT

411. Choose the wrong statement about surgical treatment at sequestration of the lung:A. Can be recommended for asymptomatic patientsB. Is obligatory after diagnosticsC. Is recommended in compression of normal lung massD. It is preventive measure

E. *Should be performed only in patients with recurrent infections412. What is the main distinguishing clinical feature of Mounier-Kuhn syndrome

(tracheobronchomegaly)?A. Attacks of spasmodic coughB. Dry unproductive coughC. Wet unproductive coughD. *Brassy paroxismal cough E. Dry cough

413. What is the main distinguishing clinical feature of tracheobronchomegaly?A. Spasmodic coughB. Dry unproductive coughC. Wet unproductive coughD. *Brassy paroxismal cough E. Dry productive cough

414. What auscultation data are characteristic for tracheobronchomegaly?A. Fine moist ralesB. CrepitationC. Dry ralesD. *Variety of wheezingE. All mentioned above

415. Choose the best method of tracheobronchomegaly diagnostics?A. Chest x-rayB. *BronchoscopyC. Chest ultrasoundD. Chest CT-scanE. Angiography

416. Choose the synonym of the Williams– Campbell syndrome:A. TracheobronchomegalyB. Sclerotic changes in the peribronchial tissuesC. Deformation of trachea and bronchiD. Expansion of the lumen of the trachea and main bronchiE. *Congenital generalized bronchiectasis

417. What is the reason of Williams– Campbell syndrome?A. *Bronchial cartilage defectB. Bronchial muscular defectC. Mucociliary clearance insufficiencyD. Atrophy of the longitudinal elastic fibresE. Disturbance of the bronchial drainage function

418. What is the typical localization of pathological process at Williams– Campbell syndrome?A. Upper lobes of lungsB. *Low lobes of lungsC. Middle lobe of right lungD. MediastinumE. Trachea

419. Choose RIGHT statement about Williams– Campbell syndrome.A. The clinical signs are typical for bronchial obstructionB. Bronchopulmonary infection is frequent symptomC. The pathological process becomes chronic alwaysD. Is typical chest deformationE. *All is correct

420. Choose WRONG statement about Williams– Campbell syndrome.A. The clinical signs are typical for bronchial obstructionB. Bronchopulmonary infection is frequent symptom

C. The pathological process becomes chronic alwaysD. Is typical chest deformationE. *All is correct

421. Choose the best method of Williams– Campbell syndrome diagnostics?A. Chest x-rayB. Chest ultrasoundC. *BronchographyD. CT-scanE. Angiography

422. Choose the WRONG statement about congenital lobar emphysema:A. *There is abundant anomalyB. Respiratory insufficiency can develop immediately after birthC. Surgical treatment is necessaryD. Can be diagnosed with the help of chest x-rayE. Is typical absence or hypoplasia of bronchi cartilages at the part of lung

423. What percussion data are characteristic for congenital lobar emphysema?A. Local dullnessB. Tympanic soundC. Diffuse dullnessD. *Bandbox soundE. Tympanic sound

424. Cyanosis, shortness of breath, coughing, choking during breastfeeding is the most typical for:A. *Tracheoesophageal and bronchoesophageal fistulasB. Atresia of the esophagusC. TracheobronchomegaliaD. Atonia of esophagusE. Cogenital achalasia of esophagus

425. What type of inheritance is characteristic for Kartagener syndrome?A. Autosomal dominantB. *Autosomal recessiveC. X-linked dominantD. X-linked recessiveE. Y-linked

426. Hamman-Rich syndrome is characterized by:A. *Rapidly progressive diffuse pulmonary fibrosisB. Bronchial agenesiaC. TracheobronchomegaliaD. Bronchial aplasiaE. All is correct

427. What main treatment is used at Hamman-Rich syndrome?A. *CorticosteroidsB. AntibioticsC. BronchodilatatorsD. ExpectorantsE. Surgical

428. What changes in biochemical blood analysis are typical for the idiopathic pulmonary hemosiderosis:A. Hypoproteinemia and hypobilirubinemiaB. Increased level of direct bilirubin and transaminasesC. Decreased level of serum iron and hypoproteinemiaD. Increased level of serum iron and hypobilirubinemiaE. *Decreased level of serum iron and increased level of indirect bilirubin

429. Treatment of idiopathic pulmonary hemosiderosis includes:

A. Oxygen therapyB. GlucocorticoidsC. CytostaticsD. DeferoxamineE. *All mentioned above

430. What type of inheritance is characteristic for primary pulmonary hypertension?A. *Autosomal dominant B. Autosomal recessive C. Y-linkedD. X-linked recessiveE. X-linked dominant

431. What type of inheritance is characteristic for cystic fibrosis?A. Autosomal dominant B. *Autosomal recessive C. Y-linkedD. X-linked recessiveE. X-linked dominant

432. What form of cystic fibrosis is typical only for newborns?A. Pulmonary B. Enteric C. Mixed D. *Meconium ileusE. Edematous-anemic

433. What medicines are the basic in the cystic fibrosis treatment?A. AntibioticsB. MycoliticsC. *EnzymesD. CytostaticsE. Glucocorticoids

434. A typical percussion sign of acute bronchitis is: A. shortening of pulmonary sound in the upper parts of the lungs B. shortening of pulmonary sound in the lower parts of the lungs C. shortening of the lung sound all over the lungsD. local tympanic sound E. * clear lung sound

435. ?Weight deficit 10 - 20% is typical for:A. 2nd degree malnutritionB. 3rd degree of malnutrition C. eutrophia D. everything is wrong E. * 1st degree malnutrition

436. Weight deficit 21 - 30% is typical for: A. 1st degree malnutrition B. 3rd degree of malnutrition C. eutrophia D. everything is wrong E. * 2nd degree malnutrition

437. Weight deficiency more than 30% is typical for:A. 1st degree malnutrition B. 2nd degree malnutrition C. eutrophia D. everything is wrong E. * 3rd degree of malnutrition

438. Paratrofia is manifested by:A. decreased body weight proportionally to the lengthB. increased body weight in stunted growth C. decreased body weight with an accelerated growth D. everything is correct E. * increased body weight proportionally to the length

439. Paratrofia is manifested by:A. reduction of subcutaneous fat B. reduced skin elasticity C. increased soft tissues turgor D. everything is correct E. * increased subcutaneous fat

440. In the biochemical analysis of blood ща сhild with malnutrition will be:A. hyperglycemia B. hypercalcemia C. hyperalbuminemia D. everything is correct E. * hypoproteinemia

441. In coprogram of child with malnutrition will be detected signs of:A. hyperenzymopathy B. inflammation C. worms invasion D. erythrocytes E. * hypoenzymopathy

442. The main cause of malnutrition A. enzymopathy B. pneumonia C. rickets D. dysbacteriosis E. * inadequate dietary intake

443. Common form of malnutrition is: A. spasmophylia B. thrombocytopenia C. hypoplastic anemia D. everything is correct E. * iron deficiency

444. Body temperature in children with severe malnutrition is: A. increased B. normal C. subfebrile D. hectic E. * decreased

445. For severe malnutrition is typical: A. weight-length coefficient is 60 B. trophy index is 0 C. irritability of the child D. weight loss is 10-20% E. * "Old man" face

446. Therapeutic feeding for children with malnutrition:A. diet according to the age B. diet, taking into account the concomitant disease C. diet with adapted formulas D. diet with maladapted formulas

E. * rejuvenation food 447. How many days does the period of tolerance to food last in mild malnutrition?

A. 1 - 5 days B. 7 days C. 10 days D. everything is correct E. * 1 - 3 days

448. Why is not good to give full food volume for the child with malnutrition? A. Because of hyperenzymopathy B. Because of dyspancreatysm C. Because of poor appetite D. everything is wrong E. * Because of hypoenzymopathy

449. Name principles of nutrition in malnutrition: A. diet, taking into account the concomitant disease and cause of malnutrition B. diet, taking into account children's age and cause of malnutrition C. diet using adopted formulas and age of the child D. maximum feeding E. * tolerance to food establishment and food rejuvenation

450. What does the principle of two-phase feeding mean? A. establishment of tolerance to food and the maximum feeding B. establishment of tolerance to food and minimum feeding C. establishment of tolerance to food and breastfeeding D. maximum feeding E. * establishment of tolerance to food and optimal feeding

451. Using of pancreatic enzymes is appropriate:A. 1 hour before the mealB. after the mealC. 1 hour after the meal D. before eating E. * during the meal

452. Choose the Drug for replacement therapy in case of malnutrition: A. pentoxyl B. apylac C. thiotriazoline D. glucose E. * pancreatin

453. Name peculiarities of the diet in paratrophia: A. protein restriction B. restriction of vitamins C. increase in fat D. everything is wrong E. * reduce of carbohydrate

454. The aim of probiotics therapy: A. for correcting the oral cavity microflora B. for correcting the bronchial microflora C. to fight infection D. everything is correct E. * for correcting the intestinal microflora

455. Choose bacterial drugs (probiotics): A. pancreatin B. clarithromycin C. penthoxyl

D. everything is correct E. * linex

456. The main clinical syndromes of diarrhea are:A. dyspepsia, dehydration, bleeding B. dyspepsia, endotoxemia, portal hypertension C. constipation, endotoxemia, dehydration D. everything is correct E. * dyspepsia, dehydration, endotoxemia

457. Dyspeptic syndrome includes: A. constipation, flatulence, fatigueB. fever, vomiting, jaundice C. vomiting, constipation, jaundice D. diarrhea, fever, jaundice E. * diarrhea, flatulence, vomiting

458. Which symptoms belongs to the clinical syndromes of enzymopathy?A. hepatosplenomegaly B. swelling of the brain C. renal failure D. everything is correct E. * dehydration

459. Choose drug, which is used for rehydration: A. lypofundin B. jelatynol C. alvezin D. rheopolyglucine E. * 5% glucose solution

460. Choose drug, which is used for detoxification A. alvezin B. acesol C. lypofundin D. potassium chloride E. * rheosorbilact

461. It is advisable to give When diarrhea : A. glucocorticoids B. membrane stabilization medicine C. vitamins D. antibiotics E. * enterosorbents

462. Ciprofloxacin belongs to: A. macrolides B. cephalosporins C. aminoglycosides D. penicillins E. * fluoroquinolones

463. The daily dose of ciprofloxacin to the child is: A. 5 - 10 mg / kg B. 15 - 20 mg / kg C. 20 - 25 mg / kg D. 30 - 40 mg / kg E. * 10 - 15 mg / kg

464. The daily dose of cephalosporin to the child is:A. 100 - 150 mg / kg B. 150 - 200 mg / kg

C. 200 - 250 mg / kg D. 10 - 30 mg / kg E. * 50 - 100 mg / kg

465. Dehydration syndrome includes: A. polyuria B. edema C. bulging large fontanel D. everything is correct E. * loss of body weight

466. Hypertonic dehydration is characterized by: A. normal body temperature B. hypothermia C. sleepiness D. everything is correct E. * hyperthermia

467. Hypertonic dehydration is characterized by:A. sodium levels are normal B. chloropenia C. reduced hematocrit D. decreased urine density E. * increased urine density

468. Choose indications for the antibiotic therapy in case of diarrhea: A. dyspnea B. convulsions C. hyperthermia D. vomiting E. * pathological admixtures in feces

469. How many fluid does healthy infant need per day? A. 60 - 100 ml/kgB. 110 - 120 ml/kg C. 100 - 110 ml/kg D. 120-130 ml/kgE. * 130 - 150 ml/kg

470. How many fluid does infant with severe dehydration need per day? A. 150 - 160 ml B. 170 - 180 ml C. 100 - 120 ml D. 50 - 100 ml E. * 200 - 220 ml

471. What ratio of salt and water solutions for infant with hypotonic dehydration should be? A. 2: 1 B. 3: 1 C. 4: 1 D. 1: 2 E. * 1: 1

472. Name the most common cause of bronchitis A. fungi B. bacteria C. helminthD. enzymopathy E. * viruses

473. RS-infection often causes bronchiolitis in: A. preschoolers

B. toddlers C. school age childrenD. teens E. * infants

474. Helminth infection is often the cause of bronchitis in: A. infants B. toddlers C. adolescents D. school age childrenE. * preschoolers

475. The main symptoms of acute bronchitis is: A. sore throat B. running nose C. dyspnea D. fever E. * cough

476. The typical auscultatory sing of acute bronchitis is: A. weakened vesicular breathing B. puerile breathing C. local whezing D. local rales E. * harsh breathing

477. In what age group of children bronchitis is more often? A. infants B. newborns C. toddlersD. teens E. * preschoolers

478. What degree of respiratory failure severity is the most often in acute bronchitis? A. 1st degreeB. 2nd degreeC. 3rd degreeD. everything is correct E. * 0 degree

479. Radiological signs of acute bronchitis are: A. symmetrical attenuation of lung pattern B. symmetrical amplification pattern of lung with small focal hilar infiltration C. symmetrical amplification pattern of lung in hilar and lowermedial zones D. symmetrical attenuation of bases of lungsE. * symmetrical amplification of lung pattern

480. Typical changes in Common blood test of patients with acute bronchitis: A. significant leucopenia B. moderate leucocytosis C. significant leucocytosis, accelerated erythrocyte sedimentation rate D. anemiaE. * leucopenia, lymphocytosis

481. Etiotropic therapy of viral infection in the acute bronchitis is more effective when it was given: A. no effective B. for 5-7 days C. throughout the disease D. after 7th day of disease E. * in the first 2 days of illness

482. The effectiveness of expectorants in acute bronchitis depends on:

A. appointment of antiviral drugs B. appointment of antihistamines C. appointment of vitamins D. everything is correct E. * sufficient drinking

483. Typically, recurrent bronchitis exacerbation is absent in: A. spring B. winter C. autumn D. everything is correct E. * summer

484. What is a dominating symptom In clinical exacerbation of recurrent bronchitis? A. intoxication B. dyspnea C. running nose D. everything is correct E. * cough

485. Cough with recurrent bronchitis exacerbation is more severe:A. at night B. by the day C. in the eveningD. everything is correct E. * in the morning

486. Percussion in patients with recurrent bronchitis exacerbation reveals: A. clear lung sound B. shortening of sound in the lower partsC. clear sound with a short bandbox between the scapulas D. everything is correct E. * bandbox sound

487. In acute obstructive bronchitis, auscultation reveals: A. weakened breathing B. only prolonged expiration C. small bubbling rales diffusely D. small bubbling rales locallyE. * dry rales, medium bubbling rales on inspiration, prolonged expiration

488. Bronchoscopy in recurrent bronchitis exacerbation reveals: A. diffuse changes in the form of mucosal atrophy B. diffuse changes in the form of mucosal hyperemia C. local changes of bronchial mucosa D. everything is correct E. * diffuse changes in the form of mucosal hyperemia, thickening of the bronchial walls and

mucous secretion 489. Bronchoscopy in remission of recurrent bronchitis reveals:

A. hyperemia of the bronchial mucosaB. atrophy of the bronchial mucosa C. normal bronchial mucosaD. everything is correct E. * granulation on bronchial mucosa

490. The main treatment of recurrent bronchitis is: A. toxicosis liquidationB. antiviral treatmentC. dehydration liquidationD. everything is correct

E. * decrease of bronchial obstruction 491. In the treatment of acute recurrent bronchitis postural drainage is better to use:

A. after breakfast B. in the afternoon C. at bedtime D. everything is wrong E. * after awakening

492. Duration of antibiotic therapy in patients with recurrent bronchitis exacerbation is: A. 14 days B. 21 days C. 28 days D. everything is correct E. * 7 days

493. Recurrent bronchitis, which lasts more than 5 years, is a harbinger of: A. asthma B. scoliosis C. polyhypovitaminosis D. everything is wrong E. * chronic bronchitis

494. The main symptom of chronic bronchitis is: A. running nose B. fever C. dyspnea D. everything is correct E. * persistent cough

495. Name the criteria for chronic bronchitis: A. stable localized wheezing in the lungs B. wet cough C. recurrent exacerbations D. everything is correct E. * diffuse non-permanent rales in the lungs

496. The most typical symptom of bronchiectasis is: A. inspiratory dyspnea B. expiratory dyspnea C. dry cough nadsadisty D. everything is correct E. * cough

497. The final diagnosis of bronchiectasis needs: A. spirography B. bronchoscopy C. X-ray of the chest D. everything is wrong E. * bronchography

498. In chronic bronchitis are affected: A. bronchi B. lung parenchyma C. alveoli D. everything is wrong E. * all the structures of the bronchopulmonary system

499. The thorax in children with chronic bronchitis is: A. cylinder B. conical C. unmodified

D. everything is correct E. * asymmetric

500. Which of the symptoms indicate prolonged hypoxemia? A. productive cough, more in the morning B. hyperhidrosis C. perynasal cyanosis D. everything is wrong E. * clubbing fingers

501. Which of the auscultatory signs indicate chronic respiratory pathology? A. variety of wheezing and moist rales B. local small bubbling rales C. "amphora" breath D. everything is correct E. * constant variety of rales

502. Secondary chronic pneumonia develops: A. in the presence of bronchial "foreign body" B. 1 month after acute pneumonia C. on the basis of reduced immunity D. everything is correct E. * on the background of systemic and inherited pulmonary diseases

503. What is the lead point for the integrated treatment of chronic bronchopulmonary pathology in children?A. antibacterial therapy B. general-stimulation therapy C. nutritional care D. everything is correct E. * renewal of the bronchial drainage

504. What is the dose of penicillins to children with chronic bronchopulmonary pathology? A. 50 - 100 thousand IU / kg / day B. 100 - 200 thousand IU / kg / day C. 300 - 500 thousand IU / kg / day D. 10 - 25 thousand IU / kg / day E. * 100 - 150 thousand IU / kg / day

505. What is the most effective route of antibacterial drugs administration in patients with chronic respiratory pathology? A. intravenous B. endobronchial C. intramuscular D. everything is wrong E. * intraorganic electrophoresis

506. What is the duration of antibiotic therapy to children with chronic bronchopulmonary pathology? A. 2 months B. 7 - 14 days C. 1 month D. everything is correct E. * 14 - 21 days

507. What dose of aminophylline (euphylline) is used for intraorganic electrophoresis? A. 5 - 7 mg / kg B. 2 - 3 mg / kg C. 9 - 11 mg / kg D. 15 - 20 mg / kg E. * 3 - 5 mg / kg

508. What is the requirement to inhalatory antibiotics, which are used for children with chronic

bronchopulmonary pathology? A. minimal sensitization of children B. wide range of activityC. selective effect on Gram-positive flora D. everything is wrong E. * good solution in water

509. When (after exacerbation of chronic broncho-pulmonary pathology) children may be sent to the resort treatment? A. In 1 month B. Directly into the second stage C. In 6 months D. In 3 weeks E. * In 3 months

510. Which investigation is mandatory in patients with chronic respiratory pathology? A. X-ray of the chest B. spirometry C. fluorography D. everything is wrong E. * bronchography

511. What radiological changes are characteristic for chronic bronchopulmonary pathology in children? A. infiltration of lung tissue in the basal zones B. infiltration of lung tissue on the tips of the lungs C. presence of "bullas" D. everything is correct E. * amplification and distortion of lung pattern

512. The main cause of croup is: A. bacterial infections B. fungal infectionsC. parasitic lesions D. everything is correct E. * viral infections

513. Among the viruses that cause croup, the first place has: A. Adenovirus B. Influenza virus C. RS- virus D. Everything is wrong E. * Parainfluenza virus

514. Most major croup is developed: A. in the afternoon B. in the eveningC. in the morning D. everything is correctE. * at night

515. Acute stenotic laryngotracheobronchitis of the 1st degree - is: A. subcompensated croupB. de compensated croup C. asphyxia D. everything is correct E. * compensated croup

516. Name the main route of infection penetration in pneumonia: A. hematogenous B. lymphogenous

C. mixed D. everything is correct E. * bronchiogenic

517. The etiology of pneumonia is dominated by: A. klebsiella B. pathogenic fungi C. staphylococci D. viruses E. * pneumococci

518. What type of pneumonia in infants develops more often? A. croupous B. interstitial C. segmentary D. lobar E. * focal

519. Typical physical data for pneumonia are: A. weakened breathingB. diffuse small moist rales C. diffuse dry wheezing D. everything is correct E. * local small moist rales

520. Typical radiological sign for pneumonia is: A. increased lung pattern B. emphysematous lung distension C. expansion of the roots of the lungs D. everything is correct E. * the presence of infiltrative shadows

521. Duration of acute pneumonia is: A. to 6 - weeks B. to 10 - weeks C. to 12 - weeks D. up to 3 months E. * to 8 - weeks

522. Value of pulse and respiration 2 - 15: 1 is characteristic to: A. Respiratory insufficiency 1 degree B. Respiratory insufficiency 2 degree C. Respiratory insufficiency 0 degree D. everything is wrong E. * Respiratory insufficiency 3 degree

523. Blood oxygen saturation by 90% is typical for: A. Respiratory insufficiency 0 degree B. Respiratory insufficiency 2 degree C. Respiratory insufficiency 3 degree D. everything is wrong E. * Respiratory insufficiency 1 degree

524. Not stable perioral cyanosis is characteristic for: A. Respiratory insufficiency 0 degree B. Respiratory insufficiency 2 degree C. Respiratory insufficiency 3 degree D. everything is wrong E. * Respiratory insufficiency 1 degree

525. What is the most common cause of necrotizing (destructive) pneumonia: A. pneumococcus

B. klebsiella C. proteus D. everything is correct E. * staphylococcus

526. What type of oxygen therapy is the best for the child with Respiratory insufficiency 3 degree? A. the flow of oxygen in an oxygen tent B. flow of oxygen through the intranasal catheter C. the flow of oxygen through the oxygen bag D. everything is wrong E. * the flow of oxygen through the endotracheal tube

527. What type of oxygen therapy is the best for the child with Respiratory insufficiency 2 degree? A. the flow of oxygen through the oxygen bag B. flow of oxygen through the intranasal catheter C. the flow of oxygen through the endotracheal tube D. everything is wrong E. * the flow of oxygen in an oxygen tent

528. What is the typical X-ray sign in the necrotizing (destructive) pneumonia when abscess formation? A. the appearance of a round air formations on the base of the lung infiltration B. parietal and sinuses infiltration near the pulmonary infiltrationC. homogeneous total infiltration D. displacement of the mediastinal organs to the opposite E. * the appearance of a round high degree infiltration with the level of liquid on the base of

the lung infiltration 529. What is the duration of antibacterial therapy in children with mild pneumonia?

A. 3 - 5 days B. 5 - 7 days C. 10 - 14 days D. 1 - 3 days E. * 7 - 10 days

530. What is the duration of antibacterial therapy in children with moderate pneumonia? A. 7 - 10 days B. 5 - 7 days C. 14 - 20 days D. 20 - 25 days E. * 10 - 14 days

531. What is the duration of antibacterial therapy in children with severe pneumonia? A. 10 - 14 days B. 7 - 10 days C. 21 - 28 days D. everything is correct E. * 14 - 21 days

532. What is the dose of semisynthetic penicillins in children with mild pneumonia? A. 30 - 50 mg / kg / day B. 80 - 100 mg / kg per day C. 100 - 150 mg / kg / day D. 150 - 200 mg / kg / day E. * 50 - 80 mg / kg / day

533. What is the dose of semisynthetic penicillins in children with moderate pneumonia? A. 50 - 80 mg / kg / day B. 30 - 50 mg / kg / day C. 100 - 150 mg / kg / day D. 150 - 200 mg / kg / day

E. * 80 - 100 mg / kg / day 534. What is the dose of semisynthetic penicillins in children with severe pneumonia?

A. 50 - 80 mg / kg / day B. 80 - 100 mg / kg / day C. 30 - 50 mg / kg / day D. 150 - 200 mg / kg / day E. * 100 - 150 mg / kg / day

535. The hospital pneumonia is that pneumonia which developed: A. within 12 hours of hospitalization B. within 24 hours of hospitalizationC. within 6 hours of hospitalization D. everything is correct E. * within 48 hours of hospitalization

536. Most community-acquired pneumonia in children from 6 months to 6 years is caused by: A. Mycoplasma B. Chlamydia C. Staphylococcus D. E. coli E. * Pneumococcus

537. Ventilator - associated pneumonia up to four days stay on the ventilator, usually is caused by: A. Enterobacteria B. Streptococci C. Klebsiella D. everything is wrong E. * Pneumococci

538. Pneumococcal bacteria is completely resistant to: A. penicillins B. macrolides C. cephalosporins D. fluoroquinolones E. * aminoglycosides

539. The drug of choice for treatment of typical community-acquired pneumonia is: A. carbapenems B. fluoroquinolones C. aminoglycosides D. preparations of other groups E. * amino penicillins

540. What percussion data are characteristic for the focal pneumonia? A. bandbox sound over the entire surface of the lungs B. shortening of the percussion sounds at an angle of scapula C. shortening percussion sound in axillar region D. everything is correct E. * clear lung sounds over the entire surface of the lungs

541. Shortening of the percussion sounds in infants with focal pneumonia occurs: A. in 3 - 5 days of illness B. in the first day of illness C. in 10 - 15 days of illness D. in 2 - 3 day of illness E. * in 5 - 10 days of illness

542. At times, the beginning of lobar pneumonia, is misdiagnosed with: A. rheumatism B. bronchial asthma C. pyelonephritis

D. sepsis E. * appendicitis

543. A typical inspection sign in a patient with lobar pneumonia is: A. jaundiced skin B. butterfly rashC. gray color D. everything is correct E. * color, usually on the side of lesion

544. What the inspection reveals in patient with lobar pneumonia: A. synchronous movement of both chest partsB. "healthy" part of the chest lag in the breathing C. spasmodic twitching of hands and feet D. everything is wrong E. * "sick" part of the chest lag in the breathing

545. Staphylococcal pneumonia develops mainly in: A. adolescents B. preschoolers C. infantsD. everything is correct E. * schoolchildren

546. Obstructive respiratory failure is caused by: A. alveolar lesion B. violation of the breathing neuromuscular control C. capillary pulmonary circulation lesionD. everything is correct E. * development of the respiratory tract mucous edema

547. Parenchymatous respiratory failure is caused by: A. changes in respiratory muscle B. compression of the airway from the outside C. impairment of the respiratory center D. everything is correct E. * lesion of the alveoli and capillary pulmonary circulation

548. Ventilating respiratory failure is caused by: A. the presence of bronchospasm B. aspirated foreign body C. inflammatory lung diseases D. everything is correct E. * violation of the breathing neuromuscular control

549. Treatment of respiratory failure provides oxygen, the oxygen concentration must not exceed: A. 80% B. 70% C. 90% D. everything is correct E. * 60%

550. The absence of respiratory sounds during inspiration is characteristic for: A. Respiratory insufficiency 1 degree B. Respiratory insufficiency 2 degree C. Respiratory insufficiency 0 degree D. everything is correct E. * Respiratory insufficiency 3 degree

551. Bradipnoe is characteristic for: A. Respiratory insufficiency 1 degree B. Respiratory insufficiency 2 degree

C. Respiratory insufficiency 0 degree D. everything is correct E. * Respiratory insufficiency 3 degree

552. Value of pulse to the breathing 3,5 - 2,5: 1 is characteristic for: A. Respiratory insufficiency 1 degree B. Respiratory insufficiency 0 degree C. Respiratory insufficiency 3 degree D. everything is correct E. * Respiratory insufficiency 2 degree

553. Value of pulse to the breathing 2,5 - 2: 1 is typical for: A. Respiratory insufficiency 1 degree B. Respiratory insufficiency 2 degree C. Respiratory insufficiency 0 degree D. everything is correct E. * Respiratory insufficiency 3 degree

554. The most common cause of asthma is: A. medications B. foodstuffs C. bacterial allergy D. chemicals E. * house dust

555. The main source of antigens in house dust is: A. dry food for aquarium fish B. wool and scurf of animal C. hair and scurf of people D. everything is wrong E. * house dust mites

556. Mostly house dust mites are found in: A. soft toys B. upholstery C. bed linenD. pillows E. * carpets

557. The cardinal symptom of asthma is: A. itchy nose B. constant sneezing C. dry compulsive cough D. running nose E. * dyspnea

558. Mostly dyspnea in asthma patient occurs: A. at dawn B. in the morning C. by the day D. during meal E. * at night

559. What is the base anti-inflammatory therapy in mild bronchial asthma?A. oral corticosteroids B. intravenous corticosteroids C. nonsteroidal anti-inflammatory drugs D. methylxanthines E. * cromoglycate sodium

560. What inflammation of the mucous membrane is typical for bronchial asthma? A. infectious

B. infectious - allergic C. mixed D. everything is wrong E. * allergic

561. What type of dyspnea is typical for bronchial asthma? A. inspiratory B. mixed C. Schick D. Kussmaul E. * expiratory

562. What peripheral blood changes are typical for bronchial asthma?A. anemia B. leukocytosis C. lymphocytosis D. monocytosis E. * eosynophylia

563. What is reveal in percussion during bronchial asthma attack? A. expansion of the heart borders B. clear lung sound C. local shortening of lung sound D. mosaic changes E. * bandbox sound over the lungs

564. An asthmatic status means asthma attack duration: A. for 24 hours B. more than 2 hours C. more than 5 hours D. more than 10 hours E. * more than 6 hours

565. What is the aminophylline (euphyllin) dose in the mild attack of bronchial asthma? A. 1 - 2 mg / kg B. 10 - 12 mg / kg C. 24 mg / kg D. 0,5 - 1 mg / kg E. * 4 mg / kg

566. What antihystamines are used in bronchial asthma?A. diphenhydramine B. klaritin C. allergodyl D. everything is correct E. * calcium gluconate

567. What is the aminophylline (euphylline) dose in asthmatic status? A. 15 - 16 mg / kg / day B. 40 mg / kg / day C. 10 mg / kg / day D. 5 - 7 mg / kg / day E. * 24 mg / kg / day

568. Name the form of bronchial asthma according to the classification A. allergic B. nonatopic C. infectious D. recurrent E. * persistent

569. What prevails in infants in asthma attack?

A. bronchospasm B. engorgement of the lungs C. emphysema D. pneumorrhagia E. * edema of the respiratory tract mucosa

570. What prevails in schoolchildren in asthma attack? A. edema of the respiratory tract mucosaB. respiratory violationC. allergic inflammation D. engorgement of the lungs E. * bronchospasm

571. What is the aminophylline (euphylline) dose in the moderate attack of bronchial asthma?A. 10 mg / kg B. 15 mg / kg C. 3 mg / kg D. 1 - 2 mg / kg E. * 5 - 7 mg / kg

572. What is the prednisolon dose in asthmatic status?A. 2 - 3 mg / kg / day B. 6 - 8 mg / kg / day C. 3 - 5 mg / kg / day D. 0,5 - 1 mg / kg / day E. * 1 - 2 mg / kg / day

573. What is recommended for children in the period of asthma remission? A. antihystamines B. antibacterial therapy C. bronchodilator therapy D. nothing E. * anti-inflammatory therapy

574. Severe asthma requires the appointment of: A. aminophylline (euphylline) B. intal C. adrenaline D. ephedrine E. * glucocorticoids

575. What is the way of specific asthma allegro diagnostic? A. inhalation tests B. determining the percentage of eosinophils in the blood C. allergenic anamnesis D. Mantoux test E. * skin allergic tests

576. What is the atopy marker in children? A. elevated levels of immunoglobulin A B. eosinophilia C. leukopenia D. monocytosis E. * elevated levels of immunoglobulin E

577. Anti-inflammatory therapy of bronchial asthma is continued for: A. 6 months B. 2 weeks C. 1 year D. all the life E. * not less than two months

578. Which inhalers are used in the treatment of bronchial asthma? A. aminophylline B. ingalipt C. amerton D. instaryl E. * salbuthamol

579. Children with asthma are at the dispensary supervision by: A. pediatrician B. immunologist C. pulmonologist D. everything is wrong E. * allergist

580. One of the criteria for asthmatic status is: A. attack more than 10 hours B. acute adrenal insufficiency C. heart failure D. chronic pulmonary heart E. * violation of bronchial drainge function

581. What mainly leads to disruption of all organs and systems in asthma children? A. surfactant deficiency B. hemodynamics violation C. low immunity D. anemia E. * hypoxia

582. First place among the allergens that cause hives, take: A. Pollen B. Epidermal allergens C. Medicines D. Physical factors E. * Foodstuffs

583. When urticaria next changes are developed: A. The lesion of the subcutaneous layer of the dermis B. The lesion of the submucosal layer of the dermisC. The lesion of the subcutaneous tissueD. Lesion in the connective tissue of the internal organs E. * Increased microvascular permeability

584. Quincke’s edema affected deeper layers of skin in the following places, except: A. Face B. Head C. Neck D. Genitalia E. * The lateral surface of the body

585. In acute allergic urticaria rash occurs after contact with an allergen through: A. Few seconds B. Few minutes C. Few days D. Few weeks E. * Few hours

586. Elements of urticaria often are coloured as: A. Cyanotic B. Icteric C. Scarlet D. Umber

E. * As skin587. Elements of urticaria preserved for:

A. 20-30 seconds B. 5-6 minutes C. 2-3 days D. 1-2 weeks E. * 1-6 hours

588. Quincke's edema can preserved for: A. Up to 10-20 seconds B. 30-40 minutes C. 1-2 hours D. 1-2 weeks E. * 2-3 days

589. The primary element of the rash of acute hives are: A. Papule B. Pustule C. Spot D. Crust E. * Urtica

590. Base symptomatic treatment of urticaria consists in the application of: A. Antibacterial drugs B. Anti-inflammatory drugs C. Sedatives D. Desintoxication drugs E. * Antihystamines

591. Which system pathology is often in children with chronic urticaria? A. Cardiovascular B. Respiratory system C. Nervous system D. Locomotor E. * Digestive

592. What color is Quincke’s edema?A. Pale B. Cyanotic C. Gray D. Icteric E. * The same as skin

593. What is the main localization of children's eczema? A. Feet B. Elbows C. AbdomenD. Knees E. * Cheeks

594. When skin process mostly disappears in children’s eczema?A. Up to 1 year B. Up to 2 years C. Up to 3 years D. Up to 4 years E. * Up to 5 years

595. In atopic dermatitis exudative skin lesions are typical to the next age group: A. 2-4 years B. 4-6 years C. 6-8 years

D. 8-15 E. * Before 2 years

596. In atopic dermatitis erythematous-squamous skin lesions aretypical to the next age group: A. 0-3 months B. 3-9 months C. 9-18 monthsD. 12-24 months E. * 2-15 years

597. First place among the concomitant disease in atopic dermatitis belongs to the diseases of:A. Nervous system B. Respiratory systemC. Endocrine systemD. Urinary System E. * Digestive system

598. In the patient with atopic dermatitis is increased: A. Ig A B. Ig G C. Ig M D. T cells E. * Ig E

599. What is the evidence of the rheumatic fever streptococcal etiology? A. increased seromucoid B. increased sialic acids C. increased LDH D. positive formol test E. * increased antihyaluronidase

600. In rheumatism pathogenesis the leading role belongs to: A. bacterial infection B. aseptic inflammation C. allergic reactions D. everything is correct E. * immune reactions

601. What is pathogenetic link of rheumatism? A. intracellular sensitization B. constant persistence of streptococci in the blood C. persistence of streptococci in the connective tissue D. everything is wrongE. * primary streptococcal sensitization

602. Which heart lining are often affected with rheumatic disease in children? A. endocardium B. myocardium C. all D. everything is wrong E. * endomyocardium

603. What heart disease most often is formed on the background of rheumatic fever? A. stenosis of the aorticvalve B. aortic valve insufficiency C. tricuspid valve insufficiency D. pulmonary valve insufficiency E. * mitral insufficiency

604. What are peculiarities of rheumatism in children?A. only polyarthritis B. prolonged duration

C. latent course D. subacute course E. * formation of the heart defects

605. What are peculiarities of rheumatism in children?A. only polyarthritis B. only nodules C. latent course D. arthralgia E. * chorea

606. The main criteria of rheumatic fever are: A. hepatitis B. nephritis C. pneumonia D. meningitis E. * carditis

607. The main criteria of rheumatic fever are: A. meningitis B. hepatitis C. dermatitis D. pneumonia E. * polyarthritis

608. The main criteria of rheumatic fever are: A. urticaria B. hemorrhages C. pneumonia D. bleeding E. * erythema marginatum

609. Additional criteria of rheumatism are: A. hematoma B. bleeding into the joints C. petechia D. chorea E. * nosebleeds

610. Additional criteria of rheumatism are: A. arthritis B. muscle pain C. backache D. headaches E. * arthralgia

611. Criteria for rheumatic carditis are: A. only myocarditis B. cardialgia C. epicardium lesionD. only pericarditis E. * lesion of the myocardium and endocardium

612. What is typical for myocarditis? A. decreased heart size B. increased heart tones C. WPW syndrome D. CLC syndrome E. * expansion of the heart borders

613. Which joints are often affected with rheumatism? A. interphalangeal

B. mandibular C. cervical spine D. lumbar spine E. * knee

614. Rheumatic arthritis is characterized by: A. morning stiffness B. resistant strain of joints C. spine lesionD. contractures E. * volatility of joint damage

615. Chorea is characterized by: A. violation of consciousness B. central paralysis C. lesion of cranial nerves D. decrease of tendon reflexes E. * muscular hypotonia

616. Chorea is characterized by: A. impaired consciousness B. central paralysis C. lesion of cranial nerves D. decrease of tendon reflexes E. * poor coordination

617. Rheumatic endocarditis is characterized by: A. accent II tone of the aorta B. soft systolic murmur at apex C. systolic murmur over the pulmonary artery D. diastolic murmur on the apex E. * rough systolic murmur at apex

618. Rheumatic pancarditis is characterized by: A. increased heart sounds B. increased BP C. decrease in heart size D. accent II tone of the aorta E. * significant cardiomegaly

619. The ECG for rheumatism is characterized by: A. lengthening the QT interval B. deformation of the QRS complex C. increasing of the voltage D. syndrome CLC E. * lengthening of the PQ interval

620. What is an antibiotic of choice in rheumatism? A. ampicillin B. gentamicin C. kefzol D. ceftriaxone E. * penicillin

621. What is the dose of penicillin for rheumatism per kg? A. 200-250 thousand units B. 100-150 thousand units C. 10-20 thousand units D. 250 - 350 thousand unitsE. * 30-50 thousand units

622. What is advisable to appoint in acute rheumatic fever?

A. plaquenil B. delagil C. paracetamol D. sigan E. * ibuprofen

623. What is advisable to appoint in protracted rheumatic fever?A. aspirin B. voltarenC. ibuprofen D. paracetamol E. * plaquenil

624. What is advisable to appoint for treatment of rheumatism if you are allergic to penicillin? A. gentamicin B. klaforan C. chloramphenicol D. ceftriaxone E. * azithromycin

625. The dose of prednisolone in severe rheumatic carditis is: A. 1 mg / kg B. 3 mg / kg C. 4 mg / kg D. 5 mg / kg E. * 2 mg / kg

626. The duration of outpatient (dispensary) treatment of rheumatic fever depends on: A. process activity B. duration of attack C. child’s age D. everything is wrong E. * development of complications

627. What is used for year-round prevention of rheumatism?A. penicillin B. aspirin C. delagyl D. ampicillin E. * bicillin

628. What is bicillin-1 dose for schoolchildren?A. 600 thousand units 2 times a monthB. 600 thousand IU 1 time per month C. 1200 thousand units 2 times a month D. everything is correct E. * 1200 thousand units 1 time per month

629. What is bicillin-1 dose for preschoolers?A. 600 thousand IU 1 time per month B. 1200 thousand units 2 times a month C. 1200 thousand units 1 time per month D. everything is correct E. * 600 thousand units 2 times a month

630. The duration of outpatient (dispensary) treatment of complicated rheumatic fever is:A. 2 years B. 3 years C. 4 years D. 1 year E. * 5 years

631. The duration of outpatient (dispensary) treatment of uncomplicated rheumatic fever is:A. 2 years B. 4 years C. 5 years D. 1 year E. * 3 years

632. Name the complication of rheumatic disease A. myocardial infarction B. hypertonic disease C. myocardial distrophyD. everything is correct E. * myocardiosclerosis

633. Plaquenil dose per kg of body weight is:A. 3mg B. 5mg C. 10mg D. 1 mg E. * 8mg

634. What is the pathogenesis of rheumatoid arthritis?A. Bone osteomalacia B. Metabolic abnormalities in the bones C. Acute infectious inflammation of the joints D. Systemic connective tissue dysplasia E. * Autoimmune processes in the connective tissue

635. Rheumatoid arthritis is characterized by: A. No changes in the bones and joint surfaces B. Hemarthrosis availability C. The absence of effusion in the joint cavity D. Expansion of joint space E. * Narrowing of joint space

636. What means “pannus" in rheumatoid arthritis? A. Edema and hyperemia around the affected joint B. The form of joint deformation C. Effusion in the joint cavity D. The form of rheumatoid spine lesions E. * Microvilli proliferation of the synovial membrane

637. Heart damage in rheumatoid arthritis is most often seen as: A. Left ventricular hypertrophy B. Formation of mitral stenosis C. Infringement of the coronary circulation D. Development of acute cardiac insufficiency E. * The development of myocarditis

638. Which of the joints are most often affected in patients with rheumatoid arthritis? A. Ankles B. Sternocostal C. Shoulders D. Hips E. * Knees

639. Which of the following is typical rheumatoid arthritis clinic in children (unlike adults)? A. Frequent lesion of small hand joints B. Deformation of joints C. The symmetry of joint damage D. Less developed of mono- and pauciarticular form

E. * Frequent injury of the cervical spine 640. Which of the joints in JRA is deformed in globular?

A. Elbow B. Radial-carpalC. Shoulder D. Hip E. * Knee

641. Which of the joints in JRA is deformed in a spindly? A. Knee B. Radial-carpal C. Shoulder D. Hip E. * Elbow

642. The lesion of the cervical spine in JRA usually occurs at the level of: A. 1st vertebrae B. 2-3 vertebrae C. 4th vertebrae D. 7th vertebrae E. * 5-6 vertebrae

643. Violation of the bones growth in JRA occurs: A. Throughout the skeleton B. In the lower extremities C. Mainly in the flat bones D. In areas remote from the lesion E. * In areas that border to the affected joints

644. The leading symptoms of Still's syndrome are: A. Eye B. Nephritic syndrome C. Carditis and vasculitis D. The lesion of the spine E. * Fever and rash

645. Seropositive form of JRA is defined by: A. Leukocytosis in the blood B. A positive CRP C. Increased levels seromucoid D. By increasing the ESR E. * A positive rheumatoid factor

646. Systemic forms of JRA usually are characterized by: A. Meningitis B. Nephritic syndrome C. Abdominal syndrome D. Pneumonia E. * Polyserositis

647. What JRA eye damage is mainly manifested? A. conjunctivitis B. Retinopathy C. Retinal degeneration D. Cataracts E. * Iridocyclitis

648. What clinical triad often accompanies systemic form of JRA? A. Arthralgia, chorea, morning stiffness B. Arthritis, eye disease, carditis C. Rash in the area of the affected joint, fever, heart damage

D. Resistant articular syndrome, carditis, fever E. * Intermittent fever, rash at the height of fever, arthralgia

649. What from ennumerated is a clinical diagnostic criterion of JRA? A. Carditis B. Chorea C. Rheumatic nodules D. Erythema marginatum E. * Muscle atrophy

650. Which of the following drugs is a part of the JRA basic treatment? A. Hydrocortisone B. Ketotifen C. Diclofenac D. Indomethacin E. * Methotrexate

651. Non-rheumatic carditis in children most often affects: A. all of the heart lining B. endocardium C. pericardium D. endo-and pericardium E. * myocardium

652. Early congenital carditis is formed in gestational age of: A. 1 - 3 months B. 7 - 9 months C. At birth D. 10 - 12 months E. * 4 - 6 months

653. Congenital heart defects are formed in gestational age of: A. 4 - 6 months B. 7 - 9 months C. At birth D. 10 - 12 months E. * 1 - 3 months

654. Late congenital carditis is formed in gestational age of: A. 1 - 3 months B. 4 - 6 months C. At birth D. 10 - 12 months E. * 7 - 9 months

655. Fibroelastosis in a child may be suspected when there is: A. rough systolic murmur in the 5-th point B. stable bradyarrhythmia C. frequent extrasystoles D. rough systolic murmur at the apex of the heart E. * resistant, refractory to therapy, tachycardia

656. Fibroelastosis may be suspected in achild when there is: A. kyphoscoliosis B. pectus excavatum C. sinistral lordosis D. pigeon chest E. * “Heart hump”

657. What is the main complication of fibroelastosis? A. pulmonary insufficiency B. cardiosclerosis

C. ventricular extrasystoles D. arterial hypertension E. * heart failure

658. What is the prognosis of fibroelastosis?A. recovery B. death in the early school age C. death at a young age D. everything is correct E. * death up to 2 years

659. What is the drug of choice for fibroelastosis?A. aspirin B. prednisolone C. delagyl D. penicillin E. * digoxin

660. Symptomatic therapy of fibroelastosis requires the appointment of: A. hypotensive B. enzymes C. antiarrhythmic medicineD. antibiotics E. * cardiac glycosides

661. Symptomatic therapy of fibroelastosis requires the appointment of: A. hypotensive B. vitamins C. antiarrhythmic medicineD. antibiotics E. * diuretics

662. By what is characterized the clinic of the late congenital carditis? A. significant cardiomegaly B. progressive tachycardia C. persistent bradycardia D. everything is wrong E. * multiple arrhythmias

663. Acute non-rheumatic carditis in young children mainly is caused by: A. bacteria B. toxins C. allergic conditions D. fungi E. * viruses

664. Acute non-rheumatic carditis is characterized by: A. decreased heart borders B. elevated blood pressure C. increased heart sounds D. everything is correct E. * cardiac rhythm impairment

665. Acute non-rheumatic carditis is characterized by:A. organic systolic murmur B. elevated blood pressure C. increased heart sounds D. rough systolic murmur on the apex E. * functional systolic murmur

666. The ECG in acute carditis shows: A. lengthening PQ

B. shortening PQ C. increased voltageD. everything is wrong E. * decreased voltage

667. The ECG in acute carditis shows: A. lengthening PQ B. shortening PQ C. increased voltage D. everything is correct E. * ventricular extrasystols

668. The left ventricular heart failure is characterized by: A. hepatomegaly B. swelling of the neck veins C. swelling of the hands veins D. edema on feet E. * moist rales in the lungs

669. The right ventricular heart failure is characterized by: A. wet cough B. moist rales in the lungs C. accented 2nd tone of the LA D. hemoptysis E. * swelling of the neck veins

670. The left ventricular heart failure is characterized by: A. hepatomegaly B. swelling of the neck veins C. swelling of the hands veins D. accented 2nd tone of the aortha E. * accented 2nd tone of the LA

671. The 2nd -B st of the heart failure in non-rheumatic carditis is characterized by: A. dyspnea on physical loadB. anasarca C. dry cough D. everything is wrong E. * moist rales in the lungs

672. The 3rd stage of the heart failure in non-rheumatic carditis is characterized byA. pneumonia B. meningitis C. hepatitis D. splenomegaly E. * pulmonary edema

673. The 2nd -A st of the heart failure in non-rheumatic carditis is characterized by: A. swellings on the legs B. anasarca C. dry cough D. hepatomegaly E. * dyspnea at rest

674. The 1 st of the heart failure in non-rheumatic carditis is characterized by: A. swellings on the legs B. anasarca C. dry cough D. cardiac asthma E. * dyspnea on physical load

675. What is echocardioscopic sign in fibroelastosis?

A. hyperkinetic myocardial areas B. hypertrophy of the left ventricle C. the presence of fluid in the pericardium D. anatomical defects of the heart E. * increased diastolic volume of the left ventricle

676. What is echocardioscopic sign in acute carditis? A. hyperkinetic myocardial areas B. hypertrophy of the left ventricle C. dilatation of the left ventricle D. everything is wrong E. * decreased ejection fraction

677. What is echocardioscopic sign in chronic carditis? A. akinetic myocardial areas B. hyperkinetic myocardial areas C. the presence of fluid in the pericardium D. anatomical defects of the heart E. * hypertrophy of the left ventricle

678. What is the drug of choice for severe acute carditis? A. digoxin B. delagyl C. indomethacin D. penicillin E. * prednisolone

679. What is the dose of prednisolone in acute myocarditis? A. 1,5 - 2,5 mg / kg B. 2,5 - 3,5 mg / kg C. 3,5 - 4,5 mg / kg D. everything is correct E. * 0,5 - 1,5 mg / kg

680. What is the dose of delagyl at carditis? A. 1 - 2 mg / kg B. 3 - 4 mg / kg C. 7 - 8 mg / kg D. everything is wrong E. * 5 - 6 mg / kg

681. What is the maintenance dose of digoxin from the dose-saturation: A. 1/2 - 1/3 B. 1/3 - 1/4 C. 1/5 - 1/6 D. 1/7 - 1/8 E. * 1/4 - 1/5

682. Which drugs improve myocardial function?A. prednisolone B. corglycon C. methyluracil D. digoxin E. * mildronate

683. Ventricular septal defect is characterized by: A. weakened 2nd tone of the pulmonary arthery B. accented 2nd tone of the aorta C. weakened 2nd tone of the aorta D. everything is wrong E. * accented 2nd tone of the pulmonary arthery

684. Ventricular septal defect is characterized by: A. accented 2nd tone of the aorta B. weakened 2nd tone of the aorta C. soft systolic murmur over the apex D. rough systolic murmur over the pulmonary arthery E. * rough systolic murmur over the apex

685. Atrial septal defect is characterized by: A. weakened 2nd tone of the pulmonary arthery B. accented 2nd tone of the aorta C. weakened 2nd tone of the aorta D. everything is wrong E. * accented 2nd tone of the pulmonary arthery

686. Atrial septal defect is characterized by: A. rough systolic murmur over the apex B. accented 2nd tone of the aorta C. weakened 2nd tone of the aorta D. rough systolic murmur over the pulmonary arthery E. * soft systolic murmur over the apex

687. Patent ductus arteriosus is characterized by: A. rough systolic murmur over the apex B. weakened 2nd tone of the aorta C. soft systolic murmur over the apex D. rough systolic murmur over the pulmonary arthery E. * systolic-diastolic murmur of the pulmonary arthery

688. Ventricular septal defect belongs to the group of the congenital heart disease with: A. a shunt to the left B. an obstacle to blood flow C. mixing of blood in the atria D. mixing of blood in the aorta E. * a shunt to the right

689. The disease of Fallot belongs to the group of the congenital heart disease with: A. a shunt to the rightB. an obstacle to blood flow C. mixing of blood in the atria D. mixing of blood in the pulmonary arthery E. * a shunt to the left

690. The disease of Fallot is characterized by: A. pale skin B. acrocyanosis C. paratrophia D. anemia E. * dyspneic-hypercyanotic attacks

691. The disease of Fallot is characterized by: A. pale skin B. acrocyanosis C. thrombocytopenia D. anemia E. * polycythemia

692. What is x-ray finding in the Fallot disease? A. increased lung pattern B. ribs "rosary" C. reducing the size of the heart D. "uzures" on the ribs

E. * weakening of the lung pattern693. What is x-ray finding in coarctation of the aorta?

A. weakening of the lung patternB. increased lung pattern C. ribs "rosary" D. reducing the size of the heart E. * "uzures" on the ribs

694. What is x-ray finding in ventricular septal defect? A. weakening of the lung patternB. ribs "rosary" C. reducing the size of the heart D. "uzures" on the ribs E. * increased lung pattern

695. Coarctation of the aorta is characterized by: A. arterial hypotension B. paresthesia in the fingers C. muscular hypotonia of the upper extremities D. everything is wrong E. * arterial hypertension

696. Coarctation of the aorta is characterized by: A. arterial hypotension B. paresthesia in the fingers C. muscular hypotonia of the upper extremities D. everything is wrong E. * paresthesia in the lower extremities

697. What belongs tothe functional gastric disorders? A. Chronic esophagitis B. Chronic gastritis C. Stomach ulcer D. Chronic duodenitis E. * Duodenogastric reflux

698. Gastroesophageal reflux belongs to:A. Functional disorders of the stomach by the secretory type B. Chronic gastritis C. Biliary dyskinesia D. Chronic duodenitisE. * Functional disorders of the stomach by the motor type

699. What is the main pathology in gastroesophageal reflux disease? A. The pain behind the sternum B. Nausea C. Vomiting D. Flatulence E. * Heartburn

700. What is the main method for diagnosis of gastroesophageal reflux disease? A. EGDS B. Ultrasound C. X-ray of the esophagus D. Duodenal probing E. * PH-metry of the esophagus

701. Prescribe prpeparat to a patient with gastroesophageal reflux to improve motility of the stomach: A. Almagel B. Gastronorm C. Quamatel

D. No-spa E. * Motilium

702. Prescribe prpeparat tocorrect secretory disorders in reflux esophagitis: A. Motilium B. No-spa C. Gastropharm D. Smectic E. * Quamatel

703. Etiology of chronic gastroduodenitis A. streptococcus B. staphylococcus C. E. coli D. Candida E. * Helicobacter pylori

704. Name agression factors of gastric mucosa: A. mucous discharge B. prostaglandin E2 C. bicarbonate D. everything is wrong E. * hydrochloric acid

705. What is the regulator of hydrochloric acid secrection? A. somastatin B. glucagon C. insulin D. trypsin E. * gastrin

706. Name protectors of gastric mucosa: A. pepsin B. hydrochloric acid C. gastrin D. trypsin E. * mucous discharge

707. Name protectors of gastric mucosa: A. hydrochloric acid B. pepsin C. gastrin D. everything is correct E. * secretory Іg A

708. What belongs to the pathogenesis of gastritis? A. increased mucus formation B. increased secretion of bicarbonate C. physiological blood flow in the mucous D. decreased secretion of hydrochloric acidE. * increased secretion of hydrochloric acid

709. Helicobacter pylori colonizes: A. stomach cardiaB. fundus of the stomach C. pylorus D. everything is correct E. * antrum

710. Helicobacter pylori produces an enzyme: A. lactase B. peptidase

C. lipase D. amylase E. * urease

711. The enzyme urease hydrolyzed in the gastric content: A. pepsin B. HCl C. mucus D. bile E. * urea

712. Three leading syndromes of chronic gastritis are:A. pain, dyspeptic, hemorrhagic B. dysuria, dyspeptic, toxic C. pain, bleeding, dysuria D. everything is correct E. * pain, dyspeptic, toxic

713. Which syndrome is more pronounced in patients with gastritis that has increased secretion? A. dyspepsiaB. toxicC. disuric D. haemorrhagic E. * pain

714. In patients with increased secretion in gastritis pain is:A. mild B. dull C. moderate D. absent E. * intensive

715. In patients with reduced secretion in gastritis pain is:A. intensive B. acute C. absent D. everything is wrong E. * mild

716. In patients with gastritis pain occurs through: A. 30-45 min after meals B. 45-60 min after meals C. 1-1,5 hours after meals D. everything is wrong E. * 15-30 min after meals

717. In patients with duodenitis pain occurs more frequently through: A. 15-30 min after meals B. 30-45 min after meals C. 45-60 min after meals D. everything is correct E. * 1-1,5 hours after meals

718. Night pain is typical for patients with:A. normal acid-function B. reduced acid-function C. achlorhydria D. everything is correct E. * high acid-function

719. The prevalence of pain over the dyspeptic syndrome is characteristic gastroduodenitis with: A. normal acid-function

B. reduced acid-function C. achlorhydria D. everything is correct E. * high acid-function

720. The prevalence of dyspeptic over the pain syndrome is characteristic for gastroduodenitis with: A. high acid-function B. normal acid-function C. saved acid-function D. everything is wrong E. * reduced acid-function

721. Which of the syndromes is the most constant in chronic gastroduodenitis in children? A. dyspeptic B. intoxication C. epithelial D. syderopenic E. * pain

722. Severity of dyspeptic manifestations in chronic gastroduodenitis in children depends on: A. age of the child B. dietary peculiaritiesC. intervals between meals D. balance of food ingredients E. * secretory function

723. What is most often observed in the reduced acidity of gastric juice? A. tendency to constipation B. "sour" burpingC. "Hunger pains" D. vomiting E. * unstable stols

724. What secretory function is the most characteristic in childhood gastritis? A. Reduced B. reduced and normal C. increased D. conservation E. * increased and normal

725. Which of the methods is the most important for the diagnosis of chronic gastroduodenitis? A. pH meters B. fractional test of gastric contents C. atsidotest D. X-ray E. * EGDS

726. Which of the methods of Helicobacter pylori infection diagnostic belong to invasive? A. feces and saliva study by polymerase chain reaction B. breath tests C. immunoglobulins establishment D. “Aerotest” E. * Bacteriological study of gastric mucosa biopsy

727. Which of the methods of Helicobacter pylori infection diagnostic belong to non-invasive? A. urease test B. De-nol-test C. study of nucleic acids D. biopsy of the gastric or duodenum mucosa E. * "Aerotest"

728. What determines the duration of bed rest in the treatment of chronic gastroduodenitis children?

A. severity of dyspeptic manifestations B. quantity of secretion C. age of the child D. endoscopic changes E. * the severity of pain

729. Which antacid drugs is the most appropriate to be used in children?A. sodium bicarbonate B. calcium carbonate C. magnesium sulfateD. carbon dioxide E. * maalox

730. What almagel A dose is prescribed for children under 10 years? A. ? tablespoon 3 times a day B. ? teaspoon 4 times a day C. 1 tablespoon 3 times a day D. 1 dessert spoon 3 times a day E. * 1 teaspoon 4 times a day

731. Which of the products belongs to the blockers of histamine H2-receptor? A. maalox B. vikalin C. methacine D. gastropharm E. * cimetidine

732. What is the daily dose of cimetidine used in the treatment of chronic gastroduodenitis children? A. 1 mg / kg B. 3 mg / kg C. 5 mg / kg D. 20 mg / kg E. * 10 mg / kg

733. What kind of drugs refers to reparants? A. panzynorm B. renegast C. bellaspon D. ranitidine E. * gastropharm

734. What physiotherapy procedure can be applied to children with chronic gastroduodenitis with the expressed pain syndrome? A. ozokerite applications B. xylitol probingC. diathermy D. mud E. * electrophoresis of novocaine

735. Which of the following drugs are attributed to Hpylori? A. maalox B. gastropharm C. cimetidine D. penicillin E. * amoxacilline

736. What is the leading symptom in the 1st stage ulcer? A. vomiting B. belching C. heartburn D. local muscle tension

E. * pain 737. What characteristic of dyspeptic manifestations at the healing stage of ulcer?

A. expressed B. nausea C. acid regurgitation D. heartburn E. * absent

738. What feature of the ulcer disease course is typical for children? A. mild B. latent course C. sluggish course D. chronic courseE. * as a rule severe

739. What are peculiarities of the peptic ulcer, mild course? A. term healing of ulcers up to 2 weeks, with remission up to 6 months B. term healing of ulcers up to 2 months with remission less than 1 year C. term healing of ulcers 1 month, relapse up to 2 times a year D. term healing of ulcers up to 2 months, relapse once a year E. * term healing of ulcers up to 1 month with remission for over a year

740. What are peculiarities of the peptic ulcer, moderate course? A. term healing of ulcers up to 1 month with remission for over a year B. term healing of ulcers up to 2 weeks, with remission up to 6 months C. term healing of ulcers 1 month, relapse up to 2 times a year D. term healing of ulcers up to 2 months, relapse once a year E. * term healing of ulcers up to 2 months with remission less than 1 year

741. What are peculiarities of the peptic ulcer, severe course? A. term healing of ulcers up to 1 month with remission for over a year B. term healing of ulcers up to 2 weeks, with remission up to 6 months C. term healing of ulcers up to 2 months with remission less than 1 year D. term healing of ulcers up to 2 months, relapse once a year E. * term healing of ulcers 1 month, relapse up to 2 times a year

742. What is the drug of choice in Helisobaster pylori invasion? A. penicillin B. gentamicin C. cefazolin D. erythromycin E. * metronidazole

743. What is the most frequent complication of peptic ulcer in children? A. stenosis of pylorus B. penetration C. perforation D. peryvistseritis E. * bleeding

744. Name radiological signs of gastric ulcers: A. radial convergence of the of the stomach wallsB. thickening of the walls C. thickening of the mucosal folds D. smoothed mucous folds E. * "niche" with inflammatory shaft

745. What is the duration of peptic ulcer triple or quadro therapy in children? A. 5 - 7 days B. 10 - 14 days C. 14 - 21 days

D. 21 - 30 days E. * 7 - 10 days

746. Which drug is a reparant of gastric mucosa in duodenal ulcer? A. cimetidine B. renegast C. smectic D. motilium E. * spirulina

747. What is advisable to appoint in gastrointestinal dysmotility? A. no-spa B. smectic C. gastropharm D. almagel E. * motilium

748. When the cytoprotectors are given to children with peptic ulcer? A. from the beginning of treatment B. in case of hyperacidity C. in the process of ulcers healing D. in remission E. * after antihelicobacter therapy

749. Name forms of chronic hepatitis in children A. persistent, active, autoimmune B. medical, autoimmune, alcoholic C. cryptogenic, viral, toxic D. cryptogenic, autoimmune, toxic, alcoholic E. * viral, autoimmune, drug-induced, toxic

750. Name phases of chronic hepatitis activity A. active (minimal, moderate, severe) B. active (mild, moderate, severe), inactive C. active, incomplete clinical and laboratory remission, complete clinical and laboratory

remission D. incomplete clinical and laboratory remission, complete clinical and laboratory remission E. * active (minimal, moderate, severe), inactive

751. What tells us about the minimal activity of chronic hepatitis? A. normal ALT B. ALT 5 times higher than normal C. ALT 5-10 times higher than normal D. ALT levels above the norm more than 10 times E. * ALT 3 times higher than normal

752. Pain in chronic hepatitis is manifested as: A. epigastric fasting painB. nocturnal epigastric pain C. belting pain 1-2 hours after meal D. pain in the left hypochondrium during physical activity E. * pain in the right hypochondrium during physical activity

753. What are the main clinical manifestations of cholestasis syndrome? A. jaundice, hepatosplenomegaly B. pallor, jaundice, hepatosplenomegaly C. abdominal pain, jaundice D. pallor, skin itching E. * jaundice, skin itching

754. Mesenchymal-inflammatory syndrome is characterized by: A. increased AST, ALT

B. prothrombin reductionC. increased alkaline phosphatase D. raised indirect bilirubin E. * dysproteinemia

755. Viferon belongs to:A. Glucocorticoids B. Antibiotics C. AntihystaminesD. Antiinflammatory E. * Interferon

756. What is the main direction of autoimmune hepatitis treatment?A. antiviral therapy B. antibacterial therapyC. immunostimulatory therapyD. antiinflammatory therapyE. * immunosuppressive therapy

757. Name the immunosuppressive therapy of chronic hepatitis B.A. Indomethacin B. Penicillin C. Cholenzym D. Essenciale E. * Azathioprine

758. What are the signs of hypotonic biliary dyskinesia in duodenal probing? A. Increased portion A B. Increased portion C C. Decreased portion BD. Decreased portion C E. * Increased portion B

759. What are the signs of hypertonic biliary dyskinesia in duodenal probing? A. Increased portion A B. Increased portion BC. Increased portion C D. Decreased portion C E. * Decreased portion B

760. What are the signs of hypotonic biliary dyskinesia in sonogram (USE)? A. Increased liver B. Contracted gallbladder C. Reduced liver D. Normal gallbladder E. * Dilated gallbladder

761. What are the signs of hypertonic biliary dyskinesia in sonogram (USE)? A. Increased liver B. Reduced liver C. Dilated gallbladder D. Normal gallbladder E. * Contracted gallbladder

762. What is the treatment of hypotonic biliary dyskinesia? A. Analgesics B. Antispasmodic C. Hepatoprotectors D. Sedative E. * Choleretics and cholekinetics

763. What is the physiotherapy of hypotonic forms of biliary dyskinesia?

A. СаСІ2 electrophoresis B. InductothermyC. Ozokerite applications D. Microwaves therapy E. * MgSO4 electrophoresis

764. What is used in the treatment of hypertonic biliary dyskinesia? A. Analgesics B. Antibiotics C. Hepatoprotectors D. CholekyneticsE. * Antispasmodic and sedatives

765. What is the physiotherapy of hypertonic forms of biliary dyskinesia? A. MgSO4 electrophoresis B. proserin electrophoresis C. Electrical stimulation of the phrenic nerve D. Microwaves therapy E. * Inductothermy

766. What are the leading clinical syndromes in chronic cholecystocholangitis? A. Pain, dysuria B. Toxic, hemorrhagic C. Dyspeptic, dysuria D. Dyspeptic, hemorrhagic E. * Pain, dyspeptic

767. The presence of vascular asterisks is characteristic for: A. Gastritis B. Duodenitis C. Gastroenterocolitis D. Pancreatitis E. * Biliary dyskinesia

768. What determines the pain character in cholecystocholangitis? A. age of the patientB. sex of the patient C. the state of the nervous system D. the duration of the disease E. * type of dyskinesia

769. Dilated intradermal capillaries on the back is characteristic for: A. Biliary dyskinesia B. Gastritis C. Duodenitis D. Pancreatitis E. * Cholecystocholangitis

770. What are the causes of primary chronic pancreatitis? A. Pathology of the stomach B. Pathology of duodenal ulcer C. Pathology of the liver D. Pathology of intestinal E. * Viral-bacterial infection

771. What are the causes of secondary chronic pancreatitis? A. Viral-bacterial infection B. Allergic factors C. Drug therapy induced lesion D. Abdominal trauma E. * Pathology of the liver

772. Feces in chronic pancreatitis are: A. Solid B. Thick C. Putty-like D. Mixed with blood E. * Pasty or liquid

773. What is the normal level of blood serum amylase in children? A. 10-15 g / hour liter B. 16-32 g / hour literC. 33-45 g / hour liter D. 46-60 g / hour liter E. * 5 - 10 g / hour liter

774. Diastasuria is typical for: A. Hepatitis B. Cholecystitis C. Colitis D. Duodenitis E. * Pancreatitis

775. What is the diet in the early days of pancreatitis? A. Diet № 1 B. Diet № 5 C. Diet № 7 D. Diet № 9 E. * Hunger

776. What medicine is used to decrease the pain in pancreatitis?A. Aspirin B. Indomethacin C. Caffeine D. No-spa E. * Baralgin

777. What medicine is used to treat an acute attack of pancreatitis? A. Penicillin B. Heparin C. Vicasol D. Voltaren E. * Contrical

778. What medicine is used for pharmacological suppression of pancreatic function?A. Ranitidine B. Almagel C. Linex D. Quamatel E. * Creon

779. What is the physiotherapy during exacerbation of chronic pancreatitis?A. Ozokerite applications B. Inductothermy C. UHF (ultra high frequency) D. microwaves E. * Gordox electrophoresis

780. What is the leading etiologic factor of chronic hepatitis? A. bacteria B. parasites C. fungi D. prions

E. * viruses 781. What is the leading etiological factor of chronic cholecystocholangitis?

A. feeding characterB. anomaly of the liver C. genetic susceptibility D. congenital malformations E. * biliary dyskinesia

782. What complex of symptoms is characteristic for chronic hepatitis? A. abdominal pain, bleeding, arthritis B. abdominal pain, diarrhea, intoxication C. abdominal pain, splenomegaly, jaundice D. everything is correct E. * abdominal pain, hepatosplenomegaly, bleeding

783. The pathology of biliary tract is characterized by: A. abdominal pain immediately after taking food B. abdominal pain after sleeping C. abdominal pain during urination D. everything is correct E. * abdominal pain after physical exertion

784. The dyspeptic syndrome in the pathology of biliary tract is characterized by: A. diarrhea after drinking milk B. frequent profuse diarrhea C. persistent constipation since birth D. everything is correct E. * recurrent constipation

785. Hypotonic biliary dyskinesia is characterized by: A. hungry, nocturnal pain in the abdomen B. intense, cramping pain in right hypochondrium C. arching pain in umbilical area D. intense night pain E. * heaviness, discomfort in the right hypochondrium

786. Hypertonic biliary dyskinesia is characterized by: A. heaviness, discomfort in the right hypochondrium B. hungry, nocturnal pain in the abdomen C. arching pain in umbilical area D. intense night pain E. * intense, cramping pain in right hypochondrium

787. Chronic pancreatitis is characterized by: A. hepatomegaly, jaundice, recurrent constipation B. splenomegaly, jaundice, diarrhea C. local pain pyloroduodenal zone, anemia, constipation D. intense night pain E. * "girdle" abdominal pain, anemia, and diarrhea

788. What is characteristic for the liver cirrhosis? A. splenomegaly, anemia, hemorrhage B. splenomegaly, anemia, jaundice C. hepatosplenomegaly, anemia, hemorrhage D. hyposplenism E. * hepatosplenomegaly, anemia, jaundice,

789. What is characteristic for the gallstone disease? A. arching, constant pain in the abdomen B. dull, aching pain in right hypochondrium C. hungry pain in right hypochondrium

D. intense night pain E. * intense, cramping pain in the right hypochondrium

790. Which investigations are necessary for patients with hepatobilliary system diseases? A. PH-meters, ultrasound, duodenal probing B. PH-metry, EGDS, ultrasound C. EGDS, ultrasound, laparoscopy D. PH-metry, EGDS, duodenal probing, ultrasound E. * Ultrasound, laparoscopy, duodenal probing

791. Which biochemical tests are necessary for patients with hepatobilliary system diseases? A. total protein, CRP, seromucoid, bilirubin B. total protein and fractions, urea, creatinine, cholesterol C. total protein and fractions, CRP, seromucoid, urea D. total protein and fractions, seromucoid, bilirubin E. * total protein and fractions, transaminases, bilirubin, cholesterol

792. What "cytolysis" syndrome includes? A. increasing levels of cholesterol, direct bilirubin, iron, LDH B. reduction of albumin, cholesterol, bilirubin, CRP C. reduction of iron, CRP, residual nitrogen, AST, ALT D. reduction of iron, CRP, increased AST, ALT E. * increasing AST, ALT, LDH, iron, bilirubin

793. What is observed in reduced synthetic liver function? A. luminemia, hypercholesterolemia, hyperasotemia B. hyperbillirubinemia, hypercholesterolemia, hyperasotemia C. reduction of iron, CRP, increased AST, ALT D. reduction of iron, CRP, residual nitrogen, AST, ALT E. * hypoalbuminemia, reduced levels of fibrinogen and prothrombin

794. What is observed in reduced detoxication liver function? A. hyperbillirubinemia, hypoproteinemia, hypoasotemia B. hyperbillirubinemia, hypoproteinemia, hypophenolemia C. hyperasotemia, hypophenolemia, hyperammoniemia D. hypobillirubinemia, hypoproteinemia, hypophenolemia E. * hyperasotemia, hyperfenolemia, hyperammoniemia

795. Polyclonal hammapathia syndrome is characterized by: A. hyperproteinemia, hypoimmunoglobulinemia, positive sediment samples B. hypoproteinemia, hypoimmunoglobulinemia, hyperbillirubinemia C. hypoproteinemia, hyperimmunoglobulinemia, hyperbillirubinemia D. hypobillirubinemia, hypoproteinemia, hypophenolemia E. * hyperproteinemia, hyperimmunoglobulinemia, positive sediment samples

796. What ultrasound symptom is typical for cholecystocholangitis?A. enlarged liver B. gallbladder deformation C. sediment in the gallbladder D. gallbladder hypotonia E. * thickening of the gallbladder walls

797. What ultrasound symptom is typical for hepatitis?A. deformation of the bile ducts B. thick walls of the bile ducts C. multiple concrements D. liver size reduction E. * diffuse thick structure of the liver

798. On what day of the duodenogastric reflux treatment cholekinetics are prescribed? A. 20 th B. 14 th

C. 10 th D. 2nd E. * 7 th

799. What should be used for electrophoresis in duodenogastric reflux? A. procaine B. magnesium sulfate C. papaverine D. no-spa E. * neostigmine

800. What should be appointed for hypertonic biliary dyskinesia? A. holenzim B. carsil C. convoflavin D. motillium E. * no-spa

801. What should be appointed for hypotonic biliary dyskinesia? A. no-spa B. carsil C. essenciale D. motilium E. * holenzim

802. What is the drug of choice for intestine giardiasis? A. gentamicin B. penicillin C. aspirin D. pyrantel E. * furazolidone

803. Violation of nitrogen excretion renal function in the onset of glomerulonephritis is typical for: A. isolated urinary syndrome B. nephrotic syndrome C. mixed syndrome D. everything is correct E. * nephritic syndrome

804. Excretory urography give us possibility to evaluate (give the most complete answer): A. anatomical status of urinary tract and urodynamic B. calyx-pelvic system stateC. functional ability of urinary tract D. size of the kidneys E. * everything is correct

805. What time after exacerbation of chronic glomerulonephritis resort treatment is possible? A. 3 months B. 12 months C. It is not performed D. 5 years E. * 6 months

806. Which drug is used for basic therapy of acute glomerulonephritis with nephrotic syndrome? A. Curantyl B. Penicillin C. Suprastin D. Ascorutin E. * Prednisolone

807. Which factor is most weighty in the etiology of glomerulonephritis? A. S aureus

B. E. cоlі C. Influenza virus D. hepatitis A E. * Streptococci

808. Which of the pathogenic mechanisms of damage is characteristic for glomerulonephritis? A. bacterial inflammation of the kidney bowls B. immediate type allergic reaction C. violation of the urine passage D. vesicoureteral reflux E. * immune complex glomerular damage

809. The nephritic variant of acute glomerulonephritis is characterized by: A. high proteinuria B. hyperphosphaturia C. pyuria D. bacteriuria E. * hematuria

810. What level of proteinuria is a criterion for nephrotic variant of acute glomerulonephritis? A. more than 1 g / day B. less than 1 g /day C. more than 3 g / l D. up to 1 g / l E. * more than 3 g / day

811. What is characteristic for the isolated urinary syndrome?A. proteinuria up to 2 g/lB. expressed leukocyturiaC. expressed edemaD. arterial hypertensionE. * proteinuria up to 1 g/l

812. By which investigation is it possible to evaluate the state of renal concentration function?A. analysis of urineB. Nechiporenko urine analysisC. ultrasonic investigationD. excretory urographyE. * Zimnitsky test

813. The criterion of acute glomerulonephritis transition in a chronic form is saving of clinical-laboratory signs longer than:A. 6 monthsB. 3 monthsC. 6 weeksD. 9 monthsE. * 1 year

814. The expressed edema syndrome is most characteristic for:A. isolated urinary syndromeB. nephritic variant of acute glomerulonephritis C. mixed form of chronic glomerulonephritaD. gematuricheskoy forms of chronic glomerulonephritisE. * nephrotic form of chronic glomerulonephritis

815. What from this belong to pathogenetic therapy of chronic glomerulonephritis hematuric form?A. leukeranB. prednisolonC. chlorbutinD. imuranE. * delagyl

816. What is the evidence to give immune suppressors at acute glomerulonephritis?A. sudden, acute disease beginning B. presence of expressed proteinuriaC. absence of NSAIDs treatment effect for 3-4 weeks in children with the nephrotic variant

of the acute glomerulonephritis D. violation of renal function at the disease beginningE. * absence of glucocorticoids treatment effect for 3-4 weeks in children with the nephritic

variant of the acute glomerulonephritis 817. At the nephrotic form of acute glomerulonephritis prednizolon is appointed in a maximal

therapeutic dose not less than:A. 3-4 weeksB. 6-8 weeksC. 2-3 daysD. 5-10 daysE. * 2-3 weeks

818. Ultrasonic investigation of which kidney pathology is the most informing?A. hydronefrotic kidneyB. glomerulonephritis, pyelonephritisC. renal pelvic distopia D. everything is uncorrectE. * tumour, renal stones

819. What from this is one of absolute indications to chronic gemodyalisis at chronic kidney insufficiency?A. blood billirubin 150 mmol/l;B. cholesterol more than 10 mmol/l;C. urea 12-15 mmol/l;D. increased potassium to 4 mmol/lE. * blood creatinyne 0,7-0,9 mmol/l;

820. Which casts form from the protein in acid urine pH?A. ErythrocyteB. LeucocyteC. GrainyD. EpithelialE. * Hyalin

821. Leucocyturia in acute glomerulonephritis lasts for:A. 1-2 hoursB. 1-2 daysC. 10-20 daysD. 1-2 monthsE. * 1-2 weeks

822. Which casts are typical for glomerulonephritis?A. LeucocyteB. HyalinC. GrainyD. EpithelialE. * Erythrocyte

823. What dose of heparin is used in the treatment of glomerulonephritis in children? A. 50-100 IU / kg / day B. 150-200 IU / kg / day C. 100-300 IU / kg / day D. 50-500 IU / kg / day E. * 100-150 IU / kg / day

824. What dose of indomethacin is prescribed to children with nephritic variant of acute

glomerulonephritis? A. 1-2 mg / kg / day B. 3-5 mg / kg / day C. 0,5-1 mg / kg / day D. 0,8-1 mg / kg / day E. * 2,5-3 mg / kg / day

825. Which of these products are excluded from the diet 7? A. rice or buckwheat porridge with jam and sugar B. vegetable purees, vegetable soup, egg C. vegetable oil and butter D. fruit E. * meat, fish

826. Pain in the lower abdomen (suprapubic) radiating to the perineum, increasing pain at the end of the urination or after it, are typical for:A. glomerulonephritis B. pyelonephritis C. renal amyloidosis D. urolithiasis E. * cystitis

827. Which of the symptoms are characteristic for pyelonephritis? A. intoxication B. abdominal pain, lumbar C. pyuria D. pathological bacteriuria E. * everything is correct

828. How long is anibacterial therapy after the normalization of urine analysis in primary pyelonephritis?A. 1,5-3 months B. 3-6 months C. 1 year D. 6 months E. * 14 days-1 month

829. What tactics of the physician in leukocyturia identifying in girls: A. appointment of antibacterial therapy B. appointment of urosepticsC. cystoscopy D. descending urography E. * screened for helminths, with the exception of vulvovaginitis

830. Which antibacterial therapy should be preferable in treatment of active phase of pyelonephritis: A. monotherapy by antibiotic B. monotherapy by uroseptic C. phytotherapy D. physiotherapy E. * combined (antibiotic and uroseptic)

831. The secondary pyelonephritis in children most often occurs on the background: A. glomerulonephritis B. acute viral respiratory infection C. systemic lupus erythemathosus D. tonsillitis E. * anomalies of the urinary tract

832. Most of pyelonephritis are caused by: A. klebsiella B. staphylococcus

C. streptococcus D. Proteus E. * E. coli

833. Which feature of back pain in pyelonephritis? A. diffuse B. bilateral C. irradiate in the sacral region D. in the form of attacks E. * unilateral

834. Which syndrome is crucial for the diagnosis of pyelonephritis A. toxic B. pain C. dysuria D. astheno-vegetative E. * uric

835. What level of proteinuria is typical for pyelonephritis? A. up to 0,5 g / l B. up to 2 g / l C. up to 3 g / day D. up to 1 g / day E. * up to 1 g / l

836. Diagnostic criteria for pyelonephritis in the general analysis of urine is: A. crystaluria B. hematuria C. cylindruria D. glycosuria E. * piuria

837. What is the value of microbial numbers (number of bacteria in 1 ml of urine) is a criterion for pyelonephritis? A. 5,000,000 or more B. 10,000 or more C. 50,000 or more D. 1000 and more E. * 1,000,000 or more

838. What ultrasound of the kidneys in children with primary pyelonephritis is typical? A. reduced size of kidneys B. presence of salt crystals C. thick structure of the kidneys D. thinning of the parenchyma E. * pyeloectasia

839. What investigation is the most informative for urinary tract abnormalities diagnosis? A. ultrasound B. cystoscopy C. cystography D. thermography E. * excretory urography

840. What diet is appropriate at pyelonephritis? A. № 1B. № 2C. № 7D. № 9E. * № 5

841. Which factor most of all may be a cause of hematuria in secondary pyelonephritis?

A. increased permeability of the glomerular capillaries B. hemorrhage in the glomeruli C. ruptures of the glomerular capillaries D. renal intravascular coagulation E. * urinary tract mucosa damage by stones

842. Antibacterial therapy of pyelonephritis is given for: A. 7-14 days B. 14-21 days C. for 6 months D. to partial clinical and laboratory remission E. * to full clinical and laboratory remission

843. Which of the following antibiotics is appropriate to a child in the debut of pyelonephritis? A. kefzol B. palin C. benzylpenicillin D. erythromycin E. * augmentin

844. What is the duration of outpatient treatment (dispensarization) for children after acute pyelonephritis? A. 1 year B. 2 years C. 4 years D. 5 years E. * 3 years

845. What time after the onset of clinical and laboratory remission children with pyelonephritis can be sent to the resort? A. 6-9 months B. 9-12 months C. 1-3 months D. 1-2 years E. * 3-6 months

846. Which of physiotherapeutic methods is indicated for the pyelonephritis treatment? A. ozokerite applications on the right hypochondrium B. ozokerite applications on the suprapubic area C. electrophoresis with calcium chloride and vitamin C at the lumbar area D. novocaine electrophoresis at the lumbar area E. * furadonine electrophoresis at the lumbar area

847. The clinic of acute pyelonephritis in infants is dominated by: A. Dysuric disorders B. Pain syndrome C. Dysuria and pain syndrome D. Enuresis manifestation E. * Intoxication syndrome

848. What reserve antibiotic is used in the treatment of pyelonephritis? A. Ampicillin B. Ampiox C. Cephalosporins of 2nd generation D. Cephalosporins of third generation E. * Aminoglycosides

849. Secondary pyelonephritis develops on the background of: A. Organic hemodynamic changes B. Functional hemodynamic changesC. Organic urodynamic changes

D. Functional changes urodynamic E. * Everything is correct

850. What is “zigzag” diet in children who have an acute pyelonephritis? A. The alternation of salt-free and sugar-free days B. The alternation of KCl and protein products C. Rotation Diet № 5 and Diet № 15 D. The alternation of fasting and nutrition E. * Alternation products that are acidified or make alkaline urine

851. Latent course of chronic pyelonephritis is characterized by:A. The presence of only intoxication syndrome B. The presence of only pain C. The presence of pain and intoxication syndromes D. The presence of pain and urinary syndromes E. * The presence of only urine syndrome

852. In infants, patients with pyelonephritis, symptoms of intoxication are combined with dysfunction of: A. Respiratory system B. Cardiovascular systemC. Endocrine systemD. Nervous system E. * Digestive system

853. Chronic pyelonephritis is diagnosed when symptoms of pyelonephritis occur in a child longer than: A. 3 months B. 6 months C. 9 months D. 18 months E. * 12 months

854. What dose of aminoglycosides (gentamicin) is used for pyelonephritis treatment in children? A. 10-20 mg / kg / day B. 20-50 mg / kg / day C. 5-10 mg / kg / day D. 50-100 mg / kg / day E. * 4-6 mg / kg / day

855. What dose of amoxicillin is used for pyelonephritis treatment in children? A. 20-50 mg / kg / day B. 50-100 mg / kg / day C. 100-200 mg / Kg / day D. 150-250 mg / kg / day E. * 50-80 mg / kg / day

856. What dose of nitrofurans is used for pyelonephritis treatment in children? A. 50-100 mg / kg / day B. 1-2 mg / kg / day C. 15-20 mg / kg / day D. 20-50 mg / kg / day E. * 5-10 mg / kg / day

857. Which of the following preventive measures is the most effective when hypovitaminosis D? A. Storage of food at low temperatures B. The use of refined carbohydrates C. Use fresh vegetables and fruits D. Prevent the formation of smog E. * Sunbathing

858. In which period of rickets Spasmophylia is usually developde?

A. height B. initial C. residual D. anyone E. * convalescence

859. Rapid intravenous injection of calcium preparations may cause: A. bradycardia B. apneaC. tachycardia D. arrhythmia E. * bronchospasm

860. The immediate cause of spasmophylia is: A. hypoglycemia B. hypophosphatemia C. hypoproteinemia D. hypolipidemia E. * hypocalcemia

861. Vitamin D in case of spasmophylia is appointed after seizures through: A. 1 - 2 days B. 3 - 4 weeks C. 3 - 4 months D. everything is wrong E. * 3 - 4 days

862. What product must be limited to children with spasmophylia? A. breast milk B. vegetable puree C. cereal porridges D. eggs E. * cow’s milk

863. Doses of vitamin D in the treatment of rickets depends on: A. child’s weight B. body surface C. age child D. everything is correct E. * severity of illness

864. What is the duration of rickets treatment? A. 15 - 30 days B. 10 - 15 days C. 45 - 60 days D. 5 - 7 days E. * 30 - 45 days

865. What is the duration of the rickets initial period?A. 1 - 4 days B. 1 - 4 months C. 1 - 4 years D. 1 - 4 hou E. * 1 - 4 wks

866. What concentration of calcium gluconate is used for spasmophylia treatment ?A. 1 - 2% solution B. 0,1 - 0,2% solution C. 10 - 20% solution D. 3 - 5% solution E. * 5% - 10% solution

867. In order to eliminate alkalosis children with spasmophylia receive: A. Calcium chloride B. Sodium chloride C. Potassium chloride D. Physiologic saline E. * Ammonium chloride

868. Daily dose of vitamin D for term infants (the method of "small doses") is: A. 100 IU B. 1,000 IU C. 2,000 IU D. 5,000 IU E. * 500 IU

869. The daily dose of vitamin D3 is: A. 1,000 – 2,000 IU B. 500 – 1,000 IU C. 5,000 – 10,000 IU D. 500 – 1,000 IE. * 2,000 – 5,000 IU

870. Carpopedal spasm - is: A. hands and feet muscles clonic contractionsB. tonic contractions of facial muscles C. tonic contractions of glottis D. convulsions E. * hands and feet muscles tonic contractions

871. Acid - base balance in rickets is: A. shifted toward alkalosis B. not changed C. everything is wrong D. everything is corre E. * shifted toward acidosis

872. What is the criterion for vitamin D course completion in rickets treatment?A. increased muscle tone B. normalization of sleep C. closure of a large fontanel D. everything is correct E. * normalization of biochemical changes

873. Laboratory changes in the peak of rickets include: A. hypercalcemia B. hyperphosphatemia C. alkalosis D. reduction of alkaline phosphatase E. * increase alkaline phosphatase

874. What are laboratory criteria of spasmophylia? A. hyponatremia B. kaliopenia C. hypophosphataemia D. hypernatremia E. * hypocalcemia

875. What is the most dangerous manifestation of carpopedal spasm?A. spasm of the hands muscles B. spasm of the smooth muscles C. spasm of masticatory muscles D. spasm of respiratory muscles

E. * spasm of the heart muscle 876. The highest level of phosphorus in the blood occurs in patients with rickets:

A. during the height periodB. in the initial period C. in the residual period D. constantly E. * during convalescence

877. The initial period of rickets is mostly diagnosed to the child of: A. 2-3 months old B. 3-5 months old C. 5-7 months old D. After 1 year E. * 1 month

878. The normal level of phosphorus in the blood is observed in: A. height period of ricketsB. convalescence period of ricketsC. initial period of ricketsD. constantly E. * residual period of rickets

879. Rickets provoking factor is feeding with overload of: A. fat B. protein C. vitamins D. minerals E. * carbohydrates

880. In the initial period of rickets study of the nervous system detect: A. muscular hypertensionB. muscle atrophy C. decreasing muscular strength D. muscle pain E. * muscular hypotension

881. What is the activity of alkaline phosphatase in the initial stage of rickets? A. normal B. reduced C. invariable D. everything is wrong E. * increased

882. In rickets the deformation of the chest appears in: A. 1 -3 months B. 6 - 9 months C. 9 - 12 months D. 0 - 1 month E. * 3 - 6 months

883. In rickets the deformation of the lower extremities occurs in: A. 3 - 6 months B. 6 - 9 months C. 1 - 3 months D. 0 - 1 month E. * 9 - 12 month

884. In rickets the bones of the skull are affected in: A. 3 - 6 months B. 6 - 9 months C. 9 - 12 month

D. 0 - 1 month E. * 1 -3 month

885. What is one of the hypocalcemia mechanisms in spasmophylia?A. hyperparathyroidism B. hyperthyroidism C. hypotyroidism D. hypogonadism E. * hypoparatyroidism

886. Anticonvulsant therapy is prescribed in spasmophylia simultaneously with: A. Vitamin D B. detoxication therapy C. rehydration therapy D. antipyretics E. * calcium supplementation

887. What is Maslov sign in spasmophylia?A. rapid feet abduction when tapped below the fibular B. contraction of the facial muscles when tapped in the facial nerve areaC. fingers convulsions when shoulder compression D. everything is wrong E. * stop of breathing at a light skin pricks

888. What is Chvostek sign in spasmophylia? A. stop of breathing at a light skin pricks B. rapid feet abduction when tapped below the fibular C. fingers convulsions when shoulder compression D. everything is wrongE. * contraction of the facial muscles when tapped in the facial nerve area

889. When is done specific antenatal prevention of rickets for healthy pregnant women? A. In 24 - 28 weeks of gestationB. It is not needed C. throughout the pregnancy D. before delivery E. * In 28 - 32 weeks of gestation

890. Specific prevention of rickets for full-term children begins: A. from their birth B. on the sixth month of life C. on the tenth month of life D. on the third month of life E. * on the second month of life

891. What is the daily dose of vitamin D3 for the antenatal prevention of rickets in healthy pregnant womenA. 200 IU B. 500 IU C. 100 IU C. 2000 D. * 100IU

892. Scific prevention of rickets to preterm children begins: A. rom their birth B. on the sixth month of life C. on the tenth month of life D. on the second month of life E. * in two weeks of life

893. What peculiarity can be cause of conjunctivitis in case of upper respiratory tract infection in early age children more ofteA. crease of local immune response

B. *Nasolacrimal duct is shortC. Contagion is highD. Rubbing eyE. Wrong bwing nos

894. When sinuses evelopment in children will finish?A. Before birthB. to 1 yearC. to 3 yearsD. to 5 yearsE. *to 12 years

895. What medical term is synonym for “laryngitis”?A. *CroupB. EpiglottitisC. VocalitisD. TonsillitisE. Chondritis

896. What organs are connected by Eustachian tube?A. *the middle ear & the throatB. the middle ear & larynxC. the inner ear & the throatD. the inner ear & the middle earE. the outer ear & the middle ear

897. What is feature of the right primary bronchus?A. *Is like a direct extension of the trachea B. Look like is separated from the trachea.C. Has specific structureD. Is longer than left oneE. Is thinner than left one

898. What is feature of the left primary bronchus?A. Is like a direct extension of the trachea B. *Look like is separated from the trachea.C. Has specific structureD. Is shorter than right oneE. Is wider than right one

899. What is typical orientation of the ribs in the infant?A. A. * Horizontal B. DownwardC. UpwardD. Without typical orientation E. Puerile

900. What is typical orientation of the ribs in 10 years old children?A. Horizontal B. *DownwardC. UpwardD. Without typical orientation E. Puerile

901. Breathing: pulse ratio from birth till 1 month of life:A. *1:3B. 1:4C. 1:5D. 1:2E. 1:6

902. Breathing: pulse ratio from 8 till 14 years of life:

A. 1:3B. *1:4C. 1:5D. 1:2E. 1:6

903. What is average respiratory rate at rest of newborn? A. 25 per minuteB. *40-60 per minuteC. 16-20 per minuteD. More then 60 per minuteE. Less then 25 per minute

904. What is average respiratory rate at rest of 5-year-old child? A. *25 per minuteB. 40-60 per minuteC. 16-20 per minuteD. More then 60 per minuteE. Less then 25 per minute

905. Choose description of «Hyperpnea»:A. *Increase of the respiratory depth B. Increase of respiratory rate and deptC. Increase of the respiratory rate D. Distress during breathingE. Cessation of breathing

906. Kussmaul respiration is characterized by: A. *Slow deep breathing, hyperventilation, gasping and labored respirationB. Totally irregular breathing with no pattern C. Cyclical increase and decrease in depth of respiration D. The chest falls on inspiration and rises on expirationE. Decrease depth and irregular rhythm of respiration

907. What sounds you can determine over solid areas during percussion of the lung?A. Resonant B. Hyper-resonanceC. Tympanic D. *FlatE. Cracked-pot sound

908. Choose description of Seesaw (paradoxic) respirations:A. cyclical increase and decrease in depth of respiration B. slow deep breathing, hyperventilation, gasping and labored respiration C. *the chest falls on inspiration and rises on expiration. D. totally irregular respirations with no pattern E. an increasing of rate and depth of respiration

909. When does a hyper resonant (ban-box) sound may be determined during percussion of the lung?A. *AsthmaB. PneumoniaC. Pleural effusionsD. HaemothoraxE. Hydrothorax

910. What sounds are soft, blowing, lower pitched during auscultation of the lung?A. *VesicularB. BronchialC. TrachealD. PuerileE. Wheezes

911. What sounds are loud and high in pitch with a short pause between inspiration and expiration during auscultation of the lunA. VesicarB. *BrchialC. TrachealD. PuerileE. Wheezes

912. During auscultation of the lung louder shot inspiration and a hollow expiratory phase, blowing character is typical foA. Vesilar breathB. Bronchial breathC. Tracheal breathD. *Puerile breathE. Wheezes

913. Choose group of sounds, which can be determined over the normal lung tissue: A. *Tracheal, bronchial, broncho-vesicular and vesicular soundsB. Wheeze, bronchial, broncho-vesicular and vesicular soundsC. Tracheal, bronchial, broncho-vesicular sounds, rhonchiD. Tracheal, bronchial, broncho-vesicular sounds, crackles E. Tracheal, bronchial, rales, vesicular sounds

914. Puerile breath during auscultation of the lung is typical for:A. PneumoniaB. AtelectasisC. *Healthy children till three years oldD. Healthy children after age 3 years E. Asthma

915. Bronchial breath sound is abnormal in such places during auscultation of the lung:A. Over the large airways B. Over the manubrium of the sternumC. *Peripheral areas of the lungD. In the anterior chest wallE. In the posterior right interscapular space

916. What types of breath sounds are «adventitious» breath sounds?A. *Rales, wheezes, pleural friction rubs, stridorB. Rales, wheezes, puerile sounds, stridorC. Rales, ban-box, pleural friction rubs, stridorD. Crackles, wheezes, pleural friction rubs, vesicular soundsE. Crackles, wheezes, pleural friction rubs, tracheal sounds

917. Stridor can be indicative of:A. PneumoniaB. *Serious airway obstructionC. Bronchial asthmaD. Purulent bronchitisE. Satisfactory condition

918. Crackles are often associated with inflammation or infection of the A. *Small bronchi, bronchioles and alveoli B. Large bronchiC. Pleural surfaces D. TracheaE. Larynx

919. What is definition of Wheeze?A. Discontinuous, nonmusical, brief sounds heard more commonly on inspirationB. Low-pitched, grating, or creaking sounds

C. High-pitched harsh sound heard during inspirationD. *Continuous, high pitched, hissing, whistling or sibilant soundsE. Soft, blowing, lower pitched and softer than bronchial breathing

920. Crackles are often associated with:A. *PneumoniaB. BronchitisC. TracheitisD. LaryngitisE. Pleurisy

921. Wheezes are often associated with:A. PneumoniaB. *BronchitisC. TracheitisD. LaryngitisE. Pleurisy

922. Dyspnea with long wheezing expiration, skin pallor with cyanotic shade, strain of neck muscles are typical foA. ConvulsiveB. *AsthmaticC. LaryngospasmD. HyperthermicE. Comatose

923. Which information is most important in rheumatic fever?A. A fever that started 3 days agoB. Lack of interest in foodC. *A recent episode of pharyngitisD. Vomiting for 2 daysE. Artralgia 1 month ago

924. Urinalysis includes following information EXEPT:A. Colour of urineB. Specific gravity C. Level of protein, glucoseD. Quantity of WBC and RBCE. *Daily urine volume

925. How to collect urine for urinalysis:A. Collect the morning middle portion of urine, after careful washingB. Collect all urine during nightC. Collect urine during 24 hours (8 portions)D. Collect the middle portion of urine in any time of the day, after careful washingE. *Collect all morning urine after careful washing

926. How to collect urine for Zimnitsky’s test?A. Collect the morning urine, middle portion, after careful washingB. Collect all urine during nightC. *Collect urine during 24 hours (8 portions)D. Collect the middle portion of urine in any time of the day, after careful washingE. Collect all morning urine after careful washing

927. Mild proteinuria is typical for all diseases, except:A. CystitisB. UrethritisC. VulvovaginitisD. *Glomerulonephritis with nephrotic syndromeE. Pyelopephritis

928. Nechyporenko urine test is used for evaluation of the cells in:

A. 0.1 ml of urineB. *1 ml of urineC. 5 ml of urineD. 10 ml of urineE. 1 L of urine

929. Nephrotic syndrome is characterized by (choose the most correct statement)\:{A. Heavy proteinuria, hematuria, hypoproteinemiaB. *Hypertension, hematurC. Hypoalbuminemia, pteinuria, edemaD. Pain in lumbar regn, intoxicationE. Isolated hematuri or pyuria, proteinuria

930. What laboratory sign is the most typical for pyelonephritis?A. * active leucocytes in urine;B. considerable proteinuria; C. uraturia;D. oxalaturia; E. glucosuria.

931. Significant proteinuria is the most typical for:A. CystitisB. UretritisC. VulvovaginitisD. * GlomerulonephritisE. Pyelopephritis

932. Renal function can be indicated by all tests, EXCEPT:A. Glomerular filtration rateB. Plasma Creatinine level C. Plasma Urea level D. Zimnitsky’s testE. * Nechyporenko test

933. The most common bacteria in acute bacterial pyelonephritis is:A. KlebsiellaB. Chlamydia C. * E. ColiD. PseudomonasE. Candida

934. The most diagnostic value of excretory urography is in case of:A. Glomerulonephritis B. Amiloidosis C. * Pyelonephritis D. Nephrotic syndromeE. Renal tumour

935. What day / night diuresis ratio is normal?A. 1\:1 B. * 2\:1 C. 1\:2 D. 4\:5 E. 6\:1

936. Urinary syndrome indicates maohematuriya, protein 1.6 g / liter. Which the disease is characterized by such changesA. *Glomerulonephritis B. isolated urinary syndrome C. pyelonephritis D. cystitis

E. urolithiasis955. Positive Erb, Trousseau, Maslov symptoms are typical for

A. Renal eclampsia B. Hyperthermia C. Epilepsy D. Meningoencephalitis E. * Spasmophylia

956. After the emergency treatment of the child with spasmophylia further therapeutic tactics is: A. Assign vitamin D in doses of 2000 IU a week supplementation with calcium. B. Assign vitamin D in doses of 4000 IU once. C. Do not assign vitamin D D. Assign vitamin D in doses of 500 IU immediately E. * Assign vitamin D in doss of 500 IU a week supplementation with calcium.

957. Soft edge of a large fontanel, softening of the occipital bone in baby with rickets are typical for such course of the disease:A. Subacute B. Recurrent C. Latent D. Limp E. * Acute

958. Increase the frontal and parietal tuber, thickened edges of large fontanel, ribs "rosary" are typical for such course of the rickets: A. Acute B. Recurrent C. Latent D. Limp E. * Subacute

959. Increase of frontal and parietal tubers, ribs "rosary", "bracelets", "string of pearls", thickening of the large fontanelle edges, increased liver, Harrison's groove. What course of rickets in this child? A. Acute B. Recurrent C. Latent D. Limp E. * Subacute

960. Which clinical symptoms indicate the moderate severity of rickets? A. Harrison’s groove B. The predominance of osteoid hyperplasiaC. The predominance of osteomalacia D. The age of 7 months E. * The presence of bone changes simultaneously on the head and trunk.

961. How it is possible to confirm the diagnosis of a spasmophilia?A. To do Sulkovitch’s testB. To determine a level of potassium in a bloodC. To determine a level of magnesium in a bloodD. To click the radical of tongueE. * To test Hvostek, Lust signs

962. What changes of muscular system will be present in initial stage of a rickets?A. Hypertonia of musclesB. Muscle atrophyC. Decrease of muscles forcesD. Increase of muscles forcesE. * Hypotonia of muscles

963. What medicine must be prescribed for child with gastroenteral enzymopathy?

A. Antibacterial therapyB. ImmunostimulatorsC. SorbentsD. MultivitaminsE. * Enzymes and bacterial drugs

964. What from this is better to prescribe the child to correct dysbacteriosis?A. Festal B. Apilac C. Ampicillin D. NistatinE. * Bifidumbacterin

965. Water deficiency dehydration is characterized by such dates of laboratory examinations:A. Decrease of hematocritB. Decrease of K, Na, Cl level in the blood, considerable increase of hematocritC. Decrease of K, Na, Cl level in the bloodD. Detection of a bacterial infectionE. * Increase of potassium level

966. What investigation will confirm the Salmonellosis?A. General blood test.B. Bacteriological examination of cerebrospinal fluid.C. Bacteriological investigation of nasal swab.D. Bacteriological investigation of pharyngeal swab.E. * Bacteriological examination of the feces on dysentery, typhoid, paratyphoid fever.

967. What duration of the food tolerance determination period in Malnutrition 1st degree?A. 1-2 weeks B. 2-3 weeks C. 7-14 days D. 3-4 weeksE. * 1-3 days

968. How to correct the deficit of the protein?A. By the Porridge B. By the Vegetable puree C. By the Yoghurt D. By the Fruit juice E. * By the pot cheese

969. Choose a physiometric method of investigation from the below given:A. Determination of thorax formB. Determination of vertebra formC. Determination of body weightD. Measurement of growthE. * Determination of vital capacity of lungs

970. What diagnostic method can determine pneumopathy's type in the child?A. Immunologic investigationB. Blood gasesC. ProteinogramD. Blood testE. * Chest X-ray

971. What drugs will be used as a base therapy for child with obstructive bronchitis?A. AntibioticsB. Antiallergic medicineC. MucolyticsD. HormonsE. * Broncholytics

972. Rough barking cough, hoarseness of the voice, expressed inspiratory dyspnea, involvement of the auxiliary musculature in breathing, skin pallor, tachycardia describe:A. Pneumonia B. Bronchiolitis C. Obstructive bronchitis D. PharyngitisE. * Croup syndrome

973. What is single dose of panadol?A. 0.05 g/kg B. 0.1 g/kg C. 0.2 g/kg D. 0.3 g/kgE. * 0.01 g/kg

974. On X-ray: lungs’ roots are broad, infiltrated, and in both sides are little shadows. What form of an acute pneumonia corresponds with this X-ray?A. InterstitialB. MonosegmentalC. PolysegmentalD. CruposeE. * Bronchopneumonia

975. What bacteria cause the interstitial pneumonia more often?A. StaphylococcusB. StreptococcusC. PneumocystaD. PneumococcusE. * Klebsiella

976. What auscultation picture is typical for focal bronchopneumonia in child?A. Diffuse dry ralesB. Rough breathingC. Diffuse wet ralesD. Decrease breathingE. * Local wet rales

977. What medication should be applied for emergency aid in case of Quincke's edema? A. Adrenalin B. Furosemide C. Heparin D. Seduxen E. * Prednisolone

978. What changes in the blood analysis give us the possibility to prove allergic etiology of disorders?A. AnemiaB. LymphocytosisC. NeutrophylosisD. ErythremiaE. * Eosynophylia

979. What from parameter will help to confirm the rheumatoid arthritis most faithfully? A. Hyperazotemia B. Presence of LE-cells in blood C. Hypergammaglobulinemia D. Thrombocytopenia E. * C-reactive protein

980. What indicator is connected with possible etiology of the rheumatic process?A. SeromucoidB. Creatinkinase

C. 1-antitrypsineD. Rheumatic factorE. * Antistreptolysine-0

981. What medication is the most expedient for secondary prevention of rheumatic fever?A. ErythromycinB. Bicillin-1C. AmpicillinD. OxacillinE. * Bicillin-5

982. What complication is the most possible to reveal in rheumatoid arthritis, monoarticular form?A. Stomatitis B. Pulmonitis C. Nephritis D. CarditisE. * Uveitis

983. What etiologic factor caused rheumatic arthritis?A. VirusB. StaphylococcusC. PneumococcusD. FungusE. * Beta hemolytic streptococcus

984. What is most reasonable to prescribe for long-term therapy of child with rheumatoid arthritis, polyarticular form ?A. Aspirin B. AzulfadineC. Delagyl D. Prednisolon pulse-therapyE. * Methothrexat

985. What dose of prednisolone must be used at the beginning of treatment of child with acute norheumatic carditis?A. 2 mg\kg\dayB. 3 mg\kg\dayC. 1 mg\kg\dayD. 5 mg\kg\dayE. * 0,5 mg\kg\day

986. What method of examination is the most informative in case of dilated heart boarders to the left side, aortic systolic murmur?A. SphygmographyB. PhonocardiographyC. X-rayD. CoronarographyE. * Echocardiography

987. What is the most efficient examination for valvular disorder assessment? A. Ballistocardiogram B. Chest X-ray C. Phonocardiography D. ECG E. * Echocardiography + Doppler-Echocardiography

988. Cardiomegaly, steadfast tachyarrythmia, stagnant rales in both lungs, dyspnea, increased liver to 3 cm were revealed in child with early congenital carditis. What stage of the cardiac insufficiency has this child?A. I stageB. II A

C. IIID. IVE. E. * II B

989. Irregularity of development of the upper and lower parts of body, hypotonia of muscles of the feet, an absent pulsation on the femoral arteries, systolic murmur in intrascapular region were revealed. What pathology may be suspected in the patient? A. Kawasaki disease B. Takajasu disease C. Aneurysm of aorta D. Nothing of theseE. * Coartation of the aorta

990. The patient with aquired heart failure has diastolic pressure 0 mm Hg. What heart failure does the child have?A. Rheumatic carditisB. Aortic stenosisC. Mitral insufficiency D. Mitral stenosis E. * Aortic insufficiency

991. What is the most probable leading mechanism of ulcer disease development? A. Dietary allergy B. Reduced prostaglandin synthesis C. Disorder of gastric motor activity D. Autoantibody production E. * Helicobacterial infection

992. What is the most effective medication to treat ulcer defect of the mucous membrane of the duodenum in child?A. AlmagelB. PapaverinC. No-spaD. AtropinE. * De-nol

993. What method of investigation will be the most useful for proof of ulcer disease?A. Ultrasound examination of abdomen B. pH-metryC. Ureatic testD. Stomach X-rayE. * Esophagogastroduodenoscopy with biopsy

994. What is the etiology of chronic antral gastritis?A. St. Aureus B. beta-hemolytic streptococcus of group AC. Candida D. Enterovirus E. * Неlicobacter pylori

995. What is the main diagnostic sign of functional disturbance of the stomach? A. Abdominal pain B. Regurgitation by air C. Signs of chronic intoxication D. Decrease of the appetite E. * Absence of the organic changes during esophagogastroduodenoscopy

996. Choose a drug for correction of the motility in case of duodenogastral reflux ІІ degree. A. Imodium B. Gastrocepin C. Halidor

D. Dicitel E. * Motilium

997. What drugs should be assigned for treatment of hypertonic biliary dyskinesia first of all? A. Sedative and cholikineticsB. Choleretics and cholikineticsC. Antioxidants D. Antibiotics E. * Spasmolitics and choleretics

998. What drugs should be assigned for treatment of hypotonic biliary dyskinesia?A. Analgetics B. Choleretics C. CholekineticsD. Spasmolitics E. * Choleretics and cholikinetics

999. Periodic short-lived cutting pain in the right subcostal area, which occurs after the greasy food, is typical for what type of biliary dyskinesia?A. HypotonicB. DystonicC. HepatalgicD. AsthenicE. * Hypertonic

1000. Edema, arterial hypertension, hematuria, proteinuria (1,8 g/per day), granular and erythrocital casts are typical for:A. PyelonephritisB. CystitisC. Intestinal nephritisD. Renal amyloidosisE. * Glomerulonephritis

1001. Fever, frequent painful urination, changes in urinalysis (proteinuria [0,066 g/L], leukocytouria [entirely within eyeshot]),acteriuria [105 colony forming units/mL] are typical for:A. Acute glomerulonephritisB. Acute cystitisC. Dysmetabolic nephropathyD. UrolithiasisE. * Acute pyelonephritis

1002. Which of the following is the most appropriate test for prescribing of etiotropic treatment of client with pielonephritis?A. Nechiporenko test B. Zimnitsky tesC. CystographyD. Intravenous urographyE. * Urine culture

1003. What biochemical index is of the greatest diagnostic importance in case of acute glomerulonephritis?A. Uric acid B. Fibrinogen C. Blood sodium D. Blood bilirubin E. * Blood creatinine

1004. Mild generalized edema, increased blood pressure, and the urinalysis showed increased protein, red cell casts and hyaline casts are typical for:A. Acute pyelonephritis B. Rheumatic fever

C. Essential hypertension D. Bacterial endocarditis E. * Acute glomerulonephritis

1005. What nutrition recommendations are the most suitable for the patient with chronic renal failure first degree?A. Ingestion with the increased content of the "alkaline"B. Fluid amount increaseC. Adipose controlD. Carbohydrate controlE. * Protein control

1006. Decrease of urine volume (200 mL per day), peripheral and cavity edema were revealed. Urinanalysis: protein 3,6 g/L. What is the most likely diagnosis?A. Interstitial nephritisB. Infection of urinary tractC. Chronic glomerulonephritisD. Acute glomerulonephritis with nephritic syndromeE. * Acute glomerulonephritis with nephrotic syndrome

1007. Proteinuria - 7.1 g/L, protein in daily urine - 4.2 g. Blood biochemical profile: stable hypoproteinaemia (43.2 g/L), hypercholesterinaemia (9.2 mmol/L). Which variant of glomerulonephritis is the most probable in this case?A. Isolated urinaryB. HematuricC. MixedD. NephriticE. * Nephrotic

1008. What is the urine color of the patient with acute poststreptococcal glomerulonephritis? A. dark brown urineB. bright redC. dark violetD. pinkE. * smokey brown urine

1009. What specific changes in the blood will be present in patients with chronic glomerulonephritis?A. increases bilirubinB. decline of ESRC. increases cholesterolD. increases transaminasesE. * increases kreatinin

1010. In the complete analysis of urine: dark color of urine, protein is 0,98 g/l, leucocytes 3-2 in a visual, erythrocytes – 1/3 of visual field, hyaline casts 1-2 in a visual field are observed. What variant of acute glomerulonephritis is more reliable in this child? A. NephroticB. NephriticC. Nephrotic syndrome, hematuria and arterial hypertensionD. Sub acute malignant glomerulonephritis E. * Isolated urine syndrome

1011. Appearance of a lot of changed red blood cells in the urine (1/2 of visual field) can be caused of: A. Uncomplicated urolithiasisB. acute cystitisC. paranephritisD. cancer of urinary bladderE. * acute glomerulonephritis

1012. What etiological factor is cause of rheumatic carditis?A. Staphylococcus

B. PneumococcusC. VirusD. FungusE. * Beta-hemolytic streptococcus

1013. Results of urinalysis: urine is brown, specific gravity -1025, protein - 1,2 g/l, erythrocytes cover all visual field, casts 1-2 in a visual field. Protein in day's urine - 0,78 gr. Such lab changes are typical for:A. Acute glomerulonephritis with nephrotic syndromeB. UrolythiasisC. Acute glomerulonephritis with nephrotic syndrome, hematuria and arterial hypertensionD. Acute glomerulonephritis with isolated urine syndromeE. * Acute glomerulonephritis with nephritic syndrome

1014. In the patient’s urine sediment 5-6 leucocytes and single fresh erythrocytes in a vision field were found. What investigation must be appointed to a patient for clarification of diagnosis?A. total blood countB. ECGC. Zimnitsky testD. determination of daily proteinuriaE. * Nechyporenko test

1015. Appearance of what substance in urine can be cause of cloudy urine which doesn’t disappear in an hour? A. saltsB. bilious pigmentsC. glucoseD. urinary acidE. * protei

1016. Appearance of what substance in urine can be cause of cloudy urine which disappears in an hour? A. proteinB. bilious pigmentsC. glucoseD. urinary acidE. * salts

1017. Appearance of what substance in urine makes its gravity increased? A. saltsB. proteinC. bilious pigmentsD. urinary acidE. * glucose

1020. What is the peculiarity of edema in patient with acute glomerulonephritis? A. appears in the eveningB. first appears on lower extremitiesC. first appears on overhead extremitiesD. Early development of anasarca.E. * appears on face in the morning

1021. Complains of edema in the morning which is located mainly on her face (eyeleads) can be present in case of: A. cardiovascular disorderB. Neurological pathologyC. Respiratory system pathologyD. Digestive diseasesE. * Urinary system pathology

1022. What urine color is typical for glomerulonephritis? A. red

B. color of beerC. sulphurD. straw-yellow.E. * color of «meat wastes»

1023. What investigation must be done if patient complains of edema below the eyes in the morning? A. ECGB. Ultrasound of a heartC. determination of cholesterol in bloodD. Chest X-ray.E. * general analysis of urine

1024. What pathology is characterized by nocturia? A. acute nephritisB. diabetes mellitusC. chronic cardiac insufficiencyD. diencephalic syndrome.E. * chronic kidney insufficiency

1025. Complains on pain in suprapubic area, frequent painful urination by small portions, subfebril temperature, negative Pasternatsky symptom are typical for:A. Dysmetabolic nephropathyB. Acute pyelonephritisC. UrolithiasisD. Acute glomerulonephritisE. * Acute cystitis

1026. What changes in biochemical blood test can be present in patient with renal failure? A. increase of glucose level in bloodB. increase of bilirubinC. increase of amylazeD. increase of alkaline phosphatase.E. * increase of creatinine

1027. What changes in biochemical blood test will prove kidney insufficiency? A. albuminemiaB. beta-lipoproteinaemiaC. hyperbilirubinemiaD. dysproteinemia.E. * creatininemia

1028. What investigation is it necessary to do to know the level of urinary system damaging? A. To take urine by catheterB. Bacteriological test of urine C. Ultrasound investigation D. Zimnitsky testE. * Urographia

1029. What investigation is it necessary to do before etiotropic treatment? A. CystographiaB. UrogrphiaC. Zimnitsky testD. Nechiporenko testE. * Bacteriological test of urine

1030. What dates are typical to diagnose urinary tract infection?A. Proteinuria B. LeucocyturiaC. ErytrocyturiaD. HypostenuriaE. * Bacteruria 105 and more

1031. What investigation will prove the genesis of leucocyturia? A. UrographyB. Nechiporenko testC. Endogenous creatynine clearanceD. Zimnitsky testE. * 3 glasses test

1032. A pain syndrome at ulcer of duodenum is not characterized by: A. "hungry" pain;B. nightly pain;C. late pain;D. *pain immediately after meal;E. pain after the physical loading.

1033. Antibacterial therapy for peptic ulcer disease requires the use of: A. Analgesics B. Antipyretics C. *Probiotics D. Hormones E. Cytostatics

1034. In case of gastritis abdominal pain is increased:A. * 15-30 min. after a mealB. 30-45 min. after a mealC. 45-60 min. after a mealD. 1-1,5 hours. after a mealE. 2-2,5 hours. after a meal

1035. Constipation is characteristic for gastritis in patients with:A. * increased acid productionB. normal acid productionC. decreased acid productionD. achlorhydriaE. hypohlorhydria

1036. Dyspeptic syndrome includes next symptoms:A. increased levels of total and conjugated bilirubin and cholesterol;B. increased levels of ASAT, ALAT, LDG;C. increased levels of amylase, tripsin, lipase;D. * vomiting, nausea, heartburn feeling, decrease of appetite;E. weakness, lucidity, bad sleep, headaches, irritability, tearfulness, increased disposition to

perspiration, blue shadows under the eye1037. Famotidin belongs to a group of

A. antacids B. * histamine H2-blockers C. proton pump blockers, D. reparants E. cytoprotectors

1038. Functional gastric disorders include: A. Chronic esophagitis. B. Chronic gastritis. C. * Duodenogastric reflux. D. Stomach ulcer. E. Chronic duodenitis.

1039. Gastroesophageal reflux belongs to: A. Secretory type functional disorders of the stomach. B. * Motor type functional disorders of the stomach. C. Chronic gastritis.

D. Biliary dyskinesia. E. Chronic duodenitis.

1040. Helicobacter pylori produces an enzyme: A. lactaseB. * ureaseC. peptidaseD. lipaseE. amylase

1041. Untreated Helicobacter pylori can lead to:A. esophagitisB. * gastritisC. colitisD. hepatitisE. uretritis

1042. In case of disease associated with increased gastric acid secretion more prominent is:A. * pain,B. dyspepsia,C. intoxicationD. disuriaE. hemorrhagic syndrome

1043. What medication is used for treatment of gastric ulcer?A. Analgene B. * De-nol C. Essentiale D. Prednisolone E. Aspirin

1044. Intoxication syndrome includes next symptoms:A. increased levels of total and conjugated bilirubin and cholesterol;B. increased levels of ASAT, ALAT, LDG;C. increased levels of amylase, tripsin, lipase;D. vomiting, nausea, heartburn feeling, decrease of appetite;E. * weakness, lucidity, bad sleep, headaches, irritability, tearfulness, increased disposition

to perspiration, blue shadows under the eyes, functional heart murmu1045. Belching disorders includes:

A. *AerophagiaB. Functional vomitingC. Irritable Bowel SyndromeD. Functional BloatingE. Functional Constipation

1046. Functional Bowel Disorders includes:A. AerophagiaB. Functional vomitingC. *Functional BloatingD. Functional dyspepsiaE. Functional Constipation

1047. Meteorism is characteristic sign for gastritis with:A. increased gastric acid secretionB. normal gastric acid secretioC. * decreased gastric acid secretionD. achlorhydriaE. hypohlorhydria

1048. What are aggressive factors of the gastric mucosaA. * HCl

B. mucus productionC. prostaglandin -Х2D. bicarbonatesE. Na Cl

1049. What are gastric mucosal defensive factors?A. pepsinB. HClC. * bicarbonatesD. gastrinE. Na C

1050. Empty stomach pain is typical for patients with:A. * increased gastric acid secretionB. normal gastric acid secretionC. decreased gastric acid secretionD. achlorhydriaE. hypohlorhydria

1051. Ranitidine is used to treat conditions related to the stomach:A. normoacidity B. * hyperacidity C. hypoacidity D. concomitant duodeno-gastric reflux E. achalasia of the esophagus

1052. Vomiting bile is usually a sign of disorder of:A. esophagusB. stomach C. * duodenum D. colon E. small intestinum

1053. How many components are included into Eradication Treatment of H. pylori-associated gastroduodenal disease in children?A. one component;B. two components;C. * three components;D. five components;E. a lot of components.

1054. The X-ray sign of gastric ulcer is:A. radial convergence of stomach walls;B. bulge of walls;C. bulge of mucus membraine folds;D. * presence of "Ulcer crater";E. decrease of mucus membraine folds.

1055. What medication does belong to histamine H2-blockers? A. smecta B. almagel C. acidophilus D. * famotidineE. motilium

1056. What medication does belong to Proton pump inhibitors (PPIs)?A. smecta B. almagel C. ranitidine D. * omeprazole E. motilium

1057. What medication does belong to probiotics?A. Aevit B. SmectaC. *Acidophilus D. Procainamide E. Saline

1058. What medication does belong to cytoprotectors?A. de-nol B. omeprazole C. * sukralfat D. famotidine E. maalox

1059. What medication does belong to reparants?A. * gastrofarm;B. panzynorm;C. renegastD. bellaspon;E. ranitidin.

1060. What is invasive method of helicobacteriosis diagnostics?A. * biopsy of the stomach mucosa with its bacteriological investigation;B. PCR of feces and saliva;C. Breath TestD. determination of specific immuneglobulinesE. "aerotest".

1061. What instrumental methods is most considerable for the diagnostics of chronic gastroduodenitis?A. * endoscopy;B. aerometry;C. fractional research of the gastric content;D. acid-test;E. X-ray.

1062. What clinico-pharmacological group includes De-nol? A. *antiulcer drug ;B. reparants;C. probiotics;D. proton pump inhibitors;E. histamine H2-blockers.

1063. What is gastroesophageal reflux? A. This is involuntary leakage of intestinal contents into the stomachB. * This is involuntary leakage of gastric contents into the esophagus C. This is involuntary leakage of gastric contents into the oral cavity D. This is pylorus insufficiencE. This is stomach fundus insufficiency

1064. Epigastric pain is typical for:A. *gastritisB. cholangitisC. cholecystitisD. biliary dyskinesiE. colitis

1065. Causes of right upper quadrant pain include: A. pyelonephritis B. paraesophageal hiatal hernia C. gastritisD. *cholecystitis

E. appendicitis1066. Bifidobacteria in the child body contribute to:

A. Stabilization and normalization of intestinal mikrobiocinoseB. Improve processes of absorption and hydrolysis of fatC. Improve processes of absorption and hydrolysis of carbohydratesD. Normalization of protein and mineral metabolismE. * The maintenance of nonspecific resistance

1067. Celiac disease is associated with:A. Infection enterocolitisB. DysbiosisC. Family predispositionD. * MalabsorbtionE. Functional constipation

1068. Crohn's disease is:A. Infection diseaseB. Congenital diseaseC. Hereditary diseaseD. * Autoimmune diseseE. Metabolic disease

1069. Ulcer disease can be suspected by:A. Periodic attacks of pain in the epigastriaB. Long persistent dyspeptic disordersC. The high acid-forming function of the stomachD. * Fecal occult blood testE. Vomiting

1070. Signs of dyspepsia syndrome are: A. Stomach painB. HeadacheC. General weaknessD. * NauseaE. Abdomen pain

1071. What symptom is not typical for irritable bowel syndrome?A. Abdominal pain or discomfortB. Intestinal bloatingC. Irregular bowel habits, including diarrhea, constipation, or bothD. * Blood in the stoolE. Mucus in the stool

1072. The diagnostic of functional abdominal pain is based on:A. Stool testB. Blood testC. * Symptoms and physical examinationD. EndoscopyE. Bacteriological examination of feces

1073. The diagnostic of irritable bowel syndrome is based on:A. * Clinical symptomsB. ColonoscopyC. DuodenoscopyD. RectoroscopyE. Bacteriological examination of feces

1074. What diet is recommended for children with functional constipation?A. * High in protein and fiberB. High in carbohydrates and fiberC. High in protein and carbohydrates

D. High in carbohydrates and fat E. High in protein and fat

1075. What is typical for functional abdominal pain? A. * absence of influence on the child’s physical and psychological developmentB. The reason is abnormality of intestineC. The reason is abnormality of bowelsD. Is due to mechanical disturbancesE. Is due to dysbiosis

1076. What disorder can be cause of constipation in babies?A. Breast feedingB. * RicketsC. Big amount of juiceD. Vegetables in dietE. Lactose intolerance

1077. What is the main treatment of celiac disease?A. EnzymesB. VitaminsC. LactobacteriaD. BifidobacteriE. * Gluten free diet

1078. What can be used for treatment of irritable bowel syndrome with constipation?A. PsychotherapyB. * A diet with dietary fibers C. CoordinaxD. Duphalac (lactuloseE. No-spani

1079. What medication is used for treatment of irritable bowel syndrome with diarrhea?A. * ImodiumB. SmectaC. MaaloxD. Duphalac (lactulose)E. Psychotherapy

1080. What products is gluten free?A. Malted milkB. Barley, oats, ryeC. Creamed vegetablesD. * FruitsE. Sauces

1081. What serologic markers are used for celiac disease screening?A. * Serum Ig AB. Serum Ig GC. Serum Ig ED. Serum Ig ME. Testing for gliadin antibodies

1082. Diagnostic test for detection of localization of upper gastrointestinal bleedingA. *EndoscopyB. Upper GI seriesC. Hemoglobin (Hb) levels and hematocrit (HCT)D. ArteriographyE. Ultrasound

1083. Primary diagnostic indicator for pancreatitis:A. Elevated blood urea nitrogen (BUN)B. *Elevated serum lipas

C. Elevated aspartate aminotransferase (AST)D. Increased lactate dehydrogenase (LD)E. Elevated bilirubin

1084. Feature of functional abdomen pain:A. *Normal exam and no significant weight lossB. Stool occult blood positiveC. Abnormal laboratory screenD. Stressors doesn’t exacerbate painE. Significant weight loss

1085. Common cause of gastric ulceration:A. Cryptosporidium B. *Helicobacter pylori C. Cytomegalovirus D. Treponema pallidumE. Herpes simplex virus-type

1086. What is the most effective means in treatment of autoimmune hepatitis?A. *Glucocorticoids, cytostaticsB. HepatoprotectorsC. Antibacterial medicationD. Hemosorbtion, vitamin therapyE. Antiviral medications

1087. Patient with skin itching, jaundice, discomfort in the right subcostal area. In blood: alkaline phosphatase — 2,0 mmol/hour/L, general bilirubin — 60 mkmol/L, cholesterol — 8,0 mmol/L. What is the leading syndrome in this patient?A. CytolytB. AsthecC. Mesenchymal inflammatoryD. Liver-cells insufficiencyE. *Cholestatic

1088. What method of investigation is to be used on the first stage of examining the patient with sudden acute pain in the right epigastric area after having fatty food?A. *UltrasonicB. RadionuclideC. Magnetic-resonanceD. RoentgenologicalE. Thermographic

1089. Which laboratory tests accurately characterizes the degree of cytolysis in a patient?A. ProthrombinB. Veltman’s testC. Takata-Ara’s testD. *TransaminasesE. Total protein

1090. Which laboratory tests should be carried out to confirm the diagnosis “pancreatitis”?A. Serum TransaminasesB. *Serum AmylaseC. Beta-lipoproteinsD. Blood glucoseE. The level of total bilirubin

1091. What coprologic syndrome is characteristic for pancreatitis?A. Little amount of mushy stool; creatorea, mucus, leukocytesB. *Many oily stool with putrid smell; creatorea, stearrhea, amiloreaC. Stool with mucus, pus, blood, leukocytes, erythrocytes, cylindrical epitheliumD. Foam feces with sour smell, amilorea, many acidophilic flora

E. Big amount of liquid stool without pathological inclusions1092. Resonant percussion sound is characteristic for:

A. *Healthy childrenB. NewbornsC. 6 month children.D. 5 years children.E. Only adults.

1093. Puerile respiration is auscultated in children in the age:A. Only before 6 months oldB. Only before one year old C. *From 1 months till 5 years oldD. Before 3 years oldE. After 5 years old

1094. At what age respiratory movements are diaphragmatic?A. Under 15 yearsB. At 1-2 yearC. *Under 6-7 yearsD. Under 12-13 yearsE. Under 1st months

1095. Right lung is divided into:A. Upper and lower (superior and inferior)B. Front and back (anterior and posterior)C. *Upper, middle and lower (superior, middle and inferior)D. Upper (superior) and middleE. Front, middle and back (anterior, middle and posterior)

1096. What is dyspnea?A. The increase of the respiratory rateB. *The distress during breathingC. The decrease of the respiratory rateD. The cessation of breathingE. The increase of the respiratory depth

1097. What is hyperpnea?A. The increase of the respiratory rateB. The distress during breathingC. The decrease of the respiratory rateD. The cessation of breathingE. *The increase of the respiratory depth

1098. What type of respiratory movements is in elder 7 years-old girl?A. *ThoracicB. AbdominalC. CostalD. SternlyE. Diaphragmatic

1099. Left lung is divided into:A. Front and back (anterior and posterior)B. Upper, middle and lower (superior, middle and inferior)C. *Upper and lower (superior and inferior)D. Upper (superior) and middleE. Front, middle and back (anterior, middle and posterior)

1100. What is tachypnea?A. *The increase of the respiratory rateB. The distress during breathingC. The decrease of the respiratory rate

D. The cessation of breathingE. The increase of the respiratory depth

1101. Average respiratory rate for 12 years old children:A. 20 per minuteB. 30 per minuteC. *16-20 per minuteD. 35-40 per minuteE. 30-35 per minute

1102. What is bradypnea?A. The increase of the respiratory rateB. The distress during breathingC. *The decrease of the respiratory rateD. The cessation of breathingE. The decrease of the respiratory depth

1103. What is usual ratio of breaths to heartbeats?A. 1:1B. 1:2C. 1:3D. *1:4E. 1:

1104. What is hypoventilationA. The decrease of the respiratory rate and irregular rhythmB. The distress during breathingC. The increase of the respiratory depth and irregular rhythmD. The cessation of breathingE. *The decrease of the respiratory depth and irregular rhythm

1105. Average respiratory rate for newborn:A. 20 per minuteB. 30 per minuteC. 16-20 per minuteD. *35-40 per minuteE. 30-35 per minute

1106. What is hyperventilation?A. The increase of the respiratory rate and irregular rhythmB. The distress during breathingC. *The increase of the respiratory rate and depthD. The cessation of breathingE. The decrease of the respiratory depth and irregular rhythm

1107. The tympanic resonance is determined over the lungs in case of:A. Pulmonary edemaB. TumorsC. Bifurcation lymphadenitisD. *Obstruction of a major bronchuE. Adiposity

1108. The pathological dullness during lung percussion is heard in case of:A. Asthmatic bronchitisB. Abscess of lungC. *Large infiltration at pneumoniaD. PneumothoraE. Emphysema of lung

1109. Average respiratory rate for 5 years old children:A. 20 per minuteB. 30 per minute

C. 16-20 per minuteD. 35-40 per minuteE. *25 per minute

1110. What is respiratory rhythm in newborns?A. *Arrhythmic breathingB. The distress during breathingC. The reduction of the BR by 10% and moreD. The cessation of breathingE. The decrease of the respiratory depth

1111. Expiratory dyspnoea develops in case of:A. Respiratory failure of the third degreeB. Diabetic comaC. Viral croup syndromeD. Foreign body aspirationE. *Bronchial asthma

1112. Barking cough is typical for…A. Dry pleurisy B. Pleurisy with effusion C. *LaryngitisD. Pneumothorax E. Tuberculosis

1113. Character of cough is can’t be: A. Dry B. Wet C. *Pituitary D. BitonalE. Spastic

1114. Crepitation is characterized by:A. Appears when you press chest by phonendoscope B. Is determined by percussion C. *Does not change when bending the body D. Is determined by palpation E. Depends on the density of attachment to the chest wall by phonendoscope

1115. Crepitation is heard…A. During inspiration and expiration B. *In 1st phase of inspiration C. In 1st phase of expiration D. In last phase of inspiration E. In last phase of expiration

1116. Crepitation is the symptom ofA. *Croupous pneumonia B. Acute bronchitis C. Dry pleurisy D. Chronic bronchitis E. Pulmonary emphysema

1117. During inspection of a patient with severe respiratory failure you may observe the following skin colorA. Pale skinB. Hyperemia C. *Diffuse cyanosis D. Yellow color E. Spider angiomata

1118. Typical changes of skin in patient with respiratory failure:

A. Pink skinB. Hyperemia C. *Cyanosis D. IcterusE. Grey color

1119. Harsh breathing indicates on…A. *Bronchitis B. Dry pleurisy C. Pleurisy with effusion D. Pulmonary emphysema E. Pneumonia

1120. Pulmonary root not included A. Large bronchi B. Receptacles C. Trachea-bronchial lymph nodes D. Broncho-pulmonary lymph nodes E. *Thymus gland

1121. In what disease crepitation are diffuse in nature? A. *Acute bronchiolitis B. Pneumonia C. Bronhoektatychna disease D. Local pulmonary fibrosis tissue E. Chronic bronchiolitis with obliteration

1122. In what disease crepitation are local in nature? A. *pneumonia B. Bronchitis C. Bronchial asthma D. Alveolitis E. Acute bronchiolitis

1123. Leading role in the diagnosis of respiratory tract foreign body is A. *Endoscopy airways B. Overview C. Percussion and auscultation D. Radiography of the chest E. Stripped tomography

1124. First aid during nose bleeding:A. To ask a child to blow his nose;B. *To put a tampon with 3% Hydrogen peroxide into the nose and ice on the bridge of the

nose;C. Oxygen therapy;D. To bring into the nose a tampon with Calcium chloride;E. Non of above

1125. What examinations do we use in the case of bronchitis?A. Complete blood countB. Culture of sputum,C. Culture of alveolar fluid,D. Biochemical examination of the blood (hyponatremia, hypokalemia)E. *Chest X-ray.

1126. What child infectious disease is characterized by attacks of spasmodic cough accompanied reprises? A. *Pertussis B. Measles C. Diphtheria

D. Scarlatina E. Red rash

1127. The most informative method for diagnose pneumonia is: A. * Radiography B. Tomography C. Bronchography D. Bronchoscopy E. Fluorography

1128. The clinical manifestations of acute stenotic laryngitis include: A. * Crass "barking" cough B. Dullness of percussion sound C. Have difficulties exhaling D. Moist rales in the lungs E. Emphysema

1129. Dry barking cough is characteristic for:A. *Laryngitis.B. Bronchitis.C. Flu.D. Pneumonia.E. Bronchiectasic pneumosderosis.

1130. Clear percussion sound is characteristic for:A. *Healthy children.B. Newborns.C. 6 month children.D. 5 years children.E. Adults.

1131. How many stages of respiratory stenosis are?A. 1B. 2C. *4D. 3E. 5

1132. What main clinical features are useful in the diagnosis of bronchial asthma?A. Chest pain B. *DispnaeC. TahycardiaD. VomitingE. Dry cough

1133. What conclusion after auscultation of the lungs will be in case of bronchial asthma? A. *Both types of rales B. Fine budding rales C. Sebelent dry rales D. Coarse bubling rales E. Crepitation rales

1134. What food is not limited during the acute period of glomerulonephritis? A. *Carbohydrates B. Salt C. Liquid D. Proteins E. Fats

1135. Which of the following laboratory findings is unusual in patients with simple (nutritional) rickets? A. *Hypercalciuria B. Hyperphosphaturia

C. Elevated levels of serum alkaline phosphatase D. Aminoaciduria E. Hypophosphatemia

1136. What should a daily doze of ergocalciferol be for prophylaxis of rickets for full-term breast-fed child? A. *400-500 IU B. 200-300 IU C. 300-400 IU D. 500-600 IU E. 100-200 IU

1137. What should a daily doze of ergocalciferol be for prophylaxis of rickets for premature infant? A. *1000-1200 IU B. 800-1000 IU C. 1200-1500 IU D. 400-500 IU E. 1300-1500 IU

1138. To what age should prophylaxis of rickets carry out for full-term breast-fed child? A. *1,5 years B. 2 years C. 2,5 years D. 1 year E. 6 months

1139. Determine a remedy for primary prophylaxis of spasmophilia for breast-fed child. A. *Ergocalciferol B. Calcium gluconate C. Calcium chloride D. Natrium chloride E. Calcium pantothenate

1140. What solution is necessary to prescribe for rehydratation? A. *Rehydron B. 5 \% glucose solution C. Boiled water D. Tea E. Broth of a camomile (medical)

1141. The child regurgitates after feeding by small portions of milk (changed and unchanged); stool is normal. Muscle tonus is normal. What is the most probable diagnosis? A. *Pylorospasmus B. Meningitis C. Pylorostenosis D. Microcephaly E. Craniostenosis

1142. Pollen of grasses causes: A. *Respiratory allergies B. Food allergies C. Contact allergies D. Medical allergies E. Croup syndrome

1143. Curvature of the spine in a child with rickets is due to:A. *Muscular hypotonia B. Muscle hypertonus C. Bone deformities D. Perverted osteogenesis E. Respiratory diseases

1144. “Drumsticks” symptom is characteristic for: A. congenital heart diseaseB. acute heart failure C. *chronic diseases of the heart and lungs D. polyarticular form of juvenile rheumatoid arthritis E. systemic form of juvenile rheumatoid arthritis

1145. Choice of antibacteryal preparations for treatment of pyelonephritis is determined: A. By age of the childB. By the cost of preparationC. * By the sensitiveness of bacteria cultured from urineD. By a country-producE. By duration of the disease

1146. Intravenous urography does not allow to discover: A. Position, sizes, shape of kidneysB. Functional state of nephronsC. Functional state of urinary tract D. * Anomaly of kidney vesselsE. Condition of urinary bladder

1147. For pyelonephritis typical is:A. HematuriB. ProteinuriaC. Leukocyturia and hematuriaD. * Leukocyturia and bacteriuriaE. Proteinuria and casts

1148. Investigation of sediment in 1 ml of urine is known as : A. * Nechyporenko testB. Amburzhe tesC. Reberg tesD. Kakovsky-Addys testE. Pasternatsky test

1149. The child has signs of intoxication, in a general analysis of urine significant pyuria, in daily urine - oxalic acid - 870 mmol / day (dysmetabolic nephropathy). Your diagnosis? A. * Secondary acute pyelonephritis B. Primary acute pyelonephritis C. Acute glomerulonephritis D. Interstitial nephritis E. Hereditary nephritis

1150. Main features of kidney’s edema are : A. * Appear in the morning, warm, palB. Appear in the second part of the day, cold, cyanoticC. DenseD. Disposed mainly on trunkE. Disposed mainly on sex organs

1151. Intoxication syndrome contains next symptoms : A. increased levels of total and conjugated bilirubin and cholesterol;B. increased levels of ASAT, ALAT, LDG;C. increased levels of amylase, tripsin, lipase;D. vomiting, nausea, heartburn feeling, decrease of appetite;E. * weakness, lucidity, bad sleep, headaches, irritability, tearfulness, increased disposition

to perspiration, blue shadows under the eyes, functional heart murmur.1152. The child has cloudy urine. What additional investigation will help to establish the diagnosis?

A. Zimnitsky testB. Complete blood test

C. * General urinalysis D. Analysis of urine for sugar from the urine daily amountE. Koprocytogram

1153. What is Pasternatsky’s symptom? A. Pain in palpation of lumbar region in the area of projection of kidneysB. Appearance of pain in lumbar region during trunk rotation C. * Appearance of pain at tapping lumbar region in the area of kidneysD. Appearance of pain in the lumbar area at coughingE. Appearance of pain in the projection of ureter at passing of stone from a kidney

1154. What changes in the urinary sediment are characteristic for pyelonephritis? A. Microhematuria, hyaline cylinders B. * Pyuria, bacteriuria. C. Hematuria, proteinuria. D. Proteinuria, granular cylinders, renal epithelium. E. Fresh red blood cells, oksalaturiya.

1155. Normally, in a general analysis of urine rate of red blood cells is: A. * should not beB. up to 1-2 in the field of view C. up to 5 in the field of view D. up to 15 in the field of view E. up to 10 in the field of view

1156. When does the displacement of one kidney appear on plane X-ray? A. In diffuse nephritisB. In the case of polycystosis C. In chronic kidney insufficiency of the II stage.D. * In nephroptosisE. In hydronephrosis.

1157. Dyspeptic syndrome contains next symptoms:A. increased levels of total and conjugated bilirubin and cholesterol;B. increased levels of ASAT, ALAT, LDG;C. increased levels of amylase, tripsin, lipase;D. * vomiting, nausea, heartburn feeling, decrease of appetite;E. weakness, lucidity, bad sleep, headaches, irritability, tearfulness, increased disposition to

perspiration, blue shadows under the eyes.1158. Chronic gastroduodenitis more frequent combines with diseases of:

A. CNS;B. * hepatobiliary system;C. respiratory system;D. heart and vessels;E. urinary system.

1159. Functional gastric disorders include:A. Chronic esophagitis. B. Chronic gastritis. C. * Duodenogastric reflux. D. Stomach ulcer. E. Chronic duodenitis.

1160. Pain in the epigastric area characterised:A. * gastritis B. cholangitis C. cholecystitis D. biliary dyskinesia E. colitis

Pediatrics.Tasks (ситуаційні задачі)

1. 12 years old female put is on a dispensary observation by gastroenterologist because of duodenal ulcer, biliary dyskinesia. How often antirelapse courses should be given?A. Every 2 monthsB. *Twice a yearC. Every 3 monthsD. Once a yearE. Three times a year

2. 1.5 years’ old child is ill for 1 week. Objectively: body temperature is 38.5? C, often moist cough, dyspnea in the rest. On X-ray: lungs’ roots are broad, infiltrated, and in both sides are little shadows. What form of an acute pneumonia is according to X-ray?A. InterstitialB. *BronchopneumoniaC. MonosegmentalD. PolysegmentalE. Crupous

3. 10-years’ old boy is ill for 4 days. He complaints on subfebrile temperature, dry cough. Objectively: pallor of the skin, red cheeks, greater on right side. Percussion: on right side dull sound in lowest part, and in axillar region. Auscultation: on right sight lower than angle of the scapula decrease of breathing, crepitating. What form of an acute pneumonia is possible in this case?A. BronchopneumoniaB. MonosegmentalC. PolysegmentalD. *CrupousE. Interstitial

4. 5-years' old child was hospitalized because of destructive pneumonia. The bacteriological investigation from pleural cavity has found staphylococci. What antibacterial medicine is better to prescribe?A. PenicillinB. AmpicillinC. VankomycinD. ErythromycinE. Gentamycin

5. A previously healthy 13-year –old boy develops a mild pneumonia characterised by a nonproductive cough. The therapy is:A. *MucalthinB. VentolinC. EuphullinD. PrednisolonE. Salbutamol

6. The 5-month's child with an acute pneumonia was hospitalized. The clinical blood analysis is: erythrocytes 2.86 х 10??/l, Нb - 86 g/l, CI – 0.8, leucocytes 11,2 х 109/l, eosynophyles - 3 %, neutrophyles: band forms - 2 %, segments - 30 %, lymphocytes - 58 %, monocytes - 7 %, ESR - 8 mm/hour. Name the pathological changes.A. Lymphopenia, anemiaB. *Leucocytosis, anemiaC. Leucopenia, increase of ESRD. Shift of the formula to the leftE. Lymphocytosis, monocytosis

7. The 7-months' old child has body temperature 38.3° C, cyanosis of perinasal triangle, breathing rate is 54 per minute. During percussion: dullness of the sound paravertebrally. During

auscultation: big amount of small moist riles all over the lungs, diminished breathing and crackles on the right. The most possible diagnosis is:A. Upper respiratory tract viral infectionB. Acute bronchitisC. Acute bronchiolitiD. *Acute bronchopneumoniaE. Acute interstitial pneumonia

8. The 7-years’ old child was hospitalized with complaints of high body temperature 38.5?C, often moist cough, dyspnea at rest. What investigation will resolve the diagnosis?A. Clinical analysis of a bloodB. Bacteriological investigation of sputumC. X-ray of chestD. SpirographyE. Bronchoscopy

9. The 9 months' old child who disturbs cough, dyspnea, subfebrile body temperature was examined by district pediatrician. Focal bronchopneumonia was suspected. What auscultation picture is typical for this case?A. Diffuse dry ralesB. Rough breathingC. Diffuse wet ralesD. Decrease breathingE. *Local crackles

10. The child is 7 years old. He has influenza for 5 days. The condition of the child sharply worsened. Once again has increased the body temperature, has appeared: moist cough with separation of mucous-purulent phlegm, dyspnea. Breathing - 30 per 1 min., cyanosis of perioral triangle, in lower parts of the lungs, more in the right, dullness of the lung sound, moist small rales. Pulse - 120 per 1 min., heart tones are weakened. What complication of influenza is possible?A. Croup syndromeB. *PneumoniaC. MeningitisD. MyocarditisE. Obstructive bronchitis

11. The child, 7 years old, has measles for 10 days. He complains of increasing of the body temperature to 39 ?С, general weakness, and periodic humid cough with mucous phlegm. Objectively: the general condition is moderate, skin is pale with pigmented rashes. On auscultation – dull sound, small rales in lower parts of lungs. What complication of the measles has appeared in child?A. TracheobronchitisB. Bronchitis C. BronchiolitisD. *PneumoniaE. Pharyngitis

12. An 18-month-old child is taken to hospital on the 4-th day of the disease. The disease began acutely with temperature 39°C, weakness, cough, restlessness. He is pale, has cyanosis, febrile temperature for more than 3 days. There are crackles and small bubbling rales at the auscultation on the right. Percussion sound is shortened in right under scapula area. X-ray picture: unhomogenous segmental infiltration 8-10 at the right, the increase of vascular picture, unstructural roots. What is the most likely diagnosis?A. BronchitisB. BronchiolitisC. Interstitial pneumoniaD. InfluenzaE. *Segmental pneumonia

13. 15 yr old female was admitted to thoracic surgery department with fever up to 40°C, onset of pain in the side caused by deep breathing, cough with considerable quantity of purulent sputum and blood with bad smell. What is the most likely diagnosis?A. Complication of liver echinococcosisB. Pulmonary tuberculosisC. *Abscess of the lungD. Actinomycosis of lungsE. Bronchiectatic disease

14. A 10-year-old boy complains of a headache, weakness, fever (temperature - 40° C), vomiting. On physical examination: there is an expressed dyspnea, pale skin with a flush on a right cheek, right hemithorax respiratory movement delays, dullness on percussion of the lower lobe of the right lung, weakness of vesicular respiration in this region. The abdomen is painless and soft by palpation. What disease causes these symptoms and signs?A. *Pneumonia crupousB. Intestinal infectionC. Acute appendicitisD. Acute cholecystitisE. Influenza

15. A 14-year-old patient has been treated in a hospital. A fever of 39°C, chest pain which is worsened by breathing, cough, brownish sputum appeared on the 7th day of the treatment. Chest X-ray shows left lower lobe infiltrate. Which of the following is the treatment of choice for this patient?A. *Cephalosporines of the III generationB. ErythromycinC. StreptomycinD. PenicillinE. Tetracycline

16. A 16-year-old male was admitted to the hospital complaining of stabbing back pain on inspiration and dyspnea. On exam, t - 37°C, Ps 92/min, BR of 24/min, vesicular breath sounds. There is a dry, grating, low-pitched sound heard in both expiration and inspiration in the left lower lateral part of the chest. What is the most likely diagnosis?A. *Acute fibrinous pleuritisB. Acute bronchitisC. PneumoniaD. PneumothoraxE. Myocarditis

17. A 3 month old infant suffering from acute segmental pneumonia has dyspnea (respiratory rate - 80 per minute), paradoxical breathing, tachycardia, total cyanosis. Respiration/pulse ratio is 1:2. The relative heart dullness borders are normal. Such signs are characteristic for:A. Congenital heart defectB. Respiratory failure of 0 degreeC. Respiratory failure of I degreeD. Respiratory failure of II degreeE. *Respiratory failure of III degree

18. A 6 year old girl has an acute onset of fever up to 39oC with chills, cough, and pain on respiration in the right side of her chest. On physical examination: HR - 120/min, BP- 85/45 mm Hg, RR- 36/min. There is dullness over the right lung on percussion. On X-ray: infiltrate in the right middle lobe of the lung. What is the diagnosis?A. Interstitial pneumonia B. Acute pleuritisC. Community-acquired lobar pneumonia D. Acute lung abscessE. Nosocomial lobar pneumonia

19. 8-month-old child was entered to the hospital. After bacteriological observation it is diagnosed atypical community-acquired Chlamidia trachomatis pneumonia. Select the best antibiotic.A. *Macrolides B. CarbopenemC. AminopenicillineD. Cephalosporin E. Aminoglycosides

20. A boy, 8 years old, has addressed to pediatrician with complains of increasing of the body temperature to 37.5 ?С, sore throat, cough, serous discharge from nose, lacrimation. During examination mild hyperemia and edema of the tonsils and back pharyngeal wall, conjunctives were revealed. The physician suspects adenoviral infection. Which method of express-diagnostics is better to use for acknowledgement of the diagnosis?A. The selection of the virus on tissue cultureB. *Immunofluorescent methodC. Serological investigationD. bacteriological investigationE. Bacteriosсopy investigation of pharyngeal swab

21. A patient with nosocomial pneumonia presents signs of collapse. Which of the following pneumonia complications is most likely to be accompanied by collapse?A. Exudative pleuritisB. Bronchial obstructionC. Toxic hepatitisD. *Septic shockE. Emphysema

22. An 18-month-old child is taken to hospital on the 4-th day of the disease. The disease began acutely with temperature 39°C, weakness, cough, restlessness. He is pale, has cyanosis, febrile temperature for more than 3 days. There are crackles, fine bubbling rales at the auscultation. Percussion sound is shortened in right under scapula area. X-ray picture: unhomogenous segmental infiltration 8-10 in the right, the increase of vascular picture, unstructural roots. What is the most likely diagnosis?A. BronchitisB. BronchiolitisC. Interstitial pneumoniaD. InfluenzaE. *Segmental pneumonia

23. A 7-year-old boy has body temperature 39.4°C, productive cough and intoxication. During the examination: a voice fremitus is stronger, short percussion sound, depressed breathing and bronhophonia over the right lung. The X-ray: a homogeneous infiltration of the right lung lower lobe. What is the diagnosis? A. *Acute right-sided lobar pneumoniaB. Acute right-sided segmental pneumoniaC. Acute right-sided pleuritisD. Acute right-sided pneumothoraxE. Acute right-sided intersticial pneumonia

24. A 9 month-old baby has fever, cough, dyspnea. She is sick for 5 days after contact with ARVI patient. Condition of the child is severe. Temperature 38°C, nasolabial triangle is cyanotic. RR 54 per 1 min, nostrils flaring during breathing. Percussion: a shortening of the sound below the right scapula angle, over the other sites - tympanic sound. On auscultation - small bubbling moist rales on both sides, more on the right side. What is the most likely diagnosis?A. Acute bronchiolitisB. ARVIC. Acute laryngotracheitisD. Acute bronchitis

E. *Acute pneumonia25. A child of 10 months has acute bronchopneumonia with destruction of the left lung. Which agent

is most likely caused this disease?A. Pneumococcus (S. pneumoniae)B. *St. aureusC. Colibacillus (E. Coli)D. Pseudomonas aeruginosaE. Proteus vulgaris

26. Patient 14yrs during the week noted the deterioration of general condition, sub-febrile temperature. Then suddenly developed fever to 38°C, there was pain in the right half of the chest, a dry cough. On the right of the 3 ribs downwards shortening of percussion sounds is determined. Above the shortening of the sound breathing is not listened. The blood leukocytes 10.5x109, ESR - 32mm/h. X-ray on the right intense homogenous darkening of 3 ribs down. The heart is shifted to the left. What disease should be suspected in a patient?A. *Pleural effusionB. PneumoniaC. Spontaneous pneumothoraxD. Cancer of the lungE. Acute lung abscess

27. Child 2.5 years is acutely ill. Body temperature is raised to 38° C and appeared with abdominal pain, vomiting once. Condition of the child is severe. Skin is pale. Breathing rate is 80 per minute. There is cyanosis of naso-labial triangle. Dullness of percussion sound is in the right lower part of chest, there is diminished breathing, crackles is absent. Abdomen is moderately swollen, painful to palpation in the right upper ribs region. At chest X-ray- right side is infiltrated in 7-10 segments. What is the most likely diagnosis?A. Membranous pneumoniaB. Acute appendicitisC. Right side pericardial pleurisyD. Acute obstructive bronchitisE. *Acute right side polysegmental pneumonia

28. Child in 12y.o. diagnosed pneumonia, which is caused by Mycoplasma pneumoniae. What treatment should be chosen?A. AdrenalinB. CephalosporinesC. AminopenicillinD. *MacrolideE. Antifungal drugs

29. Child is two years old. The mother appealed to the district pediatricians with complaints of child cough, runny nose, fever up to 38.5°C, weakness and decrease in appetite. Objectively: condition of the child is moderately severe, skin is pale, RR 40 per 1 min., Auscultation: lung breathing is hard, there is crackles on the lower right side. What is the diagnosis?A. Acute bronchiolitisB. *Pneumonia C. Obstructive bronchitisD. Acute bronchitisE. Bronchial asthma

30. A 2 years old child has dry cough, dyspnea, body temperature is 37.5°C. Percussion: clear pulmonary sound without dullness. Auscultation: dry whistling and different moist rales. In the peripheral blood: leukocytosis, eosynophylia, increased ESR. What disease is possible?A. Acute simple bronchitisB. *Obstructive bronchitisC. Whooping coughD. Acute pneumonia

E. Bronchial asthma31. Pediatrician was called to the 2-years old child who’s mother complaints of a subfebrile

temperature, rhinitis and dry cough. He is ill for 3 days. During percussion: a clear pulmonary sound without dullness. During auscultation: puerile breathing. Laboratory findings: leucopenia, lymphocytosis, increased ESR. What disease is possible first of all?A. Acute obstructive bronchitisB. *Acute tracheitisC. Acute bronchopneumoniaD. Recurrent bronchitisE. Acute bronchitis

32. The 5-months' old child has subfibrile temperature, paroxysmal cough and dyspnea. He is hospitalized. 3 days ago he was in a contact with sister ill on acute upper respiratory tract viral infection. Objectively: the condition is severe, skin is cyanotic, considerable expiration dyspnea, oral crepitation. Percussion: tympanic sound. Auscultation: a scattered, wet rales in both sides, respiratory rate is 80 per 1 minute. What disease is possible?A. Bronchial asthmaB. Aspiration of a foreign bodyC. Acute bronchitisD. *BronchiolitisE. Acute pneumonia

33. The 4 year child with frequent respiratory tract, which are observed from the 1st year of life, is examined in the hospital. The most typical clinical manifestations of lung malformation are the following symptoms, except:A. Recurrent bronchitis or pneumoniaB. Shortness of breath during physical exercisesC. Deformation of the chestD. *Attacks of dyspnea at nightE. Fingers clubbing and “drum sticks”

34. A district pediatrician examined the 5-year-old child, whom disturbed wet cough. During this year, same problems developed for the third time. After clinical examination a doctor thought about chronic disease of lungs. What auscultation signs are characteristic for this disease?A. Dry diffuse ralesB. Weakened breathingC. Moist ralesD. *Persistent local moist ralesE. Hard breathing

35. A patient with stomach bleeding was hospitalized. Endoscopic examination revealed an acute gastric ulcer. In the history: rheumatoid arthritis, pharmacotherapy. Name the drug that caused the development of acute gastric ulcer and bleeding.A. DelagylB. *Sodium diclofenacC. KlaritinD. Dekaris (levamisole)E. Plazmol

36. The 7-years’ old child was hospitalized with complaints on wet cough. His condition has worsened 10 days ago. He is ill during last 4 years: viral infections 3-4 times per year are accompanied by bronchitis. Objectively: cough with sputum. Percussion: a clear pulmonary sound. Auscultation: rough breathing, nonconstant single diffuse wet rales. X-ray: lung pattern is increased, roots are nonstructural. What disease is possible in this case?A. Viral infectionB. Acute bronchitisC. *Relapsed bronchitisD. Acute pneumonia

E. Chronic disease of lungs37. The child is 7 years old. He has influenza for 5 days. The condition of the child sharply worsened.

Once again has increased the body temperature, has appeared: moist cough with sputum, dyspnea. RR - 30 per min., cyanosis of perioral triangle, in lower parts of the lungs is dullness of the lung sound, moist small rales. Pulse - 120 in 1 min., heart tones are weak. What complication of influenza is possible?A. Croup syndromeB. *PneumoniaC. MeningitisD. Myocarditis E. Obstructive bronchitis

38. The child, 2 years old, is treated from influenza. His condition suddenly became worse: the body temperature has increased to 39.8 ?С, has appeared rough barking cough, hoarseness of the voice, expressed inspiratory dyspnea. Objectively: involvement of the auxiliary muscles in breathing, skin pallor, tachycardia. What complication of influenza has developed?A. PneumoniaB. BronchiolitisC. Croup syndromeD. Obstructive bronchitisE. Pharyngitis

39. A 2 years old child has dry cough, dyspnea, body temperature is 37.5°C. Percussion: tympanic sounds. Auscultation: breath sounds are coarse, dry whistling and different moist rales. In the peripheral blood: leucocytosis, eosynophylia, increased ESR. What disease is possible?A. Acute simple bronchitisB. *Obstructive bronchitisC. Whooping coughD. Acute pneumoniaE. Bronchial asthma

40. Pediatrist was called to the 2-years child in connection with subfibrile fever, rhinitis and dry cough. Child is sick 3-d day. Percussion: a clear pulmonary sounds. Auscultation: breath sounds are coarse. By results of examinations: a leucopenia, lymphocytosis, accelerated ESR. What disease is possible first of all?A. Acute obstructive bronchitisB. Acute tracheitisC. Acute bronchopneumoniaD. Relapse bronchitisE. *Acute bronchitis

41. The 9-month's child in a severe condition is hospitalized. Diagnose is bronchiolitis. What is the reason of the severe condition of the child?A. NeurotoxicosisB. Heart failurC. *Respiratory failureD. Epinephral failureE. Exicosis

42. The 5-years child has obstructive bronchitis. There is non-productive cough and dyspnea. What drugs will be as a base of therapy?A. AntibioticsB. HyposensibilizationC. *BroncholiticsD. MucoliticsE. Hormones

43. The 5-month's child with the complaints of subfibrile fever, inefficient tussis, dyspnea is hospitalized. He was ill 3 days ago after a contact with ill on virus sister. Objectively: the

condition is very severe, skin is cyanotic, considerable expiration dyspnea, oral crepitation. Percussion: sound boxes. Auscultation: prolonged expiratory, scattered whistling sounds, RR 80 per 1 minute. What disease is possible?A. Bronchial asthmaB. Aspiration of a foreign bodyC. Acute bronchitisD. *BronchiolitiE. Acute pneumonia

44. 12-month old child after contact with cat has a frequent paroxysmal, irritative, nonproductive cough. The child has short breathing, he tries to breathe more deeply and the expiratory phase becomes prolonged and is accompanied by an audible wheezing. His lips are cyanotic, cyanosis observed in the nail beds and skin, especially around the mouth. The child is restless and anxious. Sweating is prominent as the attack progresses. In history: allergy on food products. Put your diagnosis.A. Acute pneumoniaB. Respiratory virus infectionC. *Bronchial asthmaD. Obstructive bronchitisE. BronhiolitisF. 45. The child of 6 years – diagnose asthma bronchial the first time. After liquidation of

attack period it was decided to prescribe antiinflamation therapy. It is necessary to give:A. AspirinB. DiclofenacC. PrednizolonD. *Nedocromil E. Іndometacini45. The child of 10 years – diagnose bronchial asthma during 5 yrs. Attack periods appear in summer

during flowering. In period between attacks the child needs such treatment:A. AntiinflamationB. AntibacterialC. BroncholiticalD. MucoliticalE. *Inhaled corticosteroids

46. The child of 10 years – diagnose bronchial asthma during 5 years. Duration of an attack period is more than 6 hours. It is necessary to prescribe:A. *PrednizolonB. AdrenalinC. ІntalD. SuprastinE. Еfedrin

47. 1.5 years old child is sick the second day. Body temperature is 37.8°C, restless, barking cough, hoarse voice, noisy breathing, stridor, pallor of skin, retraction at rest. Put diagnosis.A. PneumoniaB. *LaryngitisC. BronchiolitisD. Foreign body aspirationE. Bronchial asthma

48. A child 10 months was entered to the hospital in severe condition with expiratory shortness of breath, dry cough, the temperature 38°C. At percussion over lungs there is tympanic sound. Auscultation reveals prolonged expiration, many dry wheezing and occasional wet rales on both sides. What is your diagnosis?A. Bronchial asthmaB. Pneumonia

C. *Acute obstructive bronchitisD. PertussisE. Acute bronchitis

49. A child is 11 months. He is ill ARI. On the second day it was marked the emergence of a barking cough, hoarse voice, strdor, breath difficulties, shortness of breath, cyanosis. In what department is hospitalized child?A. PulmonaryB. InfectiousC. Junior childhoodD. OtolaryngologyE. *Intensive care unit

50. A child 5 years is ill. Premorbid anamnesis is good. There is a mild indisposition, increasing t° to 37.3°C, decreased appetite, frequent coughing, significant nasal drainage. Objectively: RR-25 per 1 min., percussion lung sound, auscultation reveals different bubble rales, after the cough character of rales changes. It was diagnosed ARI, acute bronchitis. What kind of treatment is most faithful?A. AminoglycosidesB. CephalosporinesC. SulphamidsD. *Mucolytics and vitaminsE. Antihistamines

51. A child 7 years is ill. There is a slight indisposition, increasing t° to 37.3°C, decrease of appetite, frequent coughing, significant nasal drainage. ARI is diagnosed, acute bronchitis. What kind of treatment is most faithful?A. AntibioticsB. VitaminsC. *MucolyticsD. PhysiotherapyE. Antihistamine

52. A child of 9 months is acutely ill: increased body temperature to 38,8°C, cough, runny nose. On 3d day of illness, at night, the condition became worse: the child became restless, barking cough, stridor and hoarse voice appeared, inspiratory shortness of breath. It was diagnosed croup. Which of the pathogens most likely is the reason of croup in children?A. AdenovirusB. Influenza C. *Virus parainfluenzaD. RynovirusE. Enteroviruse

53. Children under eight months came to hospital complaining of non-productive, intense cough for 2 days, wheezing breath, increased body t° to 37.8°C. Objectively: perioral cyanosis; barrel thorax; auscultation: different wet rales. It was diagnosed obstructive bronchitis. What is the basic therapy?A. *BroncholyticsB. CorticosteroidsC. AntibioticsD. MukolyticsE. Enzymes

54. Child is 2 years. He is Ill at the first time. Sick 2 days: t° - 37.4°C, dry cough, RR 60 per min, expiratory dyspnea. There is box sound over lungs. Auscultation reveals hard breathing, crepitation and dry whistling. What is the diagnosis?A. BronchiolitisB. Congenital stridorC. Acute bronchitis

D. PneumoniaE. *Acute obstructive bronchitis

55. 8 year old child complains of coughing with purulent sputum, especially in the morning. He is ill 5 years after foreign body aspiration episode. After that he had pneumonia two times. What disease is the most probable?A. *Chronic disease of lungsB. Recurrent pneumoniaC. Recurrent bronchitisD. Pulmonary tuberculosisE. Bronchial asthma

56. Child is 2 years. He is Ill at the first time. Sick 2 days: t° - 37.4°C, dry cough, RR 30 per min. There is clear lung sound. Auscultation reveals hard breathing, crepitation and dry rales. The X-ray shows increasing of pulmonary pattern. What is the most likely diagnosis?A. *Acute (simple) bronchitisB. Obstructive bronchitisC. Congenital strydorD. PneumoniaE. Bronchiolitis

57. Child is four months old. He is acutely ill with fever to 37.8°C, coughing. On 3d day cough increased, shortness of breath appeared, percussion tympanic sound over lungs, auscultation: prolonged expiration, a large number of fine wet and wheezing rales on exhalation on both sides. What is your preliminary diagnosis?A. ARI, pneumoniaB. *ARI, acute bronchiolitisC. ARI, acute bronchitisD. ARI, obstructive bronchitisE. ARI, focal pneumonia

58. Child is 6 months. He is acutely ill with fever to 37.8°C, coughing. On 3d day cough increased, shortness of breath appeared, percussion tympanic sound over lungs, auscultation: on both sides a large number of fine wet and wheezing rales on exhalation. Acute bronchiolitis was diagnosed. What is the reason of such disease?A. RinovirusB. FluC. AdenovirusD. Virus parainfluenzaE. *Respiratory syncytial virus

59. The girl 10 months was entered to the hospital the next day of illness with complaints of increase body temperature to 39°C, dry, barking cough. After clinical examination the diagnosis of acute laryngitis was established. What breathing disorder is characteristic for this situation?A. Mixed breathlessnessB. Expiratory dyspneaC. Hoarse breathD. *Inspiratory dyspneaE. Stridor breath

60. Three-year-old boy was delivered to the hospital at night by ambulance. He is ill 2 days. On background of ARVI loud inspiratory dyspnea occurred with retraction of intercostal spaces, blowing nostrils and barking cough. The most probable diagnose is:A. EpiglottitisB. Bronchial asthmaC. *Viral false croupD. Real (diphtheric) croupE. Foreign body of air ways

61. 5-year-old child is hospitalized with complaints of wet cough. He is ill for 3 years, he had left side low lobe pneumonia three times before. During examination: skin is pale, perioral and periorbital cyanosis. During percussion: local dullness below the lower corner of the left scapulae. During auscultation: many fine moist rales under the left scapulae. X-ray of chest: deformation of bronchial pattern on the left side. Which of the following will prove diagnosis?A. Biplane (two-dimensional) chest x-rayB. Chest ultrasoundC. CT of chestD. *Bronchoscopy or bronchographyE. Spirography

62. 6-year-old child is hospitalized with constant complaints of cough with purulent sputum. He is ill during 3 years, exacerbations periods develop 3-4 times per year. After the clinical and instrumental examinations was diagnosed chronic disease of lungs. What is the basic complex treatment of this disease?A. *Restoration of drainage function of bronchiB. Antibacterial therapyC. PhysiotherapyD. Desensitization therapyE. Physiotherapy

63. The child 5 years was admitted to the hospital with rapid breathing disorders. Skin is pale, acrocyanosis, stenotic breathing, breathing with participation of auxiliary muscles, retractions at rest, hoarse voice. He has contact with ill ARI person. Your diagnosis is:A. *LaryngotracheitisB. Laryngeal papillomatosisC. BronchitisD. Foreign body of larynx E. Foreign body of trachea

64. A 2 year boy is admitted to the hospital with complaints of persistent cough, periodic diarrhea and malnutrition. He had bronchitis 4 times before and pneumonia 2 times before. Also was diagnosed sinusitis. What is the most likely preliminary diagnosis?A. Chronic pneumoniaB. TracheobronchomegalyC. *Cystic fibrosisD. Intestinal infectionE. ?-1-antitripsin deficiency

65. A 2 months boy is admitted to the hospital with complaints of persistent cough, dyspnea, cyanosis, prolonged jaundice, hepatomegaly. During lungs percussion there is bandbox sound. What is the most likely preliminary diagnosis?A. Chronic pneumoniaB. TracheobronchomegalyC. Cystic fibrosisD. Intestinal infectionE. *?-1-antitripsin deficiency

66. A 5 year girl is suffering from bronchitis frequently. The physical development is delayed. There is persistent cough with mucous-purulent sputum. The doctor suspected chronic disease of lungs. What symptoms will allow to the physician to make such conclusion?A. Delay in physical developmentB. Cough with mucous-purulent sputumC. Relapsing course of disease D. Persistent local changes in lungE. *All mentioned above

67. A 5 year child is suffering from bronchitis frequently. The physical development is delayed. There is persistent cough with mucous-purulent sputum. The doctor suspected chronic disease of

lungs. Which method is the most informative to confirm this diagnosis?A. *BronchographyB. SpirographyC. EchocardiographyD. X-ray of the chestE. Sputum analyses

68. A 5 year old boy was hospitalized with a preliminary diagnosis of chronic disease of lungs. He is ill during 4 years. There is a constant wet cough, persistent moist rales in the lower lobe of the right lung. Which of the following will prove diagnosis?A. Biplane (two-dimensional) chest x-ray B. Chest ultrasoundC. BronchoscopyD. *BronchographyE. Spirography

69. In 5 year child who has frequent maxillary sinusitis and respiratory diseases was suspected Kartagener syndrome. All symptoms are typical for this disease EXEPT:A. Situs inversusB. Chronic bronchitis with bronchiectasisC. EthmoidoantritisD. *NephritisE. Endocrine glands hypofunction

70. 5-year-old child is hospitalized with complaints of wet cough, shortness of breath during exercise. Condition worsened 10 days ago after contact with patients with acute respiratory infection. From anamnesis it is known that a child is suffering from pneumonia 2-3 times per year. There was a suspicion of chronic disease of lungs. What research will be definitive in establishing of the final diagnosis?A. ArteriographyB. X-ray of the chestC. SpirographyD. BronchoscopyE. *Bronchography

71. 8-year-old child is treated in a hospital from chronic disease of lungs. Sputum was taken for bacteriological study from the bronchi during medical and diagnostic bronchoscopy. Pneumococci were revealed, sensitive to cephalosporines. Which way is the best for introduction of antibiotics?A. OralB. IntravenousC. *EndobronchialD. IntramuscularE. Electrophoresis

72. In 3 year child was revealed situs inversus, chronic recurrent bronchitis, frontal sinuses hypoplasia, congenital heart defect. The most probable diagnosis is:A. Mounier-Kuhn syndromeB. Williams-Campbell syndromeC. ?-1-antitripsin deficiencyD. *Kartagener syndromeE. Cystic fibrosis

73. A district pediatrician examined the 9-year-old child, whom disturbed wet cough and shortness of breath during physical exercises. Child is sick during 6 years: frequent respiratory diseases, 1-2 times per year - pneumonia. After analysis of anamnesis and clinical examination chronic disease of lungs was diagnosed. What is the most typical symptom of this disease?A. DyspneaB. Perioral cyanosis

C. Pale skinD. *Persistent coughE. Subfebrile temperature

74. The 7 year boy is suffering from persistent wet cough. There was a suspicion of chronic disease of lungs. What chest X-ray changes are the most typical for the chronic disease of lungs?A. Local infiltration of lung tissueB. Presence of the round shape shadowC. *Intensification and deformation of the lungs patternD. Lungs extension and hyperaerationE. Heart extension

75. A 5 year boy is suffering from bronchitis and pneumonia often. At bronchologic examination bronchiectasis were revealed. In addition, the child suffers from chronic sinusitis, there is dextracardia. The most likely diagnosis is:A. Mounier-Kuhn syndromeB. *Kartagener syndromeC. Williams-Campbell syndromeD. Hammen-Rich syndromeE. Cystic fibrosis

76. In 1 year child was revealed chronic recurrent bronchitis and tracheobronchomegaly, ethmoidoantritis. The most probable diagnosis is:A. *Mounier-Kuhn syndromeB. Williams-Kempbell syndromeC. Alfa-1-antitripsin deficiencyD. Kartagener syndromeE. Cystic fibrosis

77. In 5 year child was revealed dyspnea during physical exercises, heart pain, collapse attacks, cyanosis, 2 tone accent above the pulmonary artery, systolic murmur above the pulmonary artery. Chest x-ray revealed deceased intensity of the lungs pattern, enlarged heart. The most probable diagnosis is:A. Mounier-Kuhn syndromeB. Williams-Campbell syndromeC. ?-1-antitripsin deficiencyD. *Primary pulmonary hypertensionE. Idiopathic pulmonary hemosiderosis

78. In 5 year child was revealed dyspnea during physical exercises, heart pain, collapse attacks, cyanosis, 2 tone accent above the pulmonary artery, systolic murmur above the pulmonary artery. Chest x-ray revealed deceased intensity of the lungs pattern, enlarged heart. The most probable diagnosis is:A. Mounier-Kuhn syndromeB. Williams-Kampbell syndromeC. alfa-1-antitripsin deficiencyD. *Primary pulmonary hypertensionE. Idiopathic pulmonary hemosiderosis

79. In 8 month child there is high possibility of cystic fibrosis according the clinical and anamnestic criteria. What examination will prove the diagnosis?A. Level of lipids in bloodB. *Level of chlorides in the sweatC. KoprogramD. Chest x-rayE. Level of ?-1-antitripsin in blod

80. The 6 year boy was hospitalized with the complaints of shortness of breath, frequent cough, loss of weight. He is ill during 4 years since the idiopathic fibrosing alveolitis was diagnosed. What is pathogenic treatment:

A. AntibioticsB. *CorticosteroidsC. NSAID’sD. BroncholyticsE. Immune modulators

81. A 7-year-old child was hospitalized with exacerbation of chronic disease of lungs. A child was born from premature pregnancy. He suffered from rickets and anemia during the first year of life, in 1,5 yr - acute pneumonia, in two years a foreign body aspiration (button). He has acute respiratory tract infections 3-4 times per year. What probably caused the development of chronic disease of lungs?A. PrematurityB. RicketsC. AnemiaD. *Foreign body aspirationE. Frequent acute respiratory tract infections

82. A 10 year child is suffering from bronchitis frequently. The physical development is delayed. There is persistent cough with mucous-purulent sputum. The doctor suspected bronchiectasis. Select the primary method for diagnosis of bronchiectasis:A. *BronchographyB. BronchoscopyC. CT of the chestD. Chest x-rayE. Scintigraphy

83. Patient is 5 years old. He has chronic cough, recurrent pneumonias and poor physical exercises resistance. Auscultation reveals a murmur and congestive cardiac failure.A. iogram reveals signs of lungs tissue consolidation. Vomiting, failure to thrive and

abdominal pain periodically occur. Very often there are cough, fever, shortness of breath and small bubbling moist rales. What diagnosis is suspected?

B. *Cystic adenomatous malformationC. Sequestration of the lung D. Aplasia of lungE. Lung atelectasisF. Idiopathic fibroalveolitis

84. Child is 6 years old. He has frequent respiratory diseases. There is a poor tolerance to physical activity, development of shortness of breath, sometimes accompanied by attacks of breathlessness. Syncope often appears at an exercise, heart failure signs are present. Put the most probable diagnosis.A. *Idiopathic pulmonary hypertensionB. Idiopathic hemosiderosisC. Idiopathic fibroalveolitisD. Congenital carditisE. Cardiomyopathy

85. At 3 months child three times bronchoobstructive syndrome occurred without previous catarrhal phenomena. The cough is unproductive. Stool has unpleasant smell from the first days of life. Diagnosis of cystic fibrosis, mixed form was suggested. What research can confirm the diagnosis?A. Radiography of the chestB. X-ray of lungsC. CoprogramD. *Determination of sweat chlorides E. Determination of blood lipids

86. The child is 4 years old. During the first year of life an acute pneumonia occured, and then a chronic bronchopulmonary process was formed eventually. Objectively chest is looking like hump.The cough is resistant with shortness of breath. At percussion of the lungs there is bandbox

sound, at auscultation– dry and moist rales of various sizes. Phalanges and nails become "drumsticks", "hour-glass", a violation of external respiration is present. Radiological findings in the lungs: increased pulmonary pattern, the phenomenon of emphysema. At bronchography there are determined generalize bronchiectasis with balloon expansion during inspiration and collapse. Put diagnosis.A. *Williams– Campbell syndromeB. Kartagener syndromeC. Idiopathic fibroalveolitisD. Mounier -Kuhn syndromeE. Cystic fibrosis

87. A patient with nosocomial pneumonia has signs of collapse. Which of the following pneumonia complication is the most likely to be accompanied with collapse?A. *Septic shockB. Toxic hepatitisC. Bronchial obstructionD. EmphysemaE. Exudative pleurisy

88. A 1-year-old child with fibroelastosis fell ill with acute respiratory viral infection. Suddenly he develops anxiety, acrocyanosis, Ps- 132 per min, BR - 50/min, small bubbling rales in lower lungs, pO2 60 mmHg, pCO2 55mm Hg. On X-ray: cardiomegaly, amplification of lung pattern, radix pulmonis resembles butterfly wings. What is the most likely reason of child condition worsening?A. Pulmonary abscessB. Angioneurotic (Quincke's) edemaC. BronchiolitisD. Double-sided pneumoniaE. *Pulmonary edema

89. A 6-year-old boy has suffered from tonsillitis. In 2 weeks he started complaining of migratory joint pain, edema of joints, restriction of movements, fever. On examination: an acute rheumatic heart disease, activity of the III-rd degree, primary rheumocarditis, polyarthritis; acute course of disease, cardiovascular failure II A. What medication should be prescribed?A. DelagilB. *PrednisoneC. DiprazinumD. ErythromycinE. Cefazolin

90. The boy of 3 months has poor increase in mass at satisfactory appetite, periodic cyanosis of the child during feeding, the changes on the part of heart are auscultated from birth, deficit of weight 15 %, paleness and xeroderma; rough systolic murmur in all points, is maximum in ІІІ left intercostal area. What diagnosis is possible to suspect?A. Atrial septal defectB. *Fallout illnessC. Ventricular septal defectD. Stenosis of pulmonary arteryE. Coarctation of aorta

91. The neonatal boy was born with weight 3,100 from І normal pregnancy, which one past with toxicosis during the first trimester, has cried at once; rough systolic murmur in ІІ intercostal to the left of a breast bone, skin pink, clean. What diagnosis is possible to suspect?A. Patent ductus arteriosesB. Ventricular septum defectC. *Atrial septal defectD. Coarctation of aortaE. Stenosis of pulmonary artery

92. Child 2 month was born premature. Clinically: mild cardiomegaly, proof arrhythmias from birth, which are interrupted by medicines. EchoCG: moderate dilatation of the left ventricle, hypokinesia of its walls, without the morphological changes. Late congenital carditis was diagnosed. What can be the reason of late congenital carditis?A. Hereditary predilectionB. Anemia of pregnant womenC. *Acute respiratory infection in the last trimester of pregnancyD. Prenatal malnutritionE. Birth trauma

93. A 1.5 years girl with problematic obstetrics anamnesis, decrease of physical development, deficiency of mass 24%. During objective examination: paleness of skin and mucus, quantity of breathing-52 per minute with participation of additional muscles, expressed cardiomegalia, liver +4sm, and quantity of heart beating-145, edema. What disease can be in this case?A. Rheumatic myocarditisB. Acute no rheumatic carditisC. Exudative perycarditis D. Hereditary nonrheumatic carditisE. *Fibroelastosis

94. The 10-year-old girl marks a frequent headache, dizziness, noise in ears; such complaints are present during 6 months, with the tendency to increase; physical development normal, a hypersthenic constitution; cardiomegalia, systolic murmur on apex and between scapulae, a tachycardia; arterial pressure on hands-180/100. What diagnosis the most probable in this case?A. Essentsial hypertensionB. Renal hypertensionC. Stenosis of aortaD. *Coartation of aortaE. Pheochromocytoma

95. Parents of the 7-year-old boy come to the neurologist with complaints of decrease in his movement activity, disturbance of walking. These complaints developed gradually, without any reason. At examination: irregularity of development of the upper and lower parts of body, a hypotonia of muscles of the feet, an absent pulsation on the femoral arteries, systolic murmur in intrascapular region. What pathology can be suspected at the patient?A. *Coartation of aortaB. Kawasaki illnessC. Takajasu IllnessD. Aneurysm of aortaE. Endartereitis of femoral arteries

96. The girl of 6 months is ill during 10 days. The disease began acutely with high temperature and catarrhal syndrome. During objective examination: paleness of skin and mucus, dyspnea, acrocyanosis, cardiomegalia. During nonrheumatic carditis is surprised more often:A. All covers of heartB. *MyocardiumC. EndocardiumD. PericardiumE. Myocardium and pericardium

97. Child is 2 month old. Clinically: cardiomegaly at birth, its rapid progress, formation of cardiac hump, progressive left-heart cardiac insufficiency, refraction to the therapy. Early hereditary carditis belongs to:A. Acute myocarditisB. Chronic myocarditisC. *FibroelastosisD. Cardiac sclerosisE. Myocardiodistrophy

98. Child 2 month was born premature. Clinically: mild cardiomegaly, proof arrhythmias from birth, which are interrupted by medicines. EchoCG: moderate dilatation of the left ventricle, hypokinesia of its walls, without the morphological changes. Late congenital carditis was diagnosed. Late hereditary carditis is formed in terms of gestation:A. 1 – 3 monthsB. 4 – 6 monthsC. *7 – 9 monthsD. During a birthE. 1-6 months

99. Child is 2 month old. Clinically: cardiomegaly at birth, its rapid progress, formation of cardiac hump, progressive left-heart cardiac insufficiency, refraction to the therapy. Fibroelastosis was diagnosed. It is possible to suspect fibroelastosis when: A. *The refraction to therapy tachycardiaB. Rough systolic murmur on apexC. BradiarythmiaD. Diastolic murmur on apexE. Diastolic murmur on aorta

100. Child is 2 month old. Clinically: cardiomegaly at birth, its rapid progress, formation of cardiac hump, progressive left-heart cardiac insufficiency, refraction to the therapy. Fibroelastosis was diagnosed. For changes during fibroelastos it is not characteristic:A. Cardiac humpB. Decrease of physical developmentC. HepatomegaliaD. *LeucocytosiE. Cardiac noise

101. Child 5 yrs old was entered to the hospital with complaints of weakness, decline of appetite, shortness of breath. Clinically it was revealed cardiomegaly, weakness of heart tones, arrhythmias, soft systolic murmur on the apex. Acute nonrheumatic carditis was diagnosed. It is caused by:A. VirusesB. BacteriaC. ToxinsD. Allergic conditionsE. *All transferred

102. Child 5 yrs old was entered to the hospital with complaints of weakness, decline of appetite, shortness of breath. Clinically it was revealed cardiomegaly, weakness of heart tones, arrhythmias, soft systolic murmur on the apex. Acute nonrheumatic carditis was diagnosed. For acute nonrheumatic carditis is not characteristic: A. TachycardiaB. CardiomegaliaC. Dullness of heart tonesD. *Pain in jointsE. Bradycardia

103. Child 8 yrs old was entered to the hospital with complaints of weakness, decline of appetite, shortness of breath. Clinically it was revealed cardiomegaly, weakness of heart tones, arrhythmias, soft systolic murmur on the apex. Acute nonrheumatic carditis was diagnosed. The forecast during acute nonrheumatic carditisA. *RecoveringB. Death till 6 yearsC. Death till 3 yearsD. Death till 1 yearE. The illness during all life

104. Child 7 yrs old had acute nonrheumatic carditis 2 year ago. 1 year ago it was relapse of carditis.

Mother interrupted therapy after 1 month of treatment. Clinically it is revealed physical retardation, cardiac hump and signs of cardiac failure. A. Chronic nonrheumatic carditis is diagnosed. The forecast is:B. *RecoveringC. Cardiac insufficiencyD. Death till 3 yearsE. Death till 1 year

105. Child is 3 month old. Clinically: cardiomegaly at birth, its rapid progress, formation of cardiac hump, progressive left-heart cardiac insufficiency, refraction to the therapy. Fibroelastosis was diagnosed. Cardinal in treatment of fibroelastosis is:A. *Heart transplantationB. Surgical correction of infringements of hemodynamicsC. Catheterisation of heart cavitiesD. Antibiotic therapyE. Hormones therapy

106. Child 7 yrs old was entered to the hospital with complaints of weakness, decline of appetite, shortness of breath. Clinically it was revealed cardiomegaly, weakness of heart tones, arrhythmias, soft systolic murmur on the apex. Acute nonrheumatic carditis was diagnosed. Preparation of choise during acute nonrheumatic carditis is:A. *AspirinB. PlanquenilC. DigoxinD. DelagilE. Lasix

107. Child 7 yrs old had acute nonrheumatic carditis 1 year ago. Mother interrupted therapy after 1 month of treatmen Clinically it is observed periodic cardialgias, weak heart tones, tachyarrhythmia, functional systolic murmur. Subacute nonrheumatic carditis is diagnosed. Preparation of choise is:A. AspirinB. PrednisoloneC. DigoxinD. *DelagilE. Indometacin

108. Child 9 yrs old was entered to the hospital with complaints of weakness, decline of appetite, shortness of breath. Clinically it was revealed cardiomegaly, weakness of heart tones, arrhythmias, soft systolic murmur on the apex. Acute nonrheumatic carditis was diagnosed. The daily dose of prednisolon during acute nonrheumatic carditis is:A. *0,5 – 1,5 mg/kgB. 1,5 – 2,5 mg/kgC. 2,5 – 3,5 mg/kgD. 3,5 – 4,5 mg/kgE. 5 mg/kg

109. Child 10 yrs old had acute nonrheumatic carditis 2 year ago. 1 year ago it was relapse of carditis. Mother interrupted therapy after 1 month of treatment. Clinically it is revealed physical retardation, cardiac hump and signs of cardiac failure. Chronic nonrheumatic carditis is diagnosed. Drug which does not improve function of myocardium is:A. RiboxinB. MildronaC. FosphadenD. CardonatE. *Metiluracil

110. Child was born in term. From 2 weeks sudden episodes of bluish skin from crying or feeding was observed. Clinical examination permits to suspect congenital heart disease. What are the blood

compensatory mechanisms in cyanotic defects?A. *Polycythemia, hyperhemoglobinemia, high blood viscosityB. Leucocytosis with left shift, elevation of ESRC. Hypohemoglobinemia, low blood viscosity, erythrocytopeniaD. Thrombocytosis, high blood viscosityE. Leucocytopenia, Hypohemoglobinemia

111. Clinical examination of child has revealed tachycardia, cardiomegaly, dyspnea at physical activity. Instrumental observations were prescribed. What heart defect is characterized by round, apple-shaped heart during X-ray examination?A. Fallout tetralogyB. Pulmonary stenosisC. Atrial septal defectD. Aortic stenosisE. *Tricuspid atresia

112. Child 1 month was born in term. Clinically it is observed tachycardia, arrhythmia, respiratory problems, shortness of breath, continuous machine-like murmur. Patent ductus arteriosus is diagnosed clinically. How can you characterize the patent ductus arteriosus?A. Cyanotic heart defect with right-to-left shuntB. Acquired heart diseaseC. *Acyanotic heart defect with left-to-right shuntD. Complication of the congenital heart diseaseE. Acyanotic heart defect without shunt

113. A patient, 14 yrs old, suffering from heart failure, undergoes inpatient treatment. On ECG: changes in the form of a trough-shaped displacement of ST segment below isoline and frequent ventricular extrasystoles. What is the most probable cause of these changes?A. CarditisB. Potassium overdoseC. MyocardiodystrophyD. *Cardiac glycosides overdoseE. Myocardiosclerosis

114. A 5-year-old child had an attack of palpitation with nausea, dizziness, generalized fatigue. On ECG: tachycardia with heartbeat rate of 220/min, ventricle complexes are deformed and widened, P wave is absent. What medication is to be prescribed to provide first aid?A. SeduxenB. IsoptinC. *LidocainD. NovocainamidesE. Strophantin

115. During examination of the 4 months old boy were revealed: cardiomegaly, steadfast tachyarrythmia, stagnant wheezes in lungs, dyspnea, increasing liver to 3cm. Early inborn carditis was diagnosed. What degree of the cardiac insufficiency has this child?A. 1st B. 2A C. *2-B D. 3rd E. 4th

116. During examination of 1 year child next findings were observed: mild peryoral cyanosis, respiratory rate is 47 per min., pulse rate is 144 beats/min., auscultation: normal lung sound, diminished heart sounds and rough systolic murmur on the apex and Erb’s point; hepatomegaly +2 cm, moderate edema on legs. Such clinical findings are character to:A. The I degree of heart failureB. *The II A degree of heart failureC. The II B degree of heart failure

D. The III degree of heart failureE. The 0 degree of heart failure

117. Girl 7 years in the hospital with congestive heart failure is receiving digoxin. On the fifth day of treatment there were vomiting, nausea, anorexia, headache, disturbance of color vision, liquid emptying. What is the most likely cause of this complication?A. Acute renal failureB. Exacerbation of underlying diseaseC. *Dygitalis intoxication D. Acute meningoencephalitisE. Botulism

118. A 16-year old girl with mitral stenosis after exercise there was a mixed attack of breathlessness, cough with frothy sputum release. AP is 140/95mm Hg. What did cause the deterioration of the patient?A. Left auricular acute failureB. Bronchoobstructive syndromC. Acute right ventricular failureD. Increase in total peripheral resistanceE. *Acute left ventricular failure

119. In children 1 year with fibroelastosis on the background of ARI suddenly appeared anxious, acrocyanosis, pulse 132 per minute, BR 50 per min, fine moist rales in the lower areas of lungs, pO2 60mm Hg, pCO2 55mm Hg. On X-ray: - cardiomegaly, increased pulmonary pattern, roots are as butterfly wings. Exacerbation is the result of:A. BronchiolitisB. Angioneurotic edemC. Lung abscessD. Both sides pneumoniaE. *Pulmonary edema

120. A child 12 years old suffers from rheumatic mitral heart defect. After exercise there was shortness of breath, paroxysmal cough, unproductive cough, bubbling breath. On auscultation on the background of hard breathing there are different-sized moist rales. HR - 120 per 1 minute, AP - 110/60mm Hg. What emergency state has developed in a child?A. Acute heart failure is a total typeB. Acute heart failure, right ventricular typeC. *Acute heart failure, left ventricular type D. Acute vascular insufficiencyE. Acute respiratory failure

121. A child is 13 years old. He has combined rheumatic mitral valve defects with predominance of failure. On the background of acute respiratory viral infections increased shortness of breath, general weakness, there was a feeling of tightness in the chest, a dry cough. The position is semi-sitting. There are air of long suffering on his face and cyanosis of the lips. Pulse on peripheral vascular is weak filling. Heart rate is 150 per 1 min. In the lungs on the background of hard breathing in low back areas variegated moist rales are listened. Liver is on the costal arch. What emergency state has developed in a child?A. *Acute left ventricular heart failure B. Acute right ventricular heart failureC. Total acute heart failurD. Chronic left ventricular failureE. Acute vascular insufficiency

122. The boy 12 years, suffering from mitral stenosis, after playing basketball has increasing breathing, feels a lack of air, attack and dyspnea, bubbling secreations on the lips. In the lungs (back basal areas) there are fine moist rales. The most likely cause of worsening of the child condition is associated with:A. Acute vascular insufficiency

B. Cardiac total insufficiencyC. *Cardiac left-type insufficiency D. Acute respiratory failureE. Acute heart right-type insufficiency

123. Patient 15 years with mitral heart defect is treated by digoxin (0.00025g twice daily) and furosemid (0,04g twice a week). At 10 day of treatment appeared acute heartbeat, frequent ventricular extrasystoles, nausea, vomiting. Your tactics is:A. *Cancel digoxin, prescribe potassium preparations, unitiol i/m B. Add to digoxin indomethacinC. Reduce digoxin dose in 2 timesD. Cancel digoxin, prescribe atropine subcutaneouslyE. Replace digoxin on strophantin i/v

127. Child 4yrs old complains of unpleasant feeling in heart, pain in abdomen. The boy in the maternity hospital was diagnosed congenital heart defects (ventricular septum defect. Suddenly quickening of the heartbeat at rest to 150-200 beats per minute was observed. What must parents do?A. Call ambulance cardiologic helpB. Give PhenobarbitalC. Put child in a horizontal position D. *Put child in a horizontal position and call ambulance cardiologic helpE. Press on carotid sinus

128. Child 10yrs old with acute carditis complains of discomfort in the heart, constricting pain in the chest, pain in the epigastric region, dizziness, vomiting. Child feels fear. Skin is pale, there are acrocyanosis, swelling of neck veins. Pulse is weak, heart rate can not be counting. On ECG pulse is 260 per 1 minute. What complication did occur?A. Acute left heart failure with sinus tachycardiaB. Acute right heart failure with sinus tachycardiaC. Total heart failure with sinus tachycardiaD. Ventricular paroxysmal tachycardiaE. *Supraventricular paroxysmal tachycardia

129. Child 10yrs old with acute carditis complains of discomfort in the heart, constricting pain in the chest, pain in the epigastric region, dizziness, vomiting. ECG pulse is 240 per 1 minute, complexes QRS are not changed, wave P merges with T. Put the diagnosis.A. Acute left heart failure with sinus tachycardiaB. Acute right heart failure with sinus tachycardiaC. Total heart failure with sinus tachycardiaD. Ventricular paroxysmal tachycardiaE. *Supraventricular paroxysmal tachycardia

130. Child 10yrs old with acute carditis complains of discomfort in the heart, constricting pain in the chest, pain in the epigastric region, dizziness, vmiting. ECG pulse is 180 per 1 minute, complexes QRS are changed (ventricular extrasystoles), wave P is absent. Put the diagnosis.A. Acute left heart failure with sinus tachycardiaB. Acute right heart failure with sinus tachycardiaC. Total heart failure with sinus tachycardiaD. *Ventricular paroxysmal tachycardiaE. Supraventricular paroxysmal tachycardia

131. The 12-year-old boy had a first episode of syncope in 10 years. He was treated for "family convulsant syndrome". He has a history of ventricular tachycardia and ventricular fibrillation from 4 days of age. On ECG all T-wave are abnormal; T wave in V-4 lead is particularly changed, the final part of the T wave in V-4 lead is greater in amplitude than the initial part. Put the diagnosis.A. *Congenital elongation of QT intervalB. Ventricular paroxysmal tachycardia

C. Supraventricular paroxysmal tachycardiaD. Congenital sinus tachycardiaE. Congenital carditis

132. Child 10yrs old with acute carditis complains of discomfort in the heart, constricting pain in the chest, pain in the epigastric region, dizziness, vomiting. ECG pulse is 180 per 1 minute, complexes QRS are changed (ventricular extrasystoles), wave P is absent. What medicines must be introduced intravenously first of all?A. NovocainamidB. IsoptinC. CorgluconD. *LidocainE. Potassium orotatis

133. Child 10yrs old with acute carditis complains of discomfort in the heart, constricting pain in the chest, pain in the epigastric region, dizziness, vomiting. ECG pulse is 240 per 1 minute, complexes QRS are not changed, wave P merges with T. Syndrome of premature ventricular excitation was diagnosed. What medicines must be introduced intravenously first of allA. NovocainamidB. IsoptinC. CorgluconD. LidocainE. *Cordaron

134. Child 10 yrs old with acute carditis complains of heart beating, constricting pain in the chest, fear, lack of air. ECG: early ventricular extrasystoles, superimposed on T waves preceding cardiac cycle, frequent isolated ectopic extrasystoles (more than 5 per min), and group polytopic extrasystoles. What treatment must be prescribed?A. *AmiodaronB. IsoptinC. LidocainD. PhenobarbitalE. Electric defibrillation

135. Child is 1yr old. Mother complaints of marked anxiety, refusal to eat, pale skin, tachypnea, excessive sweating, cyanosis of nasolabial triangle and vomiting. n the ECG there is the replacement of normal P wave rapid oscillations, tachycardia 160 per min. Put diagnose.A. *Atrial fibrillationB. Ventricular fibrillationsC. Ventricular paroxysmal tachycardiaD. Supraventricular paroxysmal tachycardiaE. Sinus tachycardia

136. Child 10 yrs old with acute carditis complains of discomfort in the heart, constricting pain in the chest, pain in the epigastric region, dizziness, vomiting. ECG pulse is 240 per 1 minute, complexes QRS are not changed, wave P merges with T. Put the diagnosis. Suddenly loss of consciousness, pallor cyanosis, tonic seizures occur, stop respiratory rhythm, blood pressure is not determined. What complication did occur?A. Atrial fibrillationsB. *Morgagni-Adams-Stokes syndromeC. Left heart failureD. Right heart failurE. Total heart failure

137. During procedure of taking blood in boy for general blood analysis weakness, dizziness, nausea, vomiting appear. This is accompanied by a darkening of the eyes, tinnitus with the following brief loss of consciousness. The patient falls to the floor. Skin is pale, his pupils are dilated, react to light. Extremities are cold. Breathing is shallow, bradypnea, bradycardia, weak pulse. Heart sounds are muffled. Blood pressure is lowered. Put the diagnosis.

A. *SyncopeB. Paralytic collapseC. Vagotonic collapseD. Sympatotonic collapseE. Post hemorrhagic collapse

138. Child is 6yrs old. On background of severe pneumonia there is marked pallor, cool extremities, rising of systolic blood pressure. Heart sounds are loud, tense, urine output is decreased. Patient is exiting, reflexes are increased, convulsions. Put the diagnosis.A. Acute renal failureB. Acute heart failureC. Vagotonic collapseD. *Sympatotonic collapseE. Neurotoxicosis

139. Child is 6 yrs old. On background of severe pneumonia child became adinamic, consciousness is darkened, facial features are sharp. Marbling of the skin, cold, clammy sweat, the empty veins are observed . The first heart tone is flapping, pulse is thready, blood pressure is decreased, urine output is significantly reduced. To restore vascular tone it is necessary to enter:A. PrednisolonB. *Mezaton or adrenalinC. ReopolyglucinD. Sodium ascorbinatiE. All that is mentioned

140. Child 11 yrs old was entered to the hospital with abdominal trauma. Doctor notes in patient hiccups, vomiting, persistent red dermographism, salivation, bradycardia, increased the difference between maximum and minimum blood pressure. What complication of trauma does occur?A. Paralytic collapseB. Sympatotonic collapseC. *Vagotonic collapseD. Post hemorrhagic collapseE. Toxic collaps

141. A 5-year-old patient complains of bloating and rumbling in the abdomen, increased outgoing of gases foamy liquid stool of acid odor. Symptoms appear after eating of milk products. What is the name of such symptom complex?A. *Acid dyspepsia syndromeB. Adipose dyspepsia syndromeC. Dyskinesia syndromeD. Decaying dyspepsia syndromeE. Malabsorbtion syndrome

142. A 10 year old girl complains about abdominal pain that is arising and getting worse after eating rough or spicy food. She complains also about sour eructation, heartburn, frequent constipations, headache, irritability. She has been suffering from this for 12 months. Objectively: the girl's diet is adequate. Tongue is moist with white deposit at the root. Abdomen is soft, painful in its epigastric part. What study method will help to make a diagnosis?A. Fractional examination of gastric juiceB. *EsophagogastroduodenoscopyC. Intragastral pH-metryD. Biochemical blood analysisE. Contrast roentgenoscopy

143. A 12 year old patient complains about heartburn and dull pain in the epigastrium that appear 2-3 hours after meal. Exacerbations happen in spring and in autumn. The patient has food intolerance of eggs and fish. Objectively: stomach palpation reveals painfulness in the gastroduodenal area. Esophagogastroduodenoscopy revealed a 5 mm ulcer on the anterior wall of duodenum. Urease test is positive. What is the most probable leading mechanism of disease development?

A. Dietary allergyB. Reduced prostaglandin synthesisC. Disorder of gastric motor activityD. Autoantibody productionE. *Chelicobacterial infection

144. A 15-year-old patient with “fast food” abnormal diet complains of pain in epigastrium that occurs in 1-1,5 hours after having meal. Fibrogastroduodenoscopy: marked hyperemia, small defects, and easy appearance of sores on the mucous membrane in the antral section of the stomach. What is the most probable reason for this pathologyA. *Helicobacter pylori infectionB. Presence of antibodies to parietal cellsC. Nervous overstrainD. Alimentary factorE. Toxic action of alcohol

145. A 14-year-old boy periodically complains of pain in the epigastrium on an empty stomach, nausea and heartburn during 3 years. Gastroduodenoscopy: signs of gastroduodenitis and ulcer defect of the mucous membrane of the duodenum. What is the most effective medication to treat this child?A. *De-nolB. AlmagelC. PapaveriD. No-spaE. Atropin

146. A 16-year-old female complains of heartburn and squeezing epigastric pain 1 hour after meal. She has been ill for 2 years. On palpation, there is moderate tenderness in pyloroduodenal area. Gastroscopy: antral gastritis. What study can prove the etiology of the disease?A. Detection of autoantibodies in the serumB. Gastrin level in bloodC. Examination of stomach motor functionD. Examination of stomach secretion E. *Revealing of Helicobacter infection in gastric mucosa

147. A 1-year-old child suffers of attack-like cough. The child presents with the history of dyspepsia since birth. On physical examination there are signs of delay in physical development, bronchial obstruction, respiratory insufficiency, 1 grade. Blood count: signs of inflammatory process. Sweat chlorides 120 mEq/L. What is the most likely diagnosis?A. *Cystic fibrosisB. Bronchopulmonary dysplasiaC. Kartagener's syndromeD. Acute respiratory infection, bronchitisE. Severe bronchial asthma

148. A 4-year-old child attends the kindergarten. Complaints of the bad appetite, fatigue. Objective examination: skin and mucous membrane are pale, child is asthenic. In the hemogram: hypochromic anemia 1st, eosinofilia. What is the most likely diagnosis?A. *Worm invasionB. Lymphoprolipherative processC. Duodenal ulcerD. Hypoplastic anemiaE. Atrophic gastritis

149. A patient, aged 15, complains of frequent heart-burns, air and acid eructation, burning, constringent pains behind the breast-bone, along esophagus, that appear after meals, during forward inclination of body. The patient was not examined, takes Almagel by self-medication, after the intake of which feels better. What is the most probable diagnosis?A. CardiospasmB. Duodenal ulcer

C. Functional dyspepsiaD. Stomach ulcerE. * Gastroesophageal reflux

150. A male patient complains of heartburn wich gets stronger while bending the body, substernal pain during swallowing. There is a hiatus hernia on X-ray. What disoder should be expected at gastroscopy?A. Chronic gastritisB. Gastric peptic ulcerC. *Gastroesophageal refluxD. Acute erosive gastritisE. Duodenal peptic ulcer

151. A 17 years female complains of dull pain in the right subcostal area and epigastric area, nausea, appetite decline during 6 months. There is a history of gastric peptic ulcer. On examination: weight loss, pulse is 70 per min, AP is 120/70 mm Hg. Diffuse tenderness and resistance of muscles on palpation. There is a hard lymphatic node l x l cm in size over the left clavicle. What method of investigation will be the most useful?A. *Esophagogastroduodenoscopy with biopsyB. Ultrasound examination of abdomenC. pH-metryD. Ureatic testE. Stomach X-ray

152. 4-year-old child attends kindergarten. Complaints about poor appetite, fatigue. Results: skin and mucous membranes are pale, child is malnoirished. In hemogram: hypochromic anemia 1st degree, eosinophilia. What pathology should be excluded in the first place? A. Lymphoproliferative processB. Duodenal ulcerC. Hypoplastic anemiaD. Atrophic gastritisE. *Parasitic invasion

153. The patient of 15 years complains of frequent heartburn, air and sour burping, burning, constricting pain behind the sternum along the esophagus that occur after eating, while tilting the body forward. The patient was not examined, takes almagel and then feels better. What is the most likely diagnosis?A. *Gastroesophageal reflux diseaseB. KardiospazmC. Duodenal UlcerD. Functional dyspepsiaE. Gastric ulcer

154. 17 year old girl complains of dull pain in right hypochondrium and epigastrium region, nausea, loss of appetite for 6 months. She has the history of stomach ulcer. In inspection: weight loss, heart rate 70 beats / min, BP 120/70 mm Hg. Diffuse pain and resistance of abdominal muscles during palpation. Compacted lymph nodes are palpated 1 x 1cm above the left clavicle. Which research method would be most helpful?A. *EGDS with biopsyB. UltrasoundC. pH-metryD. Urease testE. Radiograph of the stomach

155. 12-year-old boy for 2 years complains of abdominal pain, which occur at any time of the day, often at night, sometimes is accompanied by vomiting. Occult blood test is positive. The father of a boy also has frequent abdominal pain. What is the probable diagnosis?A. *Peptic ulcer diseaseB. Meckel diverticulum

C. IleusD. Parasitic infestationE. Appendicitis

156. 13-year-old boy during a month is complaining of pain in the upper abdomen. They appears at any time of the day: morning on an empty stomach, at night, after 1-1.5 hours after eating. Gregersen test is positive. He is emotionally labile. Temperature is normal. There is a tendency to constipation. The father of a boy also has frequent abdominal pain. What is the probable diagnosis?A. Meckel diverticulumB. Biliary dyskinesiaC. *Peptic ulcerD. Ulcerative colitisE. Appendicitis

157. 16-year-old boy is suffering from duodenal ulcer with increased secretion: free HCl in the fasting gastric juice is 28 title units, the basal production is 44 units, stimulate submaximal production is 68 title units. Choose the best option of treatment tactics in this case.A. *H2-histamine blockers + De-nol + solcoserilB. H2-histamine blockers + clarythromycinC. H2-histamine blockers + antispasmodic + antacidD. H2-histamine blockers + metronidazoleE. H2-histamine blockers + antacid

158. 17 years old patient turned to a local doctor complaining heaviness in the epigastric area immediately after a meal, regurgitation, a tendency to diarrhea. She is sick for 3 years. Preliminary diagnosis: chronic atrophic gastritis with secretory insufficiency. What the X-ray data confirm the diagnosis of chronic atrophic gastritis in this patient?A. *The presence of gross rigid foldsB. The presence of filling defectC. The presence of pyloric stenosisD. The local absence of contractilityE. Rapid evacuation of barium

159. Patient A. 14 yrs old is sick for about 2 years. Concerned about hunger pains in pyloroduodenal area, heartburn, sour belching. EGDS found a defect in the duodenal front wall 0.5 x0.5 cm, covered with fibrin. Antibodies to Helicobacter pylori in the blood are found. Which drug for Hp eradication should be used?A. GastrocepinB. FamotidinC. RanitidineD. *AmoxicillinE. Almagel

160. Female 12 years old within two years has chronic gastritis. Last 6 months pain in the abdomen at night appears. What is useful to examine the patient?A. Occult blood fecal testB. *EGDSC. Gastric juice pH-metryD. Ultrasonography of the abdomenE. Fractional study of gastric juice

161. A child of 10 years old complains of appetite loss, heartburn, pain around the navel, which is dull and occurs more often within 2-3 hours after meal in the morning - on an empty stomach. The pain decreases after meal. She is sick for three years. OBJECTIVE: Skin is pale. Abdomen is soft, painful during deep palpation in the epigastric and pyloroduodenal area. Mendel’s symptom is positive. What is the most likely diagnosis?A. Chronic cholecystitisB. Crohn's disease

C. *Chronic gastroduodenitisD. Peptic ulcer of the stomachE. Mesadenitis

162. A child of 11 years is hospitalized for exacerbation of chronic gastroduodenitis. EGDS diagnosed duodenogastric reflux 2nd degree. Assign a treatment for motility correction.A. *MotiliumB. ImodiumC. GastrocepinD. AlmagelE. De-nol

163. Female 10 years old has admitted to the hospital with complaints of aching night pain in the epigastrium. Palpation of the abdomen reveals pain in the epigastric and pyloroduodenal zone. What examination should be held first of all?A. Ultrasound of the abdominal cavityB. Biochemical blood test: bilirubin, cholesterol, ALT, AST, amylaseC. *EsophagogastroduodenoscopyD. Duodenal intubationE. Complete blood test

164. 10 years old boy has admitted to the clinic with duodenal ulcer relapse, associated with helicobacter pylori. Which drug is included into H. pylori eradication scheme?A. RanitidineB. MaaloxC. *AmoxicillinD. FamotidinE. Gastrocepin

165. A boy of 14 was delivered to the clinic with complaints of weakness, dizziness, nausea, "coffee grounds" vomiting. What is the most reasonable examination?A. Abdominal X-rayB. SigmoidoscopyC. *EsophagogastroduodenoscopyD. ColonoscopE. Ultrasonography of the abdomen

166. A 17 years old patient suffers from duodenal ulcer for three years. In the first day of the exacerbation period intragastric pH-metry was done before treatment. Choose the most probable results.A. *pH in the stomach body -1.5, pH in the antrum -1.5B. pH of the stomach body -2.5, pH in the antrum -5.0C. pH of the stomach body -7.5, pH in the antrum -7.5D. pH of the stomach body -5.0, pH in the antrum -2.5E. pH in the stomach body -5.5, pH in the antrum -7.5

167. 17 year old patient complains of intensive skin itching, jaundice, bone pain. The skin is hyperpigmentated. There are multiple xanthelasma palpebrae. The liver is +6 cm enlarged with acute edge. The blood analysis revealed total bilirubin 160 mkmol/L, direct — 110 mkmol/L, AST (asparate aminotransferase) — 2,1 mmol/L per hour, ALT-1,8 mmol/L, alkaline phosphotase — 4,6 mmol/L per hour, cholesterol- 9,2 mmol/L, antimitochondrial antibodies M2 in a high titer. What is the probable diagnosis?A. *Primary biliary liver cirrhosisB. Primary liver cancerC. Acute viral hepatitis ВD. Chronic viral hepatitis ВE. Alcoholic liver cirrhosis

168. A 10 year old girl complained of attacks of right subcostal pain after fatty meal she has been suffering from for a year. Last week the attacks repeated every day and became more painful.

What diagnostic study would you recommend?A. X-ray examination of the gastrointestinal tractB. Ultrasound study of the pancreasC. *Ultrasound examination of the gallbladderD. Liver function testsE. Blood cell coun

169. A 10-year-old boy is ill with autoimmune hepatitis. Blood test: A/G ratio 0,8, bilirubin — 42 mkmol/L, transaminase: ALT — 2,3 mmol/L, AST — 1,8 mmol/L. What is the most effective means in treatment?A. *Glucocorticoids, cytostaticsB. HepatoprotectorsC. Antibacterial medicationD. Hemosorbtion, vitamin therapyE. Antiviral medications

170. A 12-year-old girl complains of dull right subcostal pain, nausea, decreased appetite. History: disease started with jaundice in 2 months after appendectomy. She was treated in an infectious hospital. 1 year later present complaints have developed. Physical examination: subicteric sclerae, enlarged firm liver. What is your preliminary diagnosis?A. *Chronic viral hepatitisB. Chronic cholangitisC. Acute viral hepatitiD. Calculous cholecystitisE. Gilbert's disease

171. A 14 year-old patient was admitted to the gasteroenterology with skin itching, jaundice, discomfort in the right subcostal area, generalized weakness. On examination: skin is jaundice, traces of scratches, liver is +5 cm, splin is 6x8cm. In blood: alkaline phosphatase — 2,0 mmol/hour/L, general bilirubin — 60 mkmol/L, cholesterol — 8,0 mmol/L. What is the leading syndrome in the patient?A. CytolyticB. AsthenicC. Mesenchymal inflammatoryD. Liver-cells insufficiencyE. *Cholestatic

172. A 15 year old patient has been suffering from chronic pancreatitis for 5 years. During the last 5 years he has been observing abatement of pain syndrome, abdominal swelling, frequent defecations up to 3-4 times a day (feces are grey, glossy, with admixtures of undigested food), progressing weight loss. Change of symptom set is caused by joining of:A. Syndrome of lactase deficiencyB. Exocrine pancreatic insufficiencyC. Chronic enterocolitisD. Endocrine pancreatic insufficiencyE. *Irritable bowels syndrome

173. A 17-years old patient has sudden acute pain in the right epigastric area after having fatty food. What method of investigation is to be used on the first stage of examining the patient?A. *UltrasonicB. RadionuclideC. Magnetic-resonanceD. RoentgenologicalE. Thermographic

174. 7 years old child complains of an acute abdominal pain, which arises after mental loading, use of cold drinks, ice-cream. The diagnosis: Dyskinesia of gallbladder, hypertonic type. What drugs should be assigned first of all for treatment?A. Sedative and cholikinetics

B. Choleretics and cholikineticsC. *Spasmolitics and cholereticsD. AntioxidantsE. Antibiotics

175. The boy of 12 years complains of a periodic short-lived cutting pain in the right subcostal area, which occurs after the greasy food. For what type of dyskinesia these complaints are typical?A. *HypertonicB. HypotonicC. DystonicD. HepatalgicE. Asthenic

176. The child 12 years old complaints of the skin and mucous membranes jaundice, clay-colored feces and dark color of the urine, weakness. He is ill for 2 weeks. Jaundice has appeared on the 10th day of the disease. Three months ago had hemotransfusion because of bleeding. The liver is +3 cm, spleen +1cm. Hepatitis В is suspected. What examination will realistically confirm the diagnosis?A. The biochemical blood testB. The urinalysis on bile pigmentsC. *Polymerase Chain ReactionD. Investigation of Anti-HAV Ig ME. Investigation of Anti-HAV Ig G

177. The child, 12 years old, was treated because of the chronic hepatitis. He discharges from the hospital on the 24th day in satisfactory condition. What is the duration of dispensary observation?A. 6 monthsB. 9 monthsC. 1 yearD. 3 yearsE. *5 years

178. To the child of 5 years, who has chronic cholecystocholangitis the biochemical blood analysis is performed. Syndrome of cholestasis is detected. What parameters will be increased?A. TransaminaseB. *Alkaline phosphataseC. DiastaseD. ThrombinogenE. Thymol test

179. 14-year-old patient has admitted to the intensive care unit with hemorrhagic shock due to gastric bleeding. He has been ill hepatitis B for the last 5 years. The source of bleeding - veins of the esophagus. What is the most effective method to control bleeding?A. *Introduction of the obturator through the nasogastric tubeB. Intravenous pituitrinC. Fresh frozen plasma I/VD. OperationE. Hemostatic therapy

180. Patient complains of pain in the epigastric region, in the right hypochondrium radiating to the right scapula. It was vomiting ithout relief. The body temperature is 37.6°C. The abdomen is moderately distended, tense and painful in the epigastrium and right hypochondrium. A mild muscle strain of the abdominal wall in the right hypochondrium is palpated. Ortner’s symptom is positive. Put a preliminary diagnosis.A. Acute appendicitisB. Acute pancreatitisC. Stomach ulcer penetrationD. *Acute cholecystitisE. Acute intestinal obstruction

181. 7 years old girl is ill during 3 years. Hypotonic type of gallbladder dyskinesia was diagnosed. Which of the following is inappropriate in the complex therapy?A. *Antispasmodic drugsB. Physiotherapy of tonic typeC. CholekineticsD. Duodenal intubationE. Mineral water of high salinity

182. A 15 years old girl has gallstone disease. She has chills, increase body temperature up to 38° C, jaundice during three days. Symptoms of peritonitis are negative, the pain is not increased. Blood bilirubin is 45 mmol / l, leucocytes -18 x 10 9/L. What kind of complications should be considered?A. Perforation of the gallbladderB. *CholedocholithiasisC. Subhepatic abscessD. Acute cholangitisE. Hemolytic jaundice

183. Patient 14yrs old complained of recurrent pain in the right hypochondrium, which irradiates to the right shoulder, periodic jaundice with fever, metallic taste in the mouth. These complaints appear after overeating. OBJECTIVE: a patient is being overweight, sclera are yellowish, local tenderness in the right hypochondrium, Ortner’s, Kera’s symptoms are positive. In the blood there is the high level of direct bilirubin. What kind of diseases you can think about?A. Chronic pancreatitisB. UrolithiasisC. Gastric ulcerD. *CholelithiasisE. Hemolytic jaundice

184. Patient 16yrs old is suffering from chronic calculous cholecystitis with periodic exacerbations, as a short-term biliary colic. After colic sclera and palate icteric, darkening of urine color was observed. After the next exacerbation and examination in the hospital a surgical method of treatment was recommended. What of the following methods will be the most reliable way to exclude choledocholithiasis in this case?A. FibrogastroduodenoscopyB. LaparoscopyC. Ultrasound of the liver and biliary tractD. Duodenal intubationE. *Cholecystocholangiography

185. Patient 12yrs old is feeling heaviness in the right hypochondrium and sometimes nausea or bitterness in his mouth, a tendency to constipation. An objective examination revealed no abnormalities. At ultrasound: the liver and the pancreas are not changed; gall bladder is enlarged, hypotonic, with parietal cholestasis. At duodenal intubation: fraction B - 90 ml, the time of its excretion - 50 minutes with no changes in the bile microscopy. Which disease is possible?A. Chronic pancreatitisB. GiardiasisC. Chronic cholecystitisD. *Biliary dyskinesiaE. Chronic hepatitis

186. Patient 17yrs old was hospitalized to the surgical department with complaints of nausea, vomiting, pain in the right hypochondrium. She has been ill for the last 3 days, when there were the same complaints. Prior to this, she ate greasy and fried dishes. Objectively: the patient's state is moderate. Abdomen is soft at palpation, tender in the right hypochondrium. Grekov-Ortner’s, Kera’s signs are positive. In the blood analysis: leukocytosis 14 x 109/l, with a shift to the left. Put the correct diagnosis.A. Acute appendicitis

B. Acute pancreatitisC. Duodenal ulcerD. *Acute cholecystitisE. Stomach ulcer

187. The patient 10 years old suddenly fell ill about 12 hours ago. There was a pain in the epigastric region, nausea, single vomiting. A few hours later the pain was localized in the right iliac region, where now the positive symptoms of peritoneum irritation are determined. In the blood analysis: leukocytosis 12.2 x 109/l, with a shift to the left. What is the most likely diagnosis?A. Acute pancreatitisB. Acute cholecystitisC. Perforated ulcerD. Right-sided renal colicE. *Acute appendicitis

188. Patient 15 years old was brought by an ambulance to the pediatric department with acute pain in the right hypochondrium and vomiting. The doctor on duty has diagnosed acute calculous cholecystitis. Which of the invasive methods of radiology screening of the patient should do?A. Computed tomographyB. Radionuclide diagnosisC. ThermographyD. Magnetic resonance imagingE. *Ultrasound

189. The 10 years old boy has chronic viral hepatitis B with maximum activity. Which laboratory tests accurately characterizes the degree of cytolysis in a patient?A. ProthrombinB. Veltman’s testC. Takata-Ara’s testD. *TransaminasesE. Total protein

190. In patient 11yrsold, that for a long time has been suffering from the liver cirrhosis, has recently appeared complaints of moderate pain in the epigastric region, constant flatulence, which intensifies after meals. OBJECTIVE: Symptoms of free fluid in the abdomen, enlarged liver and spleen. At ultrasonography: extended portal vein, enlarged liver and spleen. What kind of cirrhosis complications has this patient?A. Bleeding from esophageal varicesB. Intestinal dysbacteriosisC. PeritonitisD. Hepatocellular failureE. *Portal hypertension

191. In a patient with chronic hepatitis B jaundice was growing, increased weakness and fatigue, he became drowsy during the day. The liver is uniformly decreased. Diuresis is normal. Name the further tactics of the patient’s treatment.A. Appointment of choleretic and nonsteroidal anti-inflammatory drugB. Appointment of trental and heparinC. Laferon appointmentD. *Appointment of prednisolone and increased detoxificationE. Appointment of legalon and essentiale

192. Patient 17yrs old, student, came to the clinic of the University. The last 4 days has being complained of general weakness, fatigue and impaired appetite. Skin and sclera are yellowish, brick-colored urine. In the blood analysis: hyperbilirubinemia (35 mcmol/L) with the predominance of direct bilirubin; ALT – 2.1; AST – 1.9. What is the reason of jaundice?A. Malaria, hemolytic anemiaB. Cholelithiasis, obstructive jaundiceC. Macronodular cirrhosis

D. *Viral hepatitisE. Hemolytic anemia

193. A 8 years old boy was ill with hepatitis B a year ago. In the past two months, he is complaining about fatigue, sleep disturbance, appetite loss, nausea, especially in the morning. Skin is without jaundice, liver and spleen were palpable 1cm below the costal edge, not painful. ALT is 2,2 mmol/L. This situation can be regarded as:A. *Development of chronic hepatitis BB. Recurrence of hepatitis BC. Dyskinesia of bile ductsD. Residual phenomena of acute hepatitisE. The development of cirrhosis

194. Boy 12 years old was entered to the intensive care unit in extreme severe condition. At the inspection: absent conscious, skin and sclera are yellow. Liver is enlarged, splenomegaly and ascites. There are respiratory arrythmia, tachycardia, pulse 120 per min, AP 90/40 mm Hg, hemorrhagic syndrome, erythema of palms. Laboratory results: metabolic acidosis, pH 4.2, AST 1.8 mmol/L, ALT 2.1 mmol/L, bilirubin 334.2 mcmol/L, blood serum sodium 90 mmol/L, potassium blood serum 5.9 mmol/L. The worsening of condition is due to?A. CVF III stB. Violation of cerebral circulationC. Thyreotoxic crisisD. *Hepatic comaE. Acute renal failure

195. The child 7 months has unmotivated vomiting and fever, physical development retardation, severe progressive rickets chans in the bones. The mother considers that the child is sick from the first months of life. The examination revealedA. ercalciuria and hypokaliemia. What is the most likely diagnosis?B. Phosphate diabetesC. Disease de Toni-Debre-FanconiD. *Renal tubular acidosis type E. Renal tubular acidosis type IIF. Vitamin D-resistant rickets

196. In children 1.5 years there were found O-distortion legs, muscular hypotonia. There is physical and mental retardation. Blood: hypophosphatemia, increasing the level of alkaline phosphatase. In urine: hyperphosphaturia. What is the most probable disease?A. *Phosphate-diabetesB. Disease de Toni-Debre-FanconiC. Hereditary nephritisD. Renal tubular acidosis type IE. Renal tubular acidosis type II

197. Child 1yr old has unmotivated fever, frequent vomiting, physical development retardation, rickets changes of bones. In laboratory studies: alkaline reaction of urine, lack of alkalis in the blood. What disease is most likely a child?A. Renal tubular acidosis I typeB. *Renal tubular acidosis II typeC. The hereditary nephritis D. Phosphate-diabetes E. Disease de Toni-Debre-Fanconi

198. 15 years old patient was hospitalized with straining pain in the left hypochondrium, which irradiates to the back. He notes nausea, decreased appetite, weight loss, vomiting without relief, diarrhea. He has been ill for over 5 years. Exacerbation has developed because of errors in the diet. OBJECTIVE: t ° = 37,0°C, pulse rate 94 per minute, BP 125/75 mm Hg. Skin is pale, pain in the epigastrium, right and left hypochondrium. In the blood: leuk. 10.4 x 109/l, ESR 22 mm/hour. The worsening of what disease is the most likely in this case?

A. Stomach ulcerB. Chronic gastritisC. *Chronic pancreatitisD. Chronic cholecystitisE. Chronic enterocolitis

199. In the boy 6 years old microhematuria was accidentally revealed. Objectively: stigmas dyzembriogenesis (hypertelorism, epicant, abnormal ear auricles, gothic palate). According resulys of audiogram there is detectable hearing loss. In the analysis of urine: protein 0.099g/L, 4-3 leukocytes in vision field, alkiline erythrocytes 20-22 in vision field. What disease can be suspected?A. Chronic glomerulonephritisB. *Hereditary nephritisC. Dysmetabolitic nephropathyD. Chronic pyelonephritisE. Renal tubular acidosis

200. In the girl 2 years there is severe distortion of the feet, which appeared in 1yr old. During examination: height 70cm, weight 12kg, strong body structure, O-foot deformity. On the radiograph of bones of lower extremities: diaphysis with extensive thickening of cortical layer. Phosphate- diabetes was diagnosed. What laboratory changes will help to confirm the diagnosis?A. *Hypophosphatemia, hyperphosphaturia, increased alkaline phosphataseB. Hyperphosphatemia, hypoproteinemia, hyperoxaluriaC. Hypoproteinemia, hematuria, significance proteinuriaD. Cylindruria, hyperphosphaturia, leukocyturiaE. Hypocalcemia, significance proteinuria and reduced alkaline phosphatase

201. Patient is 3 years old. Incidentally during prophylaxis observation changes in urine were found: protein - 0,75g/l, leucocytes - 4-6 in vision field, erythrocytes - 10-12 in vision field. A child was born from full-term pregnancy (the previous two ended in miscarriage). In the hospital the diagnosis of hereditary nephritis was put. What drugs should be prescribed?A. Vitamin DB. PrednisoloneC. Heparin D. *Activators metabolism (vitamins, cocarboxilazae, ATP)E. Cytostatics

202. Patient is 1.5yrs old. Mother complains of the presence of a child's fever, frequent urination and thirst. Objectively: malnutrition, pale skin and mucous membranes, bone deformations. A child does not walk and has mental retardation. Abdomen is soft, the liver is at 3cm lower the costal arch. Blood glucose is 5.4 mmol/L, blood in urine is 1.5%. What disease is the most probable in the child?A. *Disease de Toni-Debre-Fanconi B. Phosphate-diabetesC. Diabetes mellitusD. Renal tubular acidosis type IE. Renal tubular acidosis type II

203. In child 4 years old, who is being treated at orthopedic department with the expressed O-strain feet, phosphate- diabetes is diagnosed. High doses of vitamin D were appointed. Which test is necessary to control the adequacy of therapy?A. Zymnitski testB. *Sulkovich testC. UrinalysisD. Determination of urea and creatinine in bloodE. Clearance of endogenous creatinine

204. A child with disease de Toni-Debre-Fanconi has marked bone deformations, periodic pyrexia, polyuria, polydipsia, physical and mental retardation. What kind of diet should be recommended?

A. *Potato - CabbageB. Restricted protein and saltC. Fruit - sugarD. Milk and vegetableE. Hypoallergic

205. Mother of the child 3 years old comes to the doctor. She complains of child deafness. It is known that in the family was a case of deafness in the boy, who died at age 13yrs old from kidney disease. Objectively congenital dyzmorphias were revealed. What observation is necessary to conduct to confirm Alport syndrome?A. *General analysis of urineB. Excretory urographyC. Urinalysis by ZymnitskiD. Determination of blood levels of phosphorus and calciumE. Sulkovich test

206. A child 5 years old has growth retardation, rickets changes in bones, intermittent polyuria and dehydration. Ultrasound examination revealed the presence of calculus in the kidneys. In urinalysis: urine alkaline, protein - 0.033g/L, leucocytes - 15-17 in vision field, calcium oxalate in large quantities. What is the disease most likely in a child?A. Phosphate-diabetesB. The hereditary nephritisC. *Renal tubular acidosis type ID. Renal tubular acidosis type IIE. Disease de Toni-Debre-Fanconi

207. A child 5 years old with renal tubular acidosis type I has growth retardation, rickets changes in bones, intermittent polyuria and dehydration. Ultrasound examination revealed the presence of calculus in the kidneys. The first goal of therapy is to:A. Normalise calcium level in bloodB. Normalise potassium level in bloodC. Normalise endocrine balanceD. *Neutralize acid in the bloodE. All transferredF. 208.

A. a child 4 yrs old with periodic acute pain in back and abdomen, attacks of pyelonephritis despite of proper treatment, enuresis megaureter is suspected. Examination reveals hematuria and urolithiasis. What examination is needed to confirm the diagnosis?

B. *USDC. Voiding cystouretrogramD. Three glass testE. CystoscopyF. All transferred208. A child 5 years old was hospitalized for surgical treatment of glaucoma. Objectively: thickness of

nail plates, hypoplas of the proximal radial head, iliac bone with patologic of their projections , deformity of the foot (" horse toes"), flexion contracture of the joints (especially elbows). Laboratory: no changes in blood, proteinuria, microhematuria. Put the diagnosis:A. Hereditary nephritisB. Congenital tubulopathyC. Edward syndromeD. Patau syndromeE. *Syndrome nail-patella

209. A child 4 yrs old has complains of periodic acute pain in back and abdomen. During 1 year he had 3 attack of pyelonephritis despite of proper treatment, enuresis. Objectively on the left side of abdomen there is palpable abdominal tumor formation. Examination reveals hematuria and

urolithiasis. What diagnosis is suspected?A. TubulopathyB. *MegaureterC. Cystoureter refluxD. Hereditary nephritisE. Polycystic kidney

210. A 9-year-old girl is seriously ill. The illness was manifested by high fever, chills, sweating, aching pain in lumbar area, a discomfort in urination and frequent urination. Pasternatsky’s sigh is positive in both sides. On laboratory examination, WBC is 20.000/mcL; on urinalysis protein is 0.6g/L, leukocyturia, bacteriuria. Your preliminary diagnosis is:A. *Acute pyelonephritisB. Exacerbation of chronic pyelonephritisC. Acute glomerulonephritisD. Acute cystitisE. Nephrolithiasis

211. A 9-year-old girl is seriously ill. The illness was manifested by high fever, chills, sweating, aching pain in lumbar area, a discomfort in urination and frequent urination. Pasternatsky’s sigh is positive in both sides. A pain syndrome at pyelonephritis is characterized by the following, exept:A. Sense of tension in lower backB. *Acute paroxysmal pain in lower backC. Pain increases when position is changedD. Pain diminishes at warming of lower backE. Pain presents at pattering of lower back

212. A 12-year-old girl is seriously ill. The illness was manifested by high fever, chills, sweating, aching pain in lumbar area, a discomfort in urination and frequent urination. Pasternatsky’s sigh is positive in both sides. She has chronic pyelonephritis. Biochemical blood test was done. What changes can be in it?A. Increase of glucose levelB. Increase of bilirubinC. *Increase of kreatinineD. Decrease of alfa-amylaseE. Increase of alkaline phosphatase

213. At sick boy 10 years old urine was taken for analysis. Hyaline casts were found in urine. What are hyaline casts?A. Acide, that had changed the consistency in sour urineB. *Albuminous molds of kidney tubuliC. Accumulation of bacteriaD. Pressed thrombocytesE. Salt corks

214. A 14-year-old girl is seriously ill. She has acute pyelonephritis. Doctor recommends a zigzag diet. What does this diet mean?A. Alternation of salt free and sugar free daysB. Alternation of protein free and fat free productsC. Alternation of diet № 5 and diet № 15D. *Alternation of acid and alcaline productsE. Alternation of starvation and valuable feed

215. At sick girl 7 years old impairment of urine filtration was found. The illness was manifested by high fever, sweating, aching pain in lumbar area, a discomfort in urination and frequent urination. What department of nephrone filtration of urine is performed in?A. Proximal ductuleB. Interstitium of kidneyC. GlomerulusD. *Henle loop

E. Distal ductule216. Sick girl 7 years old has pyelonephritis. The illness is manifested by high fever, sweating, aching

pain in lumbar area, a discomfort in urination and frequent urination. What laboratory sign is most characteristic for this pathology?A. *Active leucocytes in urineB. CasturiaC. Considerable proteinuriaD. UraturiaE. Oxalaturia

217. Sick girl 7 years old has pyelonephritis. The illness is manifested by high fever, sweating, aching pain in lumbar area, a discomfort in urination and frequent urination. Doctor prescribed antibiotic. Choice of antibacterial preparations for pyelonephritis treatment is determined by:A. Age of the childB. Severity of illnessC. Duration of the diseaseD. *Sensitiveness of bacteria cultured from urineE. All transferred

218. During palpation of kidneys the following was revealed: it is possible to palpate the kidney, it is easly displaceble, but does not move to the opposite side of the body. Which degree of nephroptosis is presentA. IB. *IIC. IIID. IVE. Total nephroptosis

219. A 14-year-old girl is seriously ill. She has chronic pyelonephritis. Excretory urography is prescribed. This investigation does not allow discover:A. Position, sizes, form of kidneysB. Functional state of nephronsC. Functional state of urinary tractD. *Anomaly of kidney vesselsE. State of urinary bladder

220. During investigation of patient’s urine the following findings were revealed: 5-6 leucocytes are found in 1 visual field, single fresh rd corpuscles in 1 visual field. What investigation must be appointed to this patient for diagnosis clarification? A. Complete blood countB. ECGC. *Nechiporenko’s testD. Zimnitskiy’s testE. Determination of daily proteinuria

221. A 14-year-old girl is seriously ill. She has chronic pyelonephritis with frequent exacerbations. Excretory urography is prescribed to estimate:A. Anatomic state of urinary ways and urine dynamicsB. State of the kidney’s pelvis systemC. Functional ability of urinary waysD. Sizes of kidneysE. *Everything is correct

222. A patient 8 years old had attack-like pains in lumbar area, which irradiate downward. Pain syndrome was severe with dysuric sis. What disease is the most probable?A. Acute glomerulonephritisB. *UrolithiasisC. HypernephromaD. Chronic glomerulonephritis

E. Dysmetabolic nephropathy223. Sick girl 7 years old has pyelonephritis. The illness is manifested by sweating, aching pain in

lumbar area, a discomfort in urination and frequent urination. What is not typical for acute pyelonephritis?A. Beginning of illness on a background of an acute bacterial infectionB. DysuriaC. Pain in the lower backD. *Normal body temperatureE. Nausea, vomiting

224. Sick girl 12 years old has chronic pyelonephritis for 6 yrs. During exacerbation biochemical blood test is conducted. What change will confirm the renal failure first of all?A. Increase of glucose in bloodB. Increase of bilirubinC. Increase of creatinineD. *Increase of nitrogenE. Increase of alkaline phosphatase

225. Patient 15 years old is troubled with appearance of sediment in his urine which makes the urine cloudy but disappear in an hour. Appearance of what substance in urine can you suspect?A. *SaltsB. ProteinC. Bilious pigmentsD. GlucoseE. Urinary acid

226. Patient 15 years old is troubled with appearance of sediment in his urine which makes the urine cloudy and don’t disappear in an hour. Appearance of what substance in urine can you suspect?A. SaltsB. *ProteinC. Bilious pigmentsD. GlucoseE. Urinary acid

227. Patient A., 15 years old, is troubled with increased amount of urine excreted per day. During examination it was revealed that urine specific gravity is increased. Appearance of what substance in urine makes its gravity increased?A. SaltsB. ProteinC. Bilious pigmentsD. *GlucoseE. Urinary acid

228. Patient 8 years old is ill with chronic pyelonephritis for 5 years. During exacerbation biochemical blood test is conducted. What changes in biochemical blood test will prove kidney insufficiency?A. AlbuminemiaB. DysproteinemiaC. ?-lipoproteinaemiaD. *CreatininemiaE. Hyperbillirubinemia

229. Patient has renal failure. By physical examination it was revealed swelling of subcutaneous tissue of the whole body. Accumulation of liquid in subcutaneous fat tissue on the whole body is called:A. AscitesB. *AnasarcaC. HivesD. EdemaE. Renal failure

230. Patient 8 years old complains of attack-like pains in lumbar area, which irradiate downwards.

What does can this symptom testify about?A. Acute glomerulonephritisB. *UrolithiasisC. HypernephromaD. UrethritisE. Cystitis

231. Patient 7 years old entered clinic with complaints of edema under eyes, increase of body temperature up to 37.8°С, discoloration of urination. 2 weeks ago he had tonsillitis. What test must be done first of all?A. Estimation of ASL-OB. *General analysis of urineC. Ultrasound of abdomenD. Bacteriologic study of pharynx smashE. All transferred

232. At sick boy 10 years old with pyelonephritis urine was taken for analysis. The analysis of urine in this case is not characterized by:A. *Proteinuria more than 1 g/lB. Neutrophils leukocyturiaC. Plenty of cellular epitheliumD. Sometimes plenty of saltsE. The specific gravity of urine is normal

233. The child of 10 years is ill during one week. The disease appears after cooling. Her disease symptoms are: pain in abdomen, back, body temperature 38°С. In urine analysis: leucocytes – 25-30 in the visual field, protein – 0.33 g/l. Diagnose is acute pyelonephritis. What investigation is it necessary to do before prescribing the etiotropic treatment?A. CystographyaB. UrogrphyaC. Zimnitskiy testD. Nechiporenko testE. *Bacteriological test of urine

234. The child of 10 years complains of high temperature, pain in abdomen, and pain during urination. In urinalysis: protein – 0.33 g/l, leucocytes – in all visual field, erythrocytes 5-10 in visual field. What investigation is it necessary to do to know the level of urinary system damaging?A. *UrographyaB. To take urine by catheterC. Bacteriological test of urineD. Ultrasound investigationE. Zimnitskiy test

235. The child of 5 years is ill for 2 days. He complains of: often pain during urination, urine incontinence. Temperature is normal, abdomen is painful in hypogastrium. What dates are typical to diagnose urinary tract infectioA. ProteinuriaB. LeucocyturiaC. ErythrocyturiaD. *Bacteriuria 105 and moreE. Hypostenuria

236. The child of 9 years become ill acutely: temperature is 39oС, pain in lower back, and pain during urination. Pasternatskiy symptom is positive bylaterally, more in the left. What disease has this child?A. *Acute pyelonephritisB. Acute viral infectionC. Acute cystitisD. Acute glomerulonephritis

E. Kidney colic237. The girl of 3 years has high temperature, pain in abdomen the third time during last year. In

urinalysis: leucocytes – 70-80 in visual field, erythrocytes 1-2 in visual field. What investigation will prove the genesis of leucocyturia?A. UrographyB. *3 glasses testC. Nechiporenko testD. Endogenous creatynine clearanceE. Zimnitskiy test

238. The girl of 6 years complains of temperature 39°С, vomiting, pain in abdomen, disuria. In urinalysis: protein – 0.58 g/l, leucocytes – in all visual field, erythrocytes 4-5 in visual field. Blood test: ESR – 30 mm/hour. Diagnose is acute pyelonephritis. What investigation will prove the diagnosis?A. *Bacteriological test of urineB. Zimnitskiy testC. Nechiporenko testD. Blood ureaE. Endogenous creatynine clearance

239. The girl of 10 years complains of the pain in lower back during 4 days. She has also the increased temperature, decrease of appetite, and the pain during urination. One week ago was ill (acute viral infection). What investigation is it necessary to make first of all?A. *General analysis of urineB. General analysis of bloodC. UrogrphyD. Ultrasound investigationE. Zimnitski test

240. Mother of a girl 7 years old complained of recurrent abdominal pain and skin rash, increased sweating, decrease in urine output and concentrated character. Nocturia is noted. AP is 90/60mm Hg. General urine analysis: the relative gravity of urine - 1028, protein - 0,04g/l, Leuc. - 9-10 in v/f, Eryth. – changed 6-8 in v/f, casts - not detected, salts - oxalates large number. Set a preliminary diagnosis.A. *Dysmetabolic nephropathyB. Acute glomerulonephritis with nephritic syndromeC. Urinary infectionD. TubulopathyE. Acute renal failure

241. Patient 14yrs old complained of intense pain in the right lumbar region, chills, accompanied by fever up to 39°C. The abdomen is soft, painful in the right area. Palpation of right kidney is painful. In the blood: leukocytes 30.0 x109/L, ESR - 50 mm/hour. In urine an.: acid reaction, leukocytes in the entire field of vision. According to the US - the left kidney is normal, the contours of the right kidney are increased. What disease is the most probable in this case?A. Right paranephritisB. *Acute right-sided pyelonephritisC. Tuberculosis of the right kidneyD. Swelling of the right kidneyE. Polycystic kidney degeneration

242. A child 13 years old complains of pain in the suprapubic region, frequent urination in small quantities of urine. Fever is 37.7°C. In the analysis of urine: proteinuria 0.03g/L, fresh erythrocytesin all vision field, salt-oxalate in small amount. What is the most likely diagnosis?A. Dysmetabolic nephropathyB. *Acute cystitisC. Acute glomerulonephritisD. Acute pyelonephritis

E. Urolithiasis243. Boy aged 1 month was hospitalized. Prenatally left-sided pyeloectasis was diagnosed. Intravenous

urography, cystography and ultrasound revealed hydronephrosis in child an early stage. Data about secondary pyelonephritis are absent. What tactics is appropriate in this patient?A. Observation for 6 monthsB. Observation during the yearC. Antibacterial therapyD. *SurgeryE. No need for supervision and treatment

244. In child 3 years with periodic abdominal pain and pyuria ultrasound revealed an enlarged kidney and rounded shape of the cavities, which are connected with an pelvis. Right kidney is not altered. What diagnosis is correcA. MulticystosisB. HydronephrosisC. EchinococcusisD. Kidney stoneE. *Polycystic kidney

245. The girl 4 years old is suffering from atopic dermatitis, occasionally abdominal pain disturbed. Palpation: abdomen is soft and painless. Liver is 2 cm below the costal arch. Stools and urine are normal. Urinalysis - muddy, urine pH 7.0, protein 0.05g/L, red blood cells modified and unmodified - 30-40 in v/f, white blood cells - 6-8 in v/f, salts - oxalates increased amount. Daily proteinuria is 0,03g. Put preliminary diagnosis.A. Allergic nephropathyB. PyelonephritisC. *Dysmetabolic nephropathyD. GlomerulonephritisE. Hemorrhagic cystitis

246. In the patient 13 years old with normal body mass on clinical examination of the urine calcium salts of phosphoric acid were revealed. His diet consists of rye and wheat bread, pasta, butter, boiled meat, fried fish, mashed potatoes, milk, cheese, coffee, tea, broth rose, currant jelly. Energy in diet corresponds to power consumption. What is necessary to eliminate from the diet?A. Milk and cheeseB. *Coffee and TeC. Pasta and breadD. Broth hips and jellyE. Meat and fish

247. For a boy 7 years in 2 weeks after the carried tonsillitis edema appeared on face, lower extremities. The state is heavy, predefined by intoxication syndrome. Blood pressure - 140/80. Urine has brown color. General analysis of urine: specific gravity -1015, protein – 1,2 g/l, red corpuscles cover all field of view, cylinders 1 – 2 in field of view. Protein in day's urine – 0,78 gr. What most reliable diagnosis?A. Acute glomerulonephritis with nefrotic syndromeB. NephrolithiasisC. Acute glomerulonephritis with nefrotic syndrome, hematuria and arterial hypertensionD. *Acute glomerulonephritis with nephritic syndromeE. Acute glomerulonephritis with isolated urine syndrome

248. The boy of 3 has an edema syndrome like as anasarca. Blood pressure - 95/60. In the general analysis of urine: protein – 6,3 g\l, leucocytes 2-3 in field of view, red corpuscles 1-2 in field of view, cylinders - 2-3 in field of view. General protein of blood – 44,2 g\l, albumen – 38,1%, cholesterol of blood – 8,6 mmol\l. What clinical variant of acute glomerulonephritis does take place probably?A. Acute glomerulonephritis with nephritic syndromeB. *Acute glomerulonephritis with nefrotic syndrome

C. Acute glomerulonephritis with isolated urine syndromeD. Acute glomerulonephritis with nefrotic syndrome, hematuria and arterial hypertensionE. Acute glomerulonephritis with hematuria

249. The girl of 8 years has complaints to pain in back during 4 days. She has also the increase temperature, decrease of appetite, and the pain during urination. One week ago was acute viral infection. What investigation it is necessary to make first of all?A. *General analysis of urine B. General analysis of bloodC. UrogrphiaD. Ultrasound investigatioE. Zimnitskiy test

250. The child of 10 years is ill during one week. The disease appears after cooling. The symptoms: pain in abdomen, back, temperature 38оС. In analysis of urine: leucocytes – 25-30 in field of view, protein – 0,33 g/l. Diagnose – acute pielonephritis. What investigation it is necessary to prescribe for making of etiotropic treatmentA. CistographiaB. UrogrphiaC. Zimnitskiy testD. Nechiporenko testE. *Bacteriological test of urine

251. The child of 8 years ill acute: temperature 39С, pain in back, and the pain during urination, symptom of Pasternatskiy positive from both sides, more in left. What disease is present?A. *Acute pielonephritisB. Acute viral infection. C. Acute cystitisD. Acute glomerulonephritisE. Kidney colic

252. Acute glomerulonephritis is diagnosed. What from the resulted investigations is informing for kidneys function estimation?A. General analysis of urineB. Protein estimation in day’s urineC. Nechiporenko testD. *Zimnitskiy testE. Estimation of daily diuresis

253. The child of 10 years. The complaints increase of temperature, pain in abdomen, and pain during urination. In analysis of urine: protein – 0,33 g/l, leucocytes – in all field of view, erythrocytes 5-10 in field of view. What investigation it is necessary to prescribe to know the level of damaging of urinary system?A. *UrographiaB. To take urine by catheterC. Bacteriological test of urine D. Ultrasound investigationE. Zimnitskiy test

254. The child of 5 years. The 2-day of disease. The complaints: often pain during urination, incontinence. Temperature normal, pain in abdomen in lower part. What dates is typical to diagnose: infection of urinary ways?A. Proteinuria B. LeucocyturiC. ErytrocyturiaD. *Bacteruria 10^5 or moreE. Hypostenuria

255. The girl of 6 years has temperature 39оС, vomiting, pain in abdomen, troubled urine. In analysis of urine: protein – 0,58 g/l, leucocytes – in all field of view, erythrocytes 4-5 in field of view.

Analysis of: ESR – 30 mm/hour. Diagnose – acute pielonephritis. What investigation it is necessary to make to prove the diagnose.A. *Bacteriological test of urine B. Zimnitskiy tesC. Nechiporenko testD. Urine of bloodE. Endogenic creatinin clirens

256. The girl 3 years the third time during the last year has the complaints to the high temperature, pain in abdomen. In analysis of urine: leucocytes – 70-80 in field of view, erythrocytes 1-2 in field of view. What investigation it is necessary to make to know the genesis of leucocyturia?A. UrogrphiaB. *CistographiC. Nechiporenko testD. Uria of blood and creatininE. Zimnitskiy test

257. What from the resulted symptoms is the criterion of acute glomerulonephritis, nephritic variant?A. *Hematuria B. LeucocyturiaC. Bacteruria D. AnascaE. Proteinuria more than 3 gr per day

258. A 10 years old boy has acute glomerulonephritis during a mouth. He has edema. In urine: protein – 2,5 g/l, in the biochemical blood test: total protein – 48 g/l, cholesterol- 9,8 mmol|l. What from this medicine must be appointed to the child in the complex of pathogenetical therapy?A. DelagylB. PlaquenylC. *PrednisolonD. HeparinE. Curantil

259. Dark color of urine is observed at a 9 years boy, that 3 weeks ago had an tonsillitis, arterial blood presser 100|50, in the complete analysis of urine: protein is 0,98 g|l, leucocytes 3-2 in field of view, erythrocytes on 1/3 in field of view, hyaline casts 1-2 in field of view. What variant of acute glomerulonephitis is more reliable in the child? A. NefroticB. NephriticC. *Isolated urine syndromeD. Nefrotic syndrome, hematuria and arterial hypertensionE. Subacute malignant glomerulonephritis

260. The 7 years boy with edema, headache, red urine, blood pressure 130/90. The 10 day before were pain in throat, hypothermia. In general analysis of urine: protein-2,5 g/l, leuc. – 2-3, erythr. On all field of view, casts gialine 2-3. What can lead to such changes?A. *StreptococciB. EnterovirusesC. Respiratory virusesD. StaphylococciE. Coli

261. The girl of 10 years. One month before was tonsillitis. Last 2 weeks is observed general malaise, painless of skin; urine specific gravity – 1018, protein 0.91 g/l, leukocytes in urine –2-3, erythr. - 1/2 of field of view. Blood pressure – 140/90. What drug is necessary to give:A. *IndometacinB. PrednisoloneC. ChlorbutinD. Plaquenil

E. Methothrexate262. A 7 yrs old child had elevation of temperature t° to 40°C in anamnesis. For the last 3 months he

presents fusiform swelling of fingers, ankle joints and knee joints, pain in the upper part of the sternum and cervical part of the spinal column. What is the most probable diagnosis?A. Septic arthritisB. *Juvenile rheumatoid arthritisC. Toxic synovitisD. RheumatismE. Osteoarthritis

263. A 14 year old female fell ill 3 months ago after cold exposure. She complained of pain in her hand and knee joints, morning stiffness and fever up to 38oC. Interphalangeal, metacarpophalangeal and knee joints are swollen, hot, with reduced ranges of motions; ESR of 45 mm/h, CRP (+++), RF (+). What group of medicines would you recommend to the patient?A. SulfonamidesB. TetracyclinesC. FluorchinolonesD. CephalosporinesE. *Nonsteroid anti-inflammatory drugs

264. A 4 years old girl was hospitalized with complaints of pain and swelling in the right knee and an ankle joints, morning stiffness, rapid fatigue, subfebrile temperature. She is ill for 4 months. Beginning of illness she connects with ARI. The disease began with a knee violation. She received aspirin, but the effect was absent. After 3 months the process has spread to the radiocarpal joint. Put a preliminary diagnosis.A. Rheumatic feverB. Infectious-allergic arthritisC. Systemic lupus erythematosisD. *Rheumatoid arthritisE. Systemic scleroderma

265. A 6 years old girl with eye problems was consulted by ophthalmologist. Diagnosis of uveitis was established. There are complaints of pain and swelling in the right knee and a ankle joints, rapid fatigue, subfebrile temperature. She is ill for 4 months. Affection of the eyes is special for:A. Infectious arthritisB. Acute rheumatic leverC. *Juvenile rheumathoid arthritisD. Overuse syndromeE. Osteomyelitis

266. 3 yrs old girl has fever, rash, arthritis and signs of inflammation of internal organs. Systemic form of the juvenile rheumatoid arthritis was suspected. It is characterized with: A. *Evanescent salmon-pink macular rash B. Chronic pain and swelling of many joints in a symmetric fashion C. Chronic asymmetric arthritis of large jointsD. Purpuric skin rash E. Hemarthrosis

267. Girl is 8 years old. Complaints: the general malaise, periodic arthralgia, tingling sensation in fingers, spotty rash on her face. She is ill during the year. Beginning of illness mother connects with the rest in the summer in the south. Objectively: integuments and visible mucous membranes are pale, on the face, neck, palms there is expressed capillaritis, weakness of muscles. Joints are not changed. Cardiac tones are rhythmic, weakened, delicate dyastolic murmur on the apex, HR is 100 per min.A. r previous diagnosis is:B. *Raynaud's SyndromeC. Systemic lupus erythematosisD. Nodular peryartheriitis

E. Atopic dermatitisF. Rheumatic fever

268. A 8 years old girl was hospitalized with complaints of pain and swelling in the left knee and left ankle joints, morning stiffness, rapid fatigue, subfebrile temperature. She is ill for 4 months. Beginning of illness she connects with ARI. Pauciarticular form of the juvenile rheumatoid arthritis is characterized by: A. *Chronic asymmetric arthritis of large jointsB. HepatosplenomegalyC. Salmon-pink macular rashesD. Purpuric skin rashesE. Hemarthrosis

269. The 14-years old girl has the complaints of presence of white colour patches on the wrists, legs and face. These patches are cold, firm, thick and dry. The movements of wrists joints are not limited. From the anamnesis is known that she had signs of Raynaud's syndrome the year before and there is decreased sensitivity of hands and feet now. No changes in the inner organs were founded. What observation is the most helpful to confirm the diagnosis?A. ANA testB. *Skin biopsyC. MRID. AngiographyE. General blood analysis

270. The 14-years old girl has the complaints of presence of white colour patches on the wrists, legs and face. These patches are cold, firm, thick and dry. The movements of wrists joints are not limited. From the anamnesis is known that she had signs of Raynaud's syndrome the year before and there is decreased sensitivity of hands and feet now. No changes in the inner organs were founded. Localized scleroderma was diagnosed. What treatment would be prescribed?A. PenicillamineB. PentoxifyllineC. CyclophosphamidD. PhysiotherapyE. *All mentioned above

271. The 8 years old boy complaints of body temperature 37.4 – 37.8?C, muscle weakness, pain in knees and legs. These symptoms appeared 8 days ago. During examination was noticed violet-colored rash on eyelids and around the nails, periorbital edema. Palpation of the shin muscles is painful, movements of legs are limited. There is hyperemia of the oral cavity, multiple ulcers on the gingiva. No changes in the inner organs were found. What observation is the most helpful to confirm the diagnosis?A. *Skin and muscle biopsyB. ANA testC. MRID. Serum muscle enzymes (CK,CPK, LDH, and/or aldolase)E. General blood analysis

272. A boy 10 years old has complaints of significant fatigue, fever, dysphagia, hardness in flexion of wrist fingers. He is ill during months; his mom thinks that his disease is provoked with overcooling. During examination: the weight of patient is less than normal, there are firm small tubercles under the skin and in the muscles. The movements in hand joints are painless and decreased. The oral cavity mucosa is dry with hemorrhages. Three months ago in this boy was diagnosed stomach ulcer. Put the most probable diagnosis.A. Atopic gingivatisB. Systemic vasculitisC. Myastenia gravisD. PolyneuropathyE. *Juvenile dermatomyositis

273. The 16-years old boy has the complaints of skin discoloration on the face and feeling of “hard skin” around the mouth. Also there is swelling of wrists joints and limited but painless movements in these joints. During examination there were registered dull heart sounds, tachycardia, decreased sensitivity in the hands and feet. In ultrasound examination was diagnosed heart fibrosis. Put the diagnosis.A. *Scleroderma systemicaB. JRAC. JDMD. Polyarteritis nodosaE. Restrictic cardiomyopathy

274. The 16-years old boy has the complaints of skin discoloration on the face and feeling of “hard skin” around the mouth. Also there is swelling of wrists joints and limited but painless movements in these joints. During examination there were registered dull heart sounds, tachycardia, decreased sensitivity in the hands and feet. In ultrasound examination was diagnosed heart fibrosis. What observation is the most helpful to confirm the diagnosis?A. General blood analysis, ANA testB. MRI, CTC. *Skin biopsyD. EchoCGE. ECG

275. Patient 10 years old admitted to the hospital with intermittent high fever, allergic rash, pain and swelling in the knee and ankle joints, increase of peripheral lymph nodes, liver and spleen. In general blood analysis - leukocytes 27x109/l, ESR - 65mm/hour, increased immunoglobulin "M" and "G". Which of the following diagnoses is most probable?A. SepsisB. *Systemic juvenile rheumatoid arthritiC. Systemic lupus erythematosusD. Rheumatc feverE. Leukemia

276. The 17-years old girl came to hospital because of dysphagia symptoms. According anamnesis data was revealed abnormal sensitivity to cold in the hands and feet. During examination were found: painful calcium deposits under the skin, pallor and cyanosis of hands and feet, tightening of the skin on the fingers or toes and presence of dilated capillaries on the hands and face. Put the diagnosis.A. Systemic vasculitisB. Polyarteritis nodosaC. Localized sclerodermaD. *Progressive Systemic SclerosisE. Raynaud's syndrome

277. The 13 years old girl complaints of body temperature 37.8 – 38.2?C, muscle weakness, pain in knees and legs. These symptoms appeared 4 days ago. During examination was noticed violet-colored rash on eyelids and around the nails, periorbital edema. Palpation of the shin muscles is painful, movements of legs are limited. There is hyperemia of the oral cavity, multiple ulcers on the gingiva. No changes in the inner organs were found. Put the diagnosis.A. Atopic gingivatisB. Systemic vasculitis

C. Myastenia gravisD. *Juvenile dermatomyositisE. Polyneuropathy

278. The 12-years old girl has the complaints of presence of white colour bands on the arms and legs. These bands are firm, thick and dry. The girl describes the feelings at the areas of lesions presence like “tightening” or “compression”. From the anamnesis is known that she had signs of Raynaud's

syndrome the year before and there is decreased sensitivity of hands and feet now. No changes in the inner organs were founded. Put the diagnosis.A. Atopic dermatitisB. Systemic vasculitisC. PolyneuropathyD. Obliterised endarteriitisE. *Localized scleroderma

279. A boy 5 years old was hospitalized because of pain in the neck, knees and decrease of movements in these joints especially in the morning. Two weeks ago was ARI with tonsillitis. The disease have acute onset: hyperthermia, hard movements of the head, pain and edema of joints. After anti-inflammatory drugs the pain became less intensive, but decreasing of active movements still remained. During examination: skin pallor, deformation of knee joints with decrease movements in them and neck. What examination will help to put early diagnosis of JRA?A. Coombs testB. ESR, CRPC. Pheumatoid factorD. *Biopsy of synovial membraneE. US of joints

280. During investigation of patient’s urine the folowing findings were revealed: 5-6 leucocytes are found in 1 visual field, single fresh red corpuscles in 1 visual field. What investigation must be appointed to this patient for diagnosis clarification? A. Complete blood countB. ECGC. Zimnitsky’s testD. determination of daily proteinuria.E. * Nechiporenko’s test

281. During palpation of kidneys the following was revealed: it is possible to palpate all the kidney, it is easly displaceble, but does not move to the opposite side of the body. Which degree of nephroptosis is present?A. 1st B. 3rd C. 4th D. total nephroptosis.E. * 2nd

282. For patient E., 12 years old, ill with chronic pyelonephritis for 6 yers, biochemical blood test is conducted. What changes can be when the patient has renal failure? A. increase of glucose level in bloodB. increase of bilirubinC. increase of amylazeD. increase of alkaline phosphatase.E. * increase of creatinine

283. For patient I., 14 years old, paranephritis is diagnosed. What position is typical for this case?

A. Semirecumbent position with lowered legs (orthopnoe)B. Lying on a sick sideC. Knee-elbow positionD. Sitting, bending forward.E. * On affected side with legs bended hip and knee joints and by the leg pressed to the

stomach from the side of affection284. In patient A., violation of kidneys concentration function was found. What part of nephrone

concentrates the urine? A. glomerulusB. Henle’s loop

C. proximal tubuleD. convoluted tubes.E. * distal tubule

285. Patient E. 8 years old., is ill with chronic pyelonephritis for 5 years. What changes in biochemical blood test will prove kidney insufficiency? A. albuminemiaB. beta-lipoproteinaemiaC. hyperbilirubinemiaD. dysproteinemia.E. * creatininemia

286. Patient E., 8 years old, complains of attack-like pains in lumbar area, which irradiate downwards. What does can this symptom testify about?A. acute glomerulonephritisB. hypernephromaC. chronic glomerulonephritisD. heart attack.E. * urolithiasis

287. The child of 10 years complains of high temperature, pain in abdomen, and pain during urination. In urinalysis: protein – 0,33 g/l, leucocytes – in all visual field, erythrocytes 5-10 in visual field. What investigation is it necessary to do to know the level of urinary system damaging? A. To take urine by catheterB. Bacteriological test of urine C. Ultrasound investigation D. Zimnitsky testE. * Urographia

288. The child of 10 years is ill during one week. The disease appears after cooling. Her disease symptoms are: pain in abdomen, back, temperature 38оС. In analysis of theurine: leucocytes – 25-30 in the visual field, protein – 0,33 g/l. Diagnose is – acute pyelonephritis. What investigation is it necessary to do before etiotropic treatment? A. CystographiaB. UrogrphiaC. Zimnitsky testD. Nechiporenko testE. * Bacteriological test of urine

289. The child of 5 years is ill for 2 days. He complains of: often pain during urination, urine incontinence. Temperature is normal, abdomen is painful in hypogastrium. What dates are typical to diagnose urinary tract infection?A. Proteinuria B. LeucocyturiaC. ErytrocyturiaD. HypostenuriaE. * Bacteruria 105 and more

290. The child of 9 years become ill acutely: temperature is 39oС, pain in lower back, and pain during urination, Pasternatskiy symptom is positive bylaterally, more in the left. What disease has this child? A. Acute viral infection. B. Acute cystitisC. Acute glomerulonephritisD. Kidney colic E. * Acute pyelonephritis

291. The girl of 10 years complains of the pain in lower back during 4 days. She has also the increased temperature, decrease of appetite, and the pain during urination. One week ago was ill (acute viral infection). What investigation it is necessary to make first of all?

A. General analysis of bloodB. UrogrphiaC. Ultrasound investigationD. Zimnitsky testE. * General analysis of urine

292. The girl of 3 years has high temperature, pain in abdomen the third time during last year. In urinalysis: leucocytes – 70-80 in visual field, erythrocytes 1-2 in visual field. What investigation will prove the genesis of leucocyturia? A. UrographyB. Nechiporenko testC. Endogenous creatynine clearanceD. Zimnitsky testE. * 3 glasses test

293. The girl of 6 years complains of temperature 39о С, vomiting, pain in abdomen, disuria. In urinalysis: protein – 0,58 g/l, leucocytes – in all visual field, erythrocytes 4-5 in visual field. Blood test: ESR – 30 mm/hour. Diagnose is acute pyelonephritis. What investigation will prove the diagnose? A. Zimnitsky testB. Nechiporenko testC. Blood urea D. Endogenous creatynine clearanceE. * Bacteriological test of urine

294. A 14-year-old boy has rheumatism. Over the last 2 years he has had 3 rheumatic attacks. What course of rheumatism does the patient have?A. Persistent-reccurentB. SubacuteC. AcuteD. LatentE. *Prolonged

295. After supercooling a 15-year-old girl developed muscle pain, body temperature rise up to 39oC, headache, dysuria, positive Pasternatsky's symptome. In the urine: leukocyturia, bacteriuria. In blood: decrease in Hb rate down to 103 g/l, left shift leukocytosis, ESR acceleration up to 32 mm/h. Blood urea - 6,0 millimole/l. What is the most likely diagnosis?A. Acute cystitisB. Acute glomerulonephritisC. UrolithiasisD. *Acute pyelonephritisE. Renal tuberclosis

296. An 18-month-old child was taken to a hospital on the 4-th day of the disease. The disease began acutely with temperature 39, weakness, cough, breathlessness. He is pale, cyanotic, has had febrile temperature for over 3 days. There are crepitative fine bubbling rales on auscultation. Percussion sound is shortened in the right infrascapular region. X-ray picture shows non-homogeneous segment infiltration 8-10 mm on the right, the intensification of lung pattern. Your diagnosis:A. GrippeB. Interstitial pneumonia C. *Segmentary pneumonia D. BronchitisE. Bronchiolitis

297. A 27-year-old patient with a history of bronchial asthma was stung by a bee. He had a sensation of chest compression, breath shortage, difficult expiration, sense of heat in the upper half of body, dizziness, apparent itch, convulsions. Objectively: noisy wheezing breath, AP - 90/60 mm Hg, Ps- 110 bpm. Auscultation revealed weak rhythmic heart sounds, rough respiration above

lungs, sibilant rales. What drug group should be administered in the first place?A. AnticonvulsiveB. AnalgeticsC. *GlucocorticoidsD. Cardiac glycosidesE. Methylxanthines

298. A pediatrician had a conversation with a mother of a 7-month-old breast-fed boy and found out that the child was fed 7 times a day. How many times should the child of such age be fed?A. 7 times B. 3 times C. 6 times D. *5 times E. 4 times

299. 2 weeks after recovering from angina an 8-year-old boy developed edemata of face and lower limbs. Objectively: the patient is in grave condition, AP- 120/80 mm Hg. Urine is of dark brown colour. Oliguria is present. On urine analysis: relative density -1,015, protein - 1,2 g/l, RBCs are leached and cover the whole vision field, granular casts - 1-2 in the vision field, salts are represented by urates (big number). What is the most likely diagnosis?A. *Acute glomerulonephritis with nephritic syndromeB. NephrolithiasisC. Acute glomerulonephritis with nephrotic syndromeD. Acute glomerulonephritis with nephrotic syndrome, hematuria and hypertensionE. Acute glomerulonephritis with isolated urinary syndrome

300. 79 A 16-year-old girl complains of nasal blockage, sneezing, watery nasal discharges. The body temperature is normal. Objectively - edema of nasal mucous membrane on both sides, especially of the lower turbinate (with cyanosis). Blood test data - mild leukocytosis and eosinophilia. What is the diagnosis?A. *Allergic rhinitis B. Acute sinusitis C. Infective rhinitisD. Foreign body in the noseE. Atrophic rhinitis

301. In autumn a 15-year-old patient developed stomach ache that arose 1,5-2 hours after having meals and at night. He complains about pyrosis and constipation. The pain is getting worse after consuming spicy, salty and sour food, it can be relieved by means of soda and hot-water bag. The patient has been suffering from this disease for a year. Objectively: furred moist tongue. Abdomen palpation reveals epigastrial pain on the right, resistance of abdominal muscles in the same region. What is the most likely diagnosis?A. Stomach ulcerB. Chronic pancreatitis C. Diaphragmatic hernia D. Chronic cholecystitis E. *Duodenal ulcer

302. A boy, aged 14, presents with facial edema, moderate back pains, body temperature of 37,5oC, BP- 150/100 mm Hg, hematuria (up to 100 in v/f), proteinuria (2,0 g/l), hyaline casts - 10 in v/f, specific gravity - 1020. The onset of the disease is probably connected with acute tonsillitis 2 weeks ago. The most likely diagnosis is:A. UrolithiasisB. Chronic glomerulonephritis C. *Acute glomerulonephritis D. Cancer of the kidneyE. Acute pyelonephritis

303. A 16-year-old boy with a history of rheumatic fever complains of fever up to 38-39oC, abdominal

pain, dyspnea, tachycardia. Heart borders are displaced to the left by 2 cm, systolic and diastolic murmurs above aorta, BP of 140/30 mm Hg. Petechial rash occurs after measurement of blood pressure. Liver is enlarged by 3 cm, spleen is palpable. Urine is brown-yellow. What is the most likely diagnosis?A. Rheumatic feverB. * Infectious endocarditisC. Acute nephritisD. Acute hepatitisE. Aortic regurgitation

304. A boy is 8 year old. His physical development is compliant with his age. The child has had cardiac murmur since birth. Objectively: skin and visible mucous membranes are of normal colour. AP- 100/70 mm Hg. Auscultation revealed systolo- diastolic murmur and diastolic shock above the pulmonary artery. ECG shows overload of the left heart. Roentgenoscopy shows coarsening of the lung pattern, heart shadow of normal form. What is the most likely diagnosis?A. Fallot's tetradB. Pulmonary artery stenosisC. * Atrio-septal defectD. Aorta coarctationE. Patent ductus arteriosus

305. The patient with quired heart failure has diastolic pressure of 0 mm Hg. What heart failure does the child haveA. Mitral stenosisB. Mitral insufficiencyC. Aortal stenosisD. * Aortal insufficiencyE. Rheumatism

306. ?The child is 7 months, he has artificial feeding (cow's milk, cream of wheat). He entered the hospital with fever up to 37,8 ? C, brief bouts of tonic-clonic seizures, signs of rickets 2 degree. Positive Erb, Trousseau, Maslov symptoms. What is the preliminary diagnosis? A. Renal eclampsia B. Hyperthermia C. Epilepsy D. Meningoencephalitis E. * Spasmophylia

307. The child is 1,5 months. He was born premature with a weight of 2000. During pregnancy the mother suffered from preeclampsia 1st and 2nd half of pregnancy. 2 weeks to the increased sweating, excitability. Which pathological condition is the most likely in this child? A. The consequences of perinatal lesions of the nervous system B. Functional disorders of the gastro-intestinal system C. Spasmophylia D. The initial manifestation of ARVI E. * Rickets

308. The child is 11 months. He is hospitalized because of spasmophylia as evidenced tetany. After the emergency treatment of the child health has improved, convulsions ceased. Determine further therapeutic tacticsA. Assign vitamin D in doses of 2000 IU a week supplementation with calcium. B. Assign vitamin D in doses of 4000 IU once. C. Do not assign vitamin DD. Assign vitamin D in doses of 500 IU immediately E. * Assign vitamin D in doses of 500 IU a week supplementation with calcium.

309. The child is 3 months. An objective examination of observed pallor of the skin, excessive sweating, anxiety, palpation - soft edge of a large fontanel, softening of the occipital bone. What is the course of the disease?

A. Subacute B. Recurrent C. Latent D. Limp E. * Acute

310. The child is 3 months. An objective examination has observed pallor of the skin, excessive sweating, anxiety, palpation - soft edges of a large fontanel, softening of the occipital bone. What are your recommendations. A. nalidixic acid and calcium glycerophosphate B. videin-3 in 3 days C. ultraviolet irradiation 20 sessions D. vitamin D 400 IU per day. E. * videin-3 and calcium glycerophosphate

311. The child is 6 months. Located on breastfeeding, prevention of rickets was made. Fruit and vegetable products, fruit juice does not get. His mother said that child has anxiety, increased sweating. An objective examination: severe frontal and parietal tubers, large fontanel has the size 3x4 cm, and its edges are thickened. At the ribs "rosary" are palpated. Severe muscular hypotonia: "frog belly", the child badly stands with support on his feet. Determine the course of the disease. A. Acute B. Recurrent C. Latent D. Limp E. * Subacute

312. The little girl was 7 months. The mother complained of seizures, jittery child, cyanosis, crying. The attack lasts from 30 seconds to 1 minute. Childbirth is not complicated. Located on the inadequate artificial feeding. Vegetables, fruits, juices did not receive. An objective examination: pale, overly well-fed, head of the square shape, ribs "rosary", is not sitting, on the legs is not supported. Your preliminary diagnosis? A. Rickets, severe, acute course, period of the outbreak B. Rickets, the moderate severity, residual periodC. Rickets, acute course, period of convalescence, spasmophylia, eclampsia D. De Toni Debre-Fanconi Disease. E. * Rickets, severe, subacute course, the crisis period, spasmophylia, laryngospasm

313. The little girl was 7 months. The mother complained of seizures, jittery child, cyanosis, crying. The attack lasts from 30 seconds to 1 minute. Childbirth is not complicated. Located on the misallocation of artificial feeding. Vegetables, fruits, juices did not receive. An objective examination: pale, overly well-fed, head of the square shape, ribs "rosary", is not sitting, on the legs is not supported. What laboratory tests are needed for staging the final diagnosis? A. Determining the level of phosphorus in the blood. B. Complete blood analysis. C. Determination of sweat chloride. D. Determining the level of vitamin D in the blood. E. * Determining the level of calcium in the blood.

314. Child is 7 months. Observed pallor of the skin, frontal and parietal tubers, ribs "rosary", "bracelets", "string of pearls", thickening of the large fontanelle edges, increased liver, Harrison's groove. What course of rickets in this childA. Acute B. Recurrent C. Latent D. Limp E. * Subacute

315. The child is 7 months. Observed pallor of the skin, frontal and parietal tuber, rib "rosary", "bracelets", "string of pearls", thickening of the edges of the large fontanelle, increased liver,

Harrison's groove. The diagnosis: Rickets, moderate severity. Which clinical symptoms indicate the severity of rickets? A. Harrison’s groove B. The predominance of osteoid hyperplasiaC. The predominance of osteomalacia D. The age of 7 months E. * The presence of osteoid hyperplasia signs simultaneously on the head, hands and trunk.

316. The child of 8 months is fed exclusively with cow's milk. Locomotor activity is low. He is not sitting, standing with support. Has Olympic forehead, the ribs “rosary”, “bracelets”, the lower limbs with O-shaped deformation. Heart tones are deaf. Liver and spleen are increased. Clinical manifestations correspond to: A. Rickets severe, acute course B. secondary Rickets, severe, acute course C. secondary Rickets, severe, subacute course D. Rickets mild, subacute course E. * Rickets severe, subacute course

317. Female 7 month was fed artificially, physical development is satisfactory. During the prolonged crying suddenly she was covered with sticky sweat, breath has stopped, cyanosis of the face has developed. After a few seconds - loud inbreath ("cock"-sound), after which the child become normal. Seen a girl, doctor found signs of rickets. To diagnose disease the most informative studies are: A. Electroencephalogram B. Sulkovitch’s testC. Blood sugar investigationD. Investigation of aminoaciduria E. * Investigation of blood calcium

318. The district pediatrician is examining the healthy full-term baby of 1 month old, who is breastfed. The prevention of which disease will be recommended first of all? A. Paratrofiya B. Anemia C. Hypotrophy D. Spasmophylia E. * Rickets

319. The district pediatrician is examining the child of two months. Mother complaints of periodic anxiety, increased sweating in the child. Occiput is flattened, bald. The edges of the great fontanelle are pliable. What is the disease? A. Vitamin D-resistant rickets B. Phosphate-diabetes C. Syndrome De Toni-Debre-Fanconi D. Spasmophylia E. * Rickets

320. The district pediatrician is examining the child of two months. Mother complaints of periodic anxiety, increased sweating in the child. Occiput is flattened, bald. The edges of the great fontanelle are pliable. What dose of vitamin D3 should be appointed to the child? A. 5-10 thousand IU / day B. 20-25 thousand IU / day C. 10 - 15 thousand IU / day D. 15-20 thousand IU / day E. * 2 - 5 thousand IU / day

321. Monthly child became restless, with increased sweating of the head. From the history of life: was born in September. Is fed with cow's milk. Visible craniotabes is seen. Doctor has ordered the course of UV irradiation. Does the child need ergocalciferol? A. No need

B. In combination with UV-irradiation C. Immediately after completion of the course UFO D. 1 month after completion of the course UFO E. * A 2-2,5 months after the end of the course UFO

322. During the intramuscular injection of DTP vaccine in the clinic for the child at the age of 3 months suddenly had appeared phenomena of laryngism, pale skin, cyanosis of the lips, "cock” crowing, stop of breathing, tension the entire body with his head thrown back. Allergic child's history is not complicated. Before inoculation some abnormalities were noted by pediatrician. What is the most likely diagnosis? A. Meningism, clonic-tonic convulsions B. Anaphylactic shock, clonic convulsions C. Meningoenciphalitic reaction, clonic-tonic convulsions D. Bleeding into the brain, tonic convulsions E. * Spasmophylia, tonic convulsions

323. After preventive examination of the child of 1 month, born in September, in gestation age of 38 weeks, and now is breastfed, the doctor has advised to the mother to start vitamin D 500 IU per day. Would you agree with this recommendation? If not, why not? A. No. We have to start the prevention of rickets in 2 months. B. No. Prophylactic dose of vitamin D for this child should be 1000 IU, because he was born

premature. C. No. When breastfeeding should stop, than further appoint vitamin D, it needs overlap with

mother's milk.D. No. Vitamin D is not necessary to appoint, as a child has no signs of rickets. E. * Yes. Prophylactic dose of vitamin D doctor has named correctly.

324. After preventive examination of the child of 1 month, born in September, in gestation age of 35 weeks, and now is breastfed, the doctor has advised to the mother to start vitamin D 500 IU per day. Would you agree with this recommendation? If not, why not? A. Yes. Prophylactic dose of vitamin D doctor has named correctly. B. No. We have to start the prevention of rickets in 2 months. C. No. When breastfeeding should stop, than further appoint vitamin D, it needs overlap with

mother's milkD. No. Vitamin D is not necessary to appoint, as a child has no signs of rickets. E. * No. Prophylactic dose of vitamin D for this child should be 1000 IU, because he was

born premature. 325. A 2,5-month-old child presents with muscle hypotonia, sweating, alopecia of the back of the

head. The child is prescribed massage, curative gymnastics and vitamin D. What is the dosage and frequency of vitamin D administration?A. 1000 IU every two days B. 500 IU daily C. 500 IU every two daysD. 1000 IU dailyE. * 3000 IU daily

326. Child is 7 months. The mum has addressed with the complaints on periodic twitching of a chin, which strengthens during disturbing, jerk of separate groups of muscles, shudder in dream. How it is possible to confirm the diagnosis of a spasmophilia for this child?A. To do Sulkovitch’s testB. To determine a level of potassium in a bloodC. To determine a level of magnesium in a bloodD. To click the radical of tongueE. * To test Hvostek, Lust signs

327. Child is 8 months old. His mother says that during dressing the child began to cry, noisy breathing has appeared, the child become blue, and covered by cold sweet, then was a short-time apnea. Mother washed his face by cold water, and the boy has noisy inhaled, in some minutes became in

norm, began to play. Deliver the previous diagnosis.A. Latent SpasmophyliaB. Manifestive Spasmophylia, eclampsiaC. Manifestive Spasmophylia, carpopedal spastic strictureD. RicketE. * Manifestive Spasmophylia, laryngospasm

328. Child is 8 months old. His mother says that during dressing the child began to cry, noisy breathing has appeared, the child become blue, and covered by cold sweet, then was a short-time apnea. Mother washed his face by cold water, and the boy has noisy inhaled, in some minutes became in norm, began to play. What laboratory investigations will confirm the diagnosis of Spasmophylia?A. investigation of sodium and potassium in a bloodB. investigation of magnesium in a bloodC. Sulkovitch testD. glucose tolerant testE. * investigation of Ca and Р in a blood

329. Child of 9 months old has viral infection, cramps, general cyanosis, and loss of consciousness. Objectively – clonic and tonic convulsions, skin cyanosis, foam on the lips, signs of a Rickets. Changes in lungs are not detected, as meningeal signs too. What is the first aid?A. Vitamin DB. Peroral calcium, anticramps drugs C. anticramps drugs D. Spasmolytics intravenously E. * Introducing intravenously calcium and anticramps drugs

330. District pediatrician examines a healthy carried 1-month-old child. The child is breast-fed. Prophylaxes of what disease will the doctor recommend to do first of all?A. SpasmophyliaB. MalnutritionC. ParathropyD. AnemiaE. * Rickets

331. During intramuscular DTP vaccination in clinic, a 3-month-old child developed signs of laryngospasm, paleness of skin, cyanosis of lips, "cock cry", stop of respiration, and tension of the whole body with overturned backward head. Allergological history of the child is not complicated. What is the most probable diagnosis?A. Cerebral haemorrhage, tonic spasmsB. Meningoencephalitic reaction, clonic and tonic spasmsC. Anaphylactic shock, clonic spasms D. Meningism, clonic and tonic spasmsE. * Spasmophilia, tonic spasms

332. In a medical procedures’ room during injection the 9 months’ old child has screamed, in the short-time has arisen apnea and skin cyanosis, then pallor, the child was very flabby. Life history: the child is from І normal pregnancy, artificial feeding by formula from two weeks till 2 months, then by cow’s milk, porridges from 6 months. He did not receive juices, fruits and vegetables. Objectively: body temperature is normal, skin is pale, perioral cyanosis, frontal bossing, large fontanelle is 1х1.5 cm, with infiltrated dense margins, rachitic rosary were found. The child sits with support, does not stay independently, and has not teeth. What is the diagnosis?A. Manifestive form of Spasmophylia.B. Hypervitaminosis D.C. Latent form of Spasmophylia.D. Eclampsia.E. * The Rickets ІІ stage, subacute course, the period of progression. The manifistive form of

Spasmophylia.333. In the 10 months’ old child on the background of viral infection have appeared repeated cramps.

Objectively - signs of Rickets, 2 stage are present. A level of the blood calcium is 1.6 mmol/l, interval QT on ECG – 0.33 sec. The child has artificial feeding, without vegetable foods. What disease has manifested on the phone of viral infection? A. EncephalitisB. MeningitisC. Encephalitic reactionD. NeurotoxicosisE. * Spasmophylia

334. In the 11 months’ old child, who has viral infection, have appeared cramps, general cyanosis, loss of consciousness. (In an anamnesis he received treatment by Videin-3). Objectively – clonic and tonic convulsions, skin cyanosis, foam on the lips, signs of Rickets. Changes in lungs are not detected, as meningeal signs too. What is the previous diagnosis?A. Spasmophylia, laryngospasmB. NeurotoxicosisC. Hypervitaminosis DD. MeningitisE. * Spasmophylia. Eclampsia.

335. Mother with an infant visited the pediatrician. Her baby was born with body mass of 3,2 kg and 50 cm length. He is 1 year old now. How many teeth the baby should have?A. 10 B. 20 C. 6D. 12E. * 8

336. Pediatrician has examined 6-month's old child\: the skin is pale, muscular tonus is reduced, frontal bossing is present, sizes of large fontanelle is 1х1 cm, its’ margins are dense, “rachitic rosary” were found. The diagnosis is rickets. What course of the disease is possible in this child?A. AcuteB. ChronicC. PersistentD. RelapsedE. * Subacute

337. The 10 months old child has frontal bossing, “rachitic rosary", “bow legs”, sizes of large fontanelle is 1.5х2 cm. The vitamin D3 was included in a treatment. What is its’ daily dose?A. 100-500 IUB. 1000-2000 IUC. 5000-10000 IUD. 10000-20000 IUE. * 2000-5000 IU

338. The 5-months' old girl has general cramps, loss of consciousness. It is known, that last three weeks the child had receive vitamin D3 in a dose 3000 IU and general ultra violet insolation every day to treat Rickets. Day before she had a short-time apnea. Objectively - body temperature is normal signs of the Rickets are present. What can provoke this complication?A. RicketsB. Hypervitaminosis DC. Error in a feed D. PolyhypovitaminosisE. * Spasmophylia

339. The child of 10 months on the phone of virus infection has appeared repeated cramps. At the view - the signs of a rickets of 2 stage. A level of calcium of a blood - 1,6 mmol/l, interval QT on ECG - 0,33 sec. Artificial feeding, without vegetable foods. What disease was exhibited on the phone of virus infection? A. Encephalitis

B. MeningitisC. Encephalitical reactingD. NeurotoxicosisE. * Spasmophilia

340. The doctor has examined the 3-month's child and suspected an initial stage of a rickets. What changes of muscular system will be present in him?A. Hypertonia of musclesB. Muscle atrophyC. Decrease of muscles forcesD. Increase of muscles forcesE. * Hypotonia of muscles

341. The mother of the 4-month's child has addressed to the doctor with complaints on a decrease of appetite, regurgitation, subferile temperature in her child. It is known, that from the 2nd month of life the child receives vitamin D3 in preventive dose /500 IU each day/, from the 3-rd month of life because of irritability and sweating was prescribed general ultra-violet insolation. Objectively: large fontanelle is closed, skin with perioral cyanosis. Laboratory investigations: Sulcovich test is positive (++), serum calcium – 3.5 mmol/l. Name the most probable diagnosis:A. RicketsB. SpasmophyliaC. Personal intolerance of vitamin DD. Hypovitaminosis DE. * Hypervitaminosis D

342. The pediatrician has examined 1-month's old child, which was born from the І physiological pregnancy. In 3 weeks of life because of mother’s hypohalactia he start to receive bottle feeding by cow’s milk in 2:1 dilution. He does not receive the fresh air. What disease should be prevented first of all? A. AnemiaB. MalnutritionC. SpasmophyliaD. DiarrheaE. * Rickets

343. To the doctor has addressed the mother of the 4-month's child with the complaints to a decrease of appetite, regurgitation, fervescence subfibrile. It is known, that since 2 month of life the child with the preventive purpose receives vitamin Д3 /500 IU each day /, since 3-rd month of life in connection with appearance of disturbing and sweating was intended general ultra-violet. At the view: large crown closed, skin acyanotic, periorale cyanosis. By results of examination: assay Sulcovich ++, serumal calcium - 3,5 mmol/l. To call the most probable diagnosis:A. RicketsB. SpasmophiliaC. Personal intoleranse of vitamin DD. Hypovitaminosis DE. * Hypervitaminosis D

344. 10 months’ old girl was treated from otitis, received Penicillin injections. In 4 days diarrhea has developed. What corrections in treatment are necessary?A. stop Penicillin injections.B. add bacterial drugs.C. stop Penicillin injections, add bacterial drugs.D. stop Penicillin injections, to change it for gentamicine, add bacterial drugs.E. * stop Penicillin injections, change it for ampicilline, add bacterial drugs.

345. 2-months’ old child with body weight 4,300 was hospitalized to the clinic because of gastroenteral enzymopathy, isotonic dehydration. What medicine, except rehydration therapy must be prescribed to this child?A. Antibacterial therapy

B. ImmunostimulatorC. SorbentsD. MultivitaminsE. * Enzymes and bacterial drugs

346. 3.5 months’ old child has lost more than 10% of the body weight. Dehydration features are following: shock - falling blood pressure with tachycardia, coma, anuria. What type of dehydration does this child have?A. Water deficiency B. Mixed C. Isotonic.D. Data of laboratory examinations are necessary for definition of the dehydration type E. * Salt deficiency

347. 3.5 months’ old child is treated from acute bilateral bronchopneumonia complicated by secondary gastroenteral enzymopathy. What main principles of treatment?A. Antibacterial, immunostimulation therapyB. Oral rehydration and antibacterial therapyC. Massive bacterial therapyD. Enzymes without an antibioticE. * Antibacterial, rehydration, bacterial therapy

348. 6-weeks’ old child has a frequent stool, vomits, and draws his legs up to the abdomen. The biochemical examination of the blood shows hyponatremia, hypokalemia, hypoalbuminemia mother notes that symptoms have develop after using new food (egg). Put your diagnosis.A. pyloric stenosisB. peptic ulcerC. infectious diarrhea.D. PylorospasmE. * functional diarrhea

349. A 1-year-old child suffers of attack-like cough. The child presents with the history of dyspepsia since birth. On physical examination there are signs of delay in physical development, bronchial obstruction, respiratory insufficiency, 1 grade. Blood count: signs of inflammatory process. Sweat chlorides 120 mEq/L. What is the most likely diagnosis?A. Bronchopulmonary dysplasiaB. Kartagener's syndromeC. Acute respiratory infection, bronchitis D. Severe bronchial asthmaE. * Cystic fibrosis

350. A 1-year-old infant is admitted for failure to thrive. During the neonatal period he had an exploratory laparotomy for intestinal obstruction. At 3,8 and 11 month of age, he had respiratory infections diagnosed as bronchitis. Physical examination: weight of 6,8 kg, thin extremities with very little subcutaneous tissue, and a protuberant abdomen. The essentials diagnostic study in this child is:A. Skin test for milk allergyB. BronchoscopyC. Serum immunoglobulin level D. Tuberculin skin testE. * Sweat electrolytes

351. A 2 month old full-term child was born with weight 3500 g and was on the mixed feeding. Current weight is 4900 g. Evaluate the current weight of the child: A. Hypotrophy of the II grade B. 150 g less than necessary C. Hypotrophy of the I grade D. Paratrophy of the I grade E. * Corresponding to the age

352. A 2 y.o. boy was admitted to the hospital with weight loss, unstable discharges, anorexia, following the semoli¬na's introduction (since 5 months). The child is adymanic, flabby, pale dry skin, subcutaneous layer is emaciated. Distended and tensed abdomen, tympanitis on percussion of the upper part of the abdomen, splashing sounds, feces are foamy, of light color, foul. On coprocytogram\: a lot of neutral fat. What is the cause of the disease?A. Intestinal dysbacteriosisB. Mucoviscidosis (cystic fibrosis)C. Disaccharidase insufficiencyD. Chronic enteritisE. * Celiakia (celiac disease)

353. A 6-month-old infant was born with body mass of 3 kg and 50 cm length. He is breast-fed. How many times per day should the infant is fed?A. 8 B. 4C. 6 D. 7E. * 5

354. A child was born with body weight 3250 g and body length 52 cm. At the age of 1,5 month the actual weight is sufficient (4350 g), psychophysical development corresponds with the age. The child is breast-fed, occasionally there are regurgitations. What is the cause of regurgitations?A. Esophageal atresia B. Acute gastroenteritis C. Pylorospasm D. Pylorostenosis E. * Aerophagia

355. A child, 4 years old, has Entheropathogenic Escherichiosis, moderate degree. Prescribe the pathogenetical treatment.A. Pancreatin B. Nifuroxasid C. Enterol D. SorbitE. * Oralit

356. A child, 5 years old, has dyspeptic syndrome, moderate intoxication syndrome, and abdominal pain. Salmonellosis was diagnosed. What changes in the general blood test are typical to this disease?A. Leucopenia, aneosinophylia, lymphocytosis.B. Leucocytosis, lymphocytosis.C. Leucopenia, neutrophyllosis, ESR decreasing.D. Leucocytosis, lymphomonocytosis, atypical mononuclear cells.E. * Leucocytosis, neuthrophyllosis, ESR increasing.

357. A mother consulted a pediatrician about her son. Her son was born with body mass of 3 kg and length of 48 cm. He's 1 year old now. What is the required normal mass?A. 9,0 kg B. 15,0 kg C. 11,0 kg D. 12,0 kg E. * 10,5 kg

358. A neonate from gestation with severe gestosis of the second half was born on the 41st week with 2400 g birth weight and 50cm long. On physical examination: skin is flaccid, subcutaneous fatty cellular tissue is thin, muscle hypotonia, new-born period reflexes are decreased. Internal organs are without pathological changes. How would you estimate this child?A. Term infant with normal body weightB. Premature infant

C. Postmature infantD. Immature infantE. * Term infant with pre-natal growth retardation

359. In a child, 4 months old, suddenly has increased the temperature to 38.5 ?С. Later has appeared frequent defecation near 15 times per day. Excrements are yellow-green, liquid consistency, with mucus. During examination: skin is pale, lips are bright, dry. The child has thirst. Big fontanel is 1.5х1.5 cm, sunken. Breathing is puerile, 42 per 1 minute. Heart tones are loud, rhythmic, 148 per 1 minute. The abdomen is mildly distended, painful. Salmonellas are found in excrements. Biochemical blood analyses: Nа - 163 mmol/l, K - 5.7 mmol/l. What type of dehydration is possible?A. Isotonic.B. Hypotonic.C. Salt deficient.D. Dehydration is absent.E. * Hypertonic.

360. In a child, 7 years old, dysentery reveals itself by often defecation to 20-25 times per day, skin is pale, dry, elasticity is reduced, the body temperature is 38.9 ?С, repeated vomiting, colicky pain are present. What is the severity of the disease in this caseA. Mild B. ModerateC. severe D. severe degree with prevalence of toxicosis.E. * severe degree with prevalence of the local manifestations.

361. In the 3 months' old child, who is on the breast feeding was diagnosed Malnutrition 2nd degree. What from this is better to prescribe the child to correct dysbacteriosis?A. Festal B. Apilac C. Ampicillin D. NistatinE. * Bifidumbacterin

362. In the 5 months’ old child signs of water deficiency dehydration were found. What probable dates of laboratory examinations?A. Decrease of hematocritB. Decrease of K, Na, Cl level in the blood, considerable increase of hematocriC. Decrease of K, Na, Cl level in the bloodD. Detection of a bacterial infectionE. * Increase of potassium level

363. The 4 months' old child is treated in the hospital because of Malnutrition 3rd degree. What duration of the parentheral feeding is optimal for this child?A. 1-2 days B. 10 days C. On whole period of treatment D. 2-3 weeks.E. * 7-14 days

364. The 7-months' child is treated from alimentary Malnutrition 2nd degree. The weight deficit is 25%. What accompanying disease most often could be present in this child?A. Salmonellosis B. Sepsis C. Spasmophylia D. Exudative-catarrhal diathesisE. * Rickets

365. The boy, 3 months old, is treated in infectious department because of Salmonellosis, moderate degree, caused by Salmonellae typhimurium. Which etiological treatment is reasonable to use?

A. Benzylpenicillin.B. Erythromycin.C. Cefazolin.D. Oxacillin.E. * Cefotaxim.

366. The boy, 4 years old, complains of abdominal pain, repeated vomiting, increased frequency of defecation, fluid feces, high body temperature – 38 ?С. He is ill for 4 days. Parents did not address to physician before. The patient received Ampicillin in tablets. Because of poor condition (the appearing of blood in feces) he entered to infectious department. The skin is pale tongue is covered by white stratification. The abdomen is distended, mildly painful. The feces are green-gray, with mucus, blood. Salmonellosis is suspected. What investigation will confirm the diagnosis?A. General blood test.B. Bacteriological examination of cerebrospinal fluid.C. Fat drop.D. Bacteriological investigation of pharyngeal swab.E. * Bacteriological examination of the feces on dysentery, typhoid, paratyphoid fever.

367. The child 4 months old has expressed thinning of the subcutaneous cellulose. The thickness of the pleat near the umbilicus is 0.4 cm. The child is flaccid, hypodynamic, shouted weakly, the body temperature is reduced, his face is wrinkled. Malnutrition 3rd degree was diagnosed. What weight deficit is typical for this degree of malnutritionA. 21-30% B. 10-20%C. 10-30%D. 5-25%.E. * 40-50%

368. The child 5 months' old, has entered to the clinic with Malnutrition 1st degree. The diet was prescribed to him. What duration of the food tolerance determination period in this case?A. 1-2 weeks B. 2-3 weeks C. 7-14 days D. 3-4 weeksE. * 1-3 days

369. The child 6 months' old suffers from alimentary Malnutrition 2nd degree, period of recovery. He is on the step of optimum feeding. At calculation of the feeding has appeared the deficit of the protein part of ration. How to correct the protein deficit?A. By the Porridge B. By the Vegetable puree C. By the Yoghurt D. By the Fruit juice E. * By the pot cheese

370. The Child has entered to clinic with complaints on anxiety, increased appetite, metheorism. Under objective examination thinning of the subcutaneous tissue on the abdomen and trunk was revealed. The weight deficit is 22%. Malnutrition 2nd degree was diagnosed. What part from necessary volume of the food is necessary to give the child during the first week? A. Full volume of the food B. 3/4 of the necessary volume C. 1/3 of the necessary volume D. 2/3 of the necessary volume E. * Half of the necessary volume

371. The child is 6 months. His birth weight is 3 kg, now his weight is 6.800 kg (weight deficit * 13%) This corresponds:A. The variant of the rate

B. Malnutrition 2nd degree C. Malnutrition 3rd degreeD. Paratrophia, 1st degreeE. * Malnutrition 1st degree

372. The child was born prematurely, with body weight 2.200 kg (in 33-34 weeks of gestation). The trophyc index (TI) is 0. Such value of TI is typical for:A. Prenatal Malnutrition 1st degree B. Prenatal Malnutrition 2nd degree C. Prenatal Malnutrition 3rd degree D. ParatrophyaE. * Normotrophya

373. The Child, 2 years old, is treated because of Entheroinvasive Escherichiosis. Prescribe the ethiothrope treatment.A. Nifuroxasid B. Pancreatin C. Enterol D. EnterodesE. * Ampicillin

374. The child, 3 years old, is treated in infectious department because of acute Shigellosis. During objective examination: the body temperature is 39.9 ?С, skin is pale, dry, periodic tonic twitches of the muscles, limbs are cool, abdomen is sealled, sensitive in left inguinal region, anus is open. Name the diagnose.A. Shigellosis typical form, moderate degree.B. Shigellosis, atypical form.C. Shigellosis typical form, severe degree.D. Shigellosis typical form, severe degree with prevalence of the local manifestations.E. * Shigellosis typical form, severe degree with prevalence of toxicosis.

375. The child, 7 months old, is treated in infectious department because of Salmonellosis, gastrointestinal form, moderate gravity, toxicosis with exicosis II degree, caused Salmonellae enteritidis. What percent of weight loss is probable in this case?A. 1-3 %.B. 3-6 %.C. Less than 5 %.D. 10-15 %.E. * 5-10 %.

376. The mother of 3-months' old girl has addressed to district pediatrician with complaints on anxiety of the child, small interval between feedings less than 3.5-hours. Objectively\: thinning of the subcutaneous fat on the abdomen and trunk. The malnutrition 2nd degree was diagnosed. What weight deficit is characterized for this degree of malnutrition?A. 10-20% B. 30-40%C. 40 % and more D. 5-10%E. * 20-30%

377. To study physical development of children and adolescents, anthropometric investigations are widely used. Choose a physiometric method of investigation from the below given.A. Determination of thorax formB. Determination of vertebra formC. Determination of body weightD. Measurement of growthE. * Determination of vital capacity of lungs

378. 15 y.o. female was admitted to thoracic surgery department with fever up to 40°C, onset of pain in the side caused by deep breathing, cough with considerable quantity of purulent sputum and

blood with bad smell. What is the most likely diagnosis?A. Complication of liver echinococcosisB. Pulmonary tuberculosisC. Actinomycosis of lungsD. Bronchiectatic diseaseE. * Abscess of the lung

379. 2 years old child has dry cough, dyspnea, body temperature is 37.5 °C. Percussion: clear pulmonary sound without dullness. Auscultation: dry whistling and different moist rales. In the peripheral blood: leukocytosis, eosynophylia, increased ESR. What disease is possible?A. Acute simple bronchitisB. Whooping coughC. Acute pneumoniaD. Bronchial asthmaE. * Obstructive bronchitis

380. A 1-year-old child suffers of attack-like cough. The child presents with the history of dyspepsia since birth. On physical examination there are signs of delay in physical development, bronchial obstruction, respiratory insufficiency, 1 grade. Blood count\: signs of inflammatory process. Sweat chlorides 120 mEq/L. What is the most likely diagnosis?A. Bronchopulmonary dysplasiaB. Kartagener's syndromeC. Acute respiratory infection, bronchitis D. Severe bronchial asthmaE. * Cystic fibrosis

381. A 1-year-old infant is admitted for failure to thrive. During the neonatal period he had an exploratory laparotomy for intestinal obstruction. At 3,8 and 11 month of age, he had respiratory infections diagnosed as bronchitis. Physical examination: weight of 6,8 kg, thin extremities with very little subcutaneous tissue, and a protuberant abdomen. The essentials diagnostic study in this child is:A. Skin test for milk allergyB. BronchoscopyC. Serum immunoglobulin level D. Tuberculin skin testE. * Sweat electrolytes

382. A 1,5-year-old child has following symptoms: chronic cough with purulent sputum discharge. Dyspnea, physical retardation, large amount of stool. Sweat chloride isl50 mEq/L. The child has been ill since 2 month age. Diagnosis: cystic fibrosis. What is the most suitable therapy?A. Vitamins + mucolyticsB. H2-blockers + hepatoprotectorsC. Cholepoietic+adaptogenetic medicinesD. Vitamins+antibioticsE. * Enzymes + antibiotics

383. A 10-year-old boy complains of a headache, weakness, fever [temperature — 40° C], vomiting. On physical examination: there is an expressed dyspnea, pale skin with a flush on a right cheek, right hemithorax respiratory movement delays, dullness on percussion of the lower lobe of the right lung, weakness of vesicular respiration in this region. The abdomen is painless and soft by palpation. What disease causes these symptoms and signs?A. Intestinal infectionB. Acute appendicitisC. Acute cholecystitisD. InfluenzaE. * Pneumonia croupousa

384. A 10-year-old boy has a history of recurrent pneumonias and chronic cough production of foul smell, purulent sputum, ocassionally gloom tinged, which becomes worse in the morning and in

reclining position. On physical examination, it is a chronic patient with clubbing of fingers, wet inspiratory tract at the root of lungs from behind. What is the most probable diagnosis?A. Disseminated pulmonary tuberculosisB. Pulmonary neoplasmC. Chronic bronchitisD. Chronic obstructive emphysemaE. * Bronchoectasis

385. A 14-year-old patient has been treated in a hospital. A fever of 39°C, chest pain which is worsened by breathing, cough, brownish sputum appeared on the 7th day of the treatment. Chest X-ray shows left lower lobe infiltrate. Which of the following is the treatment of choice for this patient?A. ErythromycinB. StreptomycinC. PenicillinD. TetracyclineE. * Cephalosporins of the III generation

386. A 16-year-old male was admitted to the hospital complaining of stabbing back pain on inspiration and dyspnea. On exam, t - 37°C, Ps 92/min, RR of 24/min, vesicular breath sounds. There is a dry, grating, low-pitched sound heard in both expiration and inspiration in the left lower lateral part of the chest. What is the most likely diagnosis?A. Acute bronchitis B. Pneumonia C. Pneumothorax D. MyocarditisE. * Acute fibrinous pleuritis

387. A 16-year-old male was discharged from the hospital after having an out-of-hospital pneumonia. He has no complaints. On physical xam: his temperature is — 36,6°C, RR-18/min, Ps — 78 bpm, BP — 120/80 mm Hg. During ausculation there is harsh respiration to the right of the lower part of the lung. Roentgenologically: infiltrative changes are absent, intensification of the pulmonary picture to the right in the lower lobe. How long should the doctor keep the patient under observation?A. 1 monthB. 3 monthsC. PermanentlyD. 6 monthsE. * 12 months

388. A 3 month old infant suffering from acute segmental pneumonia has dyspnea (respiration rate - 80 per minute), paradoxical breathing, tachycardia, total cyanosis. Respiration / pulse ratio is 1:2. The heart dullness under normal size. Such signs characterise: A. Congenital heart malformation B. Myocarditis C. Respiratory failure of II degree D. Respiratory failure of I degree E. * Respiratory failure of III degree

389. ?A 3 year old child with weight defficiency suffers from permanent moist cough. In history there are some pneumonias with obstruction. On examination: distended chest, dullness on percussion over the lower parts of lungs. On auscultation: a great number of different rales. Level of sweat chloride is 80 millimol/l. What is the most probable diagnosis? A. Pulmonary hypoplasia B. Recurrent bronchitis C. Bronchial asthma D. Bronchiectasis E. * Mucoviscidosis (cystic fibrosis)

390. A 3-month-old infant who is suffering from acute segmental pneumonia reveals dyspnea, respiration rate is 80 per minute, paradoxical breathing, tachicardia, total cyanosis. Respiration-pulse ratio is 1:2. The heart size is normal. What are these signs indicative for?A. Congenital heart diseaseB. Respiratory failure of I degreeC. Respiratory failure of II degreeD. MyocarditisE. * Respiratory failure of III degree

391. A 5-year-old girl with the transitory immunodeficiency according to T-system has a clinical picture of a right-sided pneumonia during 2 months. How pneumonia progress can be described?A. AcuteB. ReactivatingC. ChronicD. WavelikeE. * Delaying

392. A 6 year old girl has an acute onset of fever up to 39oC with chills, cough, and pain on respiration in the right side of her chest. On physical examination: HR - 120/min, BP- 85/45 mm Hg, RR- 36/min. There is dullness over the right lung on percussion. On X-ray: infiltrate in the right middle lobe of the lung. What is the diagnosis? A. Community-acquired lobar pneumonia of moderate severity B. Acute pleuritis C. Acute lung abscess D. Nosocomial lobar pneumonia E. * Community-acquired bronchopneumonia

393. A boy, 8 years old, has addressed to pediatrician with complains of increasing of the body temperature to 37.5 ?С, sore throat, cough, serous discharge from nose, tearing. During examination mild hyperemia and edema of the tonsils and back pharyngeal wall, conjunctives, narrowing of ocular slots were revealed. The physician suspects adenoviral infection. Which method of express-diagnostics is better to use for acknowledgement of the diagnosis?A. The separation of the virus on tissue culture B. Serological investigation C. bacteriological investigation D. Bacteriosсopy investigation of pharyngeal swabE. * Immunofluorescent method

394. A child was born at 34 weeks of gestation in bad condition. The cardinal symptoms show respiratoty disorders: sound prolonged expiration, additional muscles taking part in breathing, crepitation rales on the background of the rough breath sounds. Assesment according to Silverman's scale was 0, in 3 hours- 6 with presence of clinical data. What diagnostic method can determine pneumopathy's type in the child?A. Immunologic investigationB. Blood gasesC. ProteinogramD. Blood testE. * Chest X-ray

395. A patient with nosocomial pneumonia presents signs of collapse. Which of the following pneumonia complications is most likely to be accompanied by collapse? A. Exudative pleuritis B. Bronchial obstruction C. Toxic hepatitis D. Emphysema E. * Septic shock

396. An 18-month-old child is taken to hospital on the 4-th day of the disease. The disease began acutely with temperature 39 °C, weakness, cough, reastlessness. He is pale has cyanosis, febrile

temperature for more than 3 days. There are crepitative fine bubbling rales at the auscultation. Percussion sound is shortened in right under scapula area. X-ray picture: unhomogenous segmental infiltration 8-10 in the right, the increase of vascular picture, unstructural rools. What is the most likely diagnosis?A. BronchitisB. BronchiolitisC. Interstitial pneumoniaD. InfluenzaE. * Segmental pneumonia

397. In the anamnesis of a 2-year-old girl there are recurrent pneumonias with signs of obstruction. There are heterogeneous moist and dry rales, respiration is weakened. Dense, viscous secretion is difficult to hawk. There are "drumsticks", physical retardation. What is the most probable diagnosis?A. Pulmonary tuberculosisB. Bronchial asthmaC. Recidivating bronchitisD. Congenital pulmonary polycystosiE. * Cystic fibrosis, pulmonary form

398. In the child, 7 years old, disease has acute beginning, with increasing of the body temperature to 39 ?С. Objectively: hyperemia of the face, injection of sclera vessels, moderate cyanosis of the lips. During examination of the oral cavity exists maculous exanthema on soft palate, groiness of the back pharyngeal wall. The skin is clean, pale tachycardia is present. What disease is most probable in this case?A. Measles.B. Typhoid fever.C. Hemorrhagic fever.D. Parainfluenza.E. * Influenza.

399. In the infectious hospital has admitted 5 years old girl with pharyngoconjunctivitis phenomena. The physician has suspect adenoviral infection. What method from express-diagnostics is reasonable to use to prove this diagnosis?A. Binding complement reaction B. Indirect hemaglutination reaction C. neutralization reaction D. Direct hemaglutination reactionE. * The fluorescence antibody method

400. Pediatrician was called to the 2-years old child who’s mother complaints of a subfebrile temperature, rhinitis, dry cough. He is ill for 3 days. During percussion: a clear pulmonary sound without dullness. During auscultation\: puerile breathing. Laboratory findings: leukopenia, lymphocytosis, increased ESR. What disease is possible first of all?A. Acute obstructive bronchitisB. Acute bronchopneumoniaC. Resedive bronchitisD. Acute bronchitisE. * Acute tracheitis

401. The 5-months' old child has subfibrile temperature, paroxysmal cough, and dyspnea. He is hospitalized. 3 days ago he was in a contact with sister ill on acute upper respiratory tract viral infection. Objectively: the condition is severe skin is cyanotic, considerable expiration dyspnea, oral crepitation. Percussion: dull sound. Auscultation\: a plenty of wet rales in both sides, respiratory rate is 80 per 1 minute. What disease is possible?A. Bronchial asthmaB. Aspiration of a foreign bodyC. Acute bronchitis

D. Acute pneumoniaE. * Bronchiolitis

402. The 5-years old child has obstructive bronchitis. What drugs will be used as a base therapy?A. AntibioticsB. Antiallergic medicineC. MucolyticsD. HormonsE. * Broncholytics

403. The 7-years’ old child was hospitalized with complaints on wet cough. His condition has worsened 10 days ago. He is ill during last 4 years: viral infections 3-4 times per year are accompanied by bronchitis. Objectively: cough with slimy sputum. Percussion: a clear pulmonary sound. Auscultation: rough breathing, nonconstant single diffuse wet rales. X-ray: lung figure is increased, roots are nonstructural. What disease is possible in this case?A. Viral infectionB. Acute bronchitisC. Acute pneumoniaD. Chronic pneumoniaE. * Relapsed bronchitis.

404. The 9-month's ols child was hospitalized because of severe condition. Diagnose - bronchiolitis. What from this tells us about the severity?A. NeurotoxicosisB. Heart failureC. Suprarenal failureD. ExicosisE. * Respiratory failure

405. The boy is 8 years old. He complains of headache, increasing of the body temperature to 39 ?С, and general weakness. The diagnosis is: influenza, typical form, hard degree. How long bed regimen must be prescribed? A. 3 days B. 5 days C. 14 daysD. 18 daysE. * 7 days

406. Head circumference of a 1-month-old boy with signs of excitement is 37 cm, prefontanel is 2x2 cm large. After feeding the child regurgitates small portions of milk; stool is normal in its volume and composition. Muscle tone is within norm. What is the most likely diagnosis?A. MeningitisB. Pylorostenosis C. *Pylorospasm D. Microcephaly E. Craniostenosis

407. A 16-year-old patient complains about skin rash that appeared 2 days ago after eating smoked fish. The rash disappears after 4-6 hours but then turns up again. It is accompanied by itch. Objectively: trunk and upper limbs are covered with multiple pink blisters as big as a pea or a bean. What is the most likely diagnosis?A. Allergic dermatitisB. ToxicodermiaC. *Acute urticariaD. Quincke's edemaE. Purigo

408. Examination of a 9-month-old girl revealed skin pallor, cyanosis during excitement. Percussion revealed transverse dilatation of cardiac borders. Auscultation revealed continuous systolic murmur on the left from the breastbone in the 3-4 intercostal space. This murmur is

conducted above the whole cardiac region to the back. What congenital cardiac pathology can be suspected?A. Coarctation of aortaB. Pulmonary artery stenosisC. *Defect of interventricular septumD. Fallot's tetradE. Defect of interatrial septum

409. 130 A child was taken to a hospital with focal changes in the skin folds. The child was anxious during examination, examination revealed dry skin with solitary papulous elements and ill-defined lichenification zones. Skin eruption was accompanied by strong itch. The child usually feels better in summer, his condition is getting worse in winter. The child has been artificially fed since he was 2 months old. He has a history of exudative diathesis. Grandmother by his mother's side has bronchial asthma. What is the most likely diagnosis?A. UrticariaB. Contact dermatitisC. *Atopic dermatitisD. Seborrheal eczemaE. Strophulus

410. A 6-year-old boy was brought to the emergency room with a 3-hour history of fever up to 39,5oC and sore throat. The child looks alert, anxious and has a mild inspiratory stridor. You should immediately:A. * Prepare to establish an airwayB. Obtain an arterial blood gas and start an IV line C. Order a chest x-ray and lateral view of the neck D. Admit the child and place him in a mist tentE. Examine the throat and obtain a culture

411. A 10-year-old girl was admitted to a hospital with carditis presentations. It is known from the anamnesis that two weeks ago she had exacerbation of chronic tonsillitis. What is the most likely etiological factor in this case?A. StaphylococcusB. ProteusC. KlebsiellaD. PneumococcusE. *Streptococcus

412. An 11-year old girl was taken by an acute disease: she got pain in the lumbar region, nausea, vomiting, frequent urination, body temperature 39oC. Objectively: the abdomen is soft, painful on palpation in the lumbar region. Common urine analysis revealed considerable leukocyturia, bacteriuria. The urine contained colibacilli. What is the most likely diagnosis?A. Acute vulvovaginitisB. Acute appendicitisC. Acute glomerulonephritisD. Chronic glomerulonephritisE. *Acute pyelonephritis

413. An infant was born with body mass 3 kg and body length 50 cm. Now he is 3 years old. His brother is 7 years old, suffers from rheumatic fever. Mother asked the doctor for a cardiac check up of the 3-year-old son. Where is the left relative heart border located?A. 1 cm left from he left parasternal lineB. 1 cm right from the left medioclavicular line C. *1 cm left from the left medioclavicular lineD. Along the left medioclavicular lineE. 1 cm right from the left parasternal line

414. During examination at a military commissariat a 15-year-old teenager was found to have interval systolic murmur on the cardiac apex, diastolic shock above the pulmonary artery, tachycardia.

Which of the supplemental examination methods will be the most informative for the diagnosis specification?A. RoengenographyB. Phonocardiography C. Electrocardigraphy D. RheographyE. *Echocardiography

415. A 10-year-old girl consulted a doctor about thirst, frequent urination, weight loss. She has been observing these symptoms for about a month. Objectively: no pathology of internal organs was revealed. What laboratory analysis should be carried out in the first place?A. Glucose tolerance testB. Glucosuric profileC. Glucose in urine test on the base of daily diuresisD. *Blood glucose analysis on an empty stomachE. Acetone in urine test

416. On the third day of disease a 10 years old child with acute respiratory infection developed productive cough. The percussion is without pathologic features. The auscultation reveals bilateral rales over the lung surface. What diagnosis should be made? A. *acute bronchitis B. relapsing bronchitis C. obstructive bronchitis D. Asthma E. Pneumonia

417. An infant aged 1 year on the third day of common cold at night developed inspiratory stridor, hoarse voice and barking cough. Physical examination revealed suprasternal and intercostal chest retractions. There is a bluish skin discoloration moistly seen over the upper lip. The respiratory rate is 52 per min and pulse rate 122 per min. The body temperature is 37,50C. What disease does the infant have? A. *Acute infectious croup due to viral laryngotracheitis grade II of airway obstruction,

incomplete compensation state B. Acute laryngitis C. Bronchopneumonia without complications D. Acute bronchiolitis with respiratory distress E. Acute epiglottitis

418. One week old male infant had one episode of vomiting yesterday and 2 episodes of spitting up with poor feeding today. There is no history of fever, diarrhea or coughing. His urine output is decreased. He was born at term weight 3.2 kg. Weight now is 3.0 kg. Laboratory data: Na 128 mmol/l, K 6.9mmol/l. What is the most probable diagnosis? A. *Salt wasting form of adrenal insufficiencyB. GastroenterocolitisC. Birth trauma of central nervous systemD. PylorospasmE. Pylorostenosis

419. The boy is 10 years old. Two weeks ago he suffered from acute tonsillitis. Now he complains on common weakness, oedema of eyelids, ankle joints, headache, nausea. Skin is pale, appetite is reduced, daily diuresis is 600 ml. In urinalysis: protein – 0,066 g/l, WBC – 4-6, RBC – 40-45. The most probable diagnosis is?A. *Glomerulonephritis with nephritic syndrome B. Interstitial nephritis C. Pyelonephritis D. Glomerulonephritis with nephrotic syndrome E. Polycystic kidney disease

420. 4300-g infant was born at term gestation to a poorly controlled insulin-dependent diabetic mother.

Initially, the infant did well, but at 2 hours of age he was noted to be lethargic and than develop tonic-clonic seizures. Blood glucose is 1.6 mmol/l. What is the most likely reason of seizures? A. *hypoglycemia B. hypocalcemia C. hypomagnesemia D. hyponatremia E. pyridoxine deficiency

421. Boy is 2 months old. He was born in September. Не is on breast feeding. What should a daily doze of ergocalciferol be for prophylaxis of rickets? A. *400-500 IU B. 200-300 IU C. 300-400 IU D. 500-600 IU E. 100-200 IU

422. A 7 year old male presents to physician with the chief complaint of dark "cola colored" urine, facial puffiness and abdominal pain for the past 2 days. 14 days ago he had a sore throat and fever. He has had abdominal pain. His urine is dark. Urine analysis shows an increased specific gravity, RBCs are too numerous to count. What is the most probable diagnosis? A. *Glomerulonephritis. Nephritic Syndrome B. Glomerulonephritis. Nephrotic Syndrome C. Acute heart failure D. Acute infection of urinary tract E. Hemolytic uremic syndrome

423. A 4 year old female has been limping with swelling of her right knee for several months. Physical examination demonstrates swelling of her right knee, flexion contracture of 10 degrees and flexion to 120 degrees. Lab. data: WBC 8 g/l, with 45\% neutr., 47 lymphs\%, 8\% mon. Hgb 120 g/l. ESR 20mm/h. Rheum. factor neg., ANA 1:640 speckled. What is the most probable diagnosis? A. *Juvenile Rheumatoid Arthritis B. Rheumatism C. Lupus D. Osteomyelitis E. Infection Arthritis

424. Mother of a previously healthy 4 year old male complains of cough and wheeze. Boy had playing with a small toy. During examination the right side of a chest show hyperresonance, diminished vocal resonance and poor air entry. What is the most probable diagnosis? A. *Foreign body aspiration B. Asthma C. Pneumonia D. Bronchitis E. Bronchiolitis

425. 1 month old female presents to the intensive care unit with severe cyanosis, congestive heart failure, normal first sound, single second sound and an insignificant one to two ejection systolic murmur. The electrocardiogram shows right axis deviation and right ventricular hyperthrophy. The thoracic roentgenogram shows cardiomegaly with narrow base and plethoric lung fields. What is the most probable diagnosis? A. *Congenital heart disease, right to left shunt B. Congenital heart disease, left to right shunt C. Pneumonia D. Congenital lung malformation E. Bronchiolitis

426. 16 month old female presents with an acute onset of her hands and feet "drawing up. Both her hands are flexed at the wrists with hyperextended fingers at the proximal and distal

interphalangeal joints and flexion at the metacarpophalangeal joints. Neurologic exam reveals symmetric hyperreflexia, decreased muscle strength and tone. Lab.data: Ca 0,9 mmol/l, P 0.4 mmol/l. What is the most probable diagnosis? A. *Vitamin D deficiency, spasmophylia B. Epilepsy C. Acute infection of central nervous system D. DiGeorge syndrome E. Glycogenosis

427. Child is 2 months old. He is premature infant. What should a daily doze of ergocalciferol be for prophylaxis of rickets? A. *1000-1200 IU B. 800-1000 IU C. 1200-1500 IU D. 400-500 IU E. 1300-1500 IU

428. Girl is 4 months old. She was born in October. She is on breast feeding. To what age should prophylaxis of rickets carry out? A. *1,5 years B. 2 years C. 2,5 years D. 1 year E. 6 months

429. Boy is 4 months old. Не is on breast feeding. Determine a remedy for primary prophylaxis of spasmophilia. A. *Ergocalciferol B. Calcium gluconate C. Calcium chloride D. Natrium chloride E. Calcium pantothenate

430. Child is 3 months old. At investigation in a polyclinic rickets was diagnosed. The basic process characteristic for rickets is infringement of a metabolism. A. *Calcium and phosphorus B. Calcium and potassium C. Calcium and magnesium D. Calcium and sodium E. Calcium and zinc A

431. The child is 7 months old. He suffers from spasmophilia. What infringement of a kind of an exchange is a pathogenic part at spasmophilia? A. *Phosphoric-calcic B. Phosphoric-potassic C. Phosphoric-sodium D. Phosphoric-magnesion E. Phosphoric-zinc

432. A 6-year-old boy with asthma has had mild wheezing only four times since you began treating him 3 months ago with Cromolyn inhalation twice each day. For the past 2 days, he has again had mild coughing and wheezing. What should you recommend to treat acute attack of asthma? A. *inhalation Salbutamol B. inhalation corticosteroids C. Loratadine D. Aspirin E. Theophylline

433. Full term newborn has developed jaundice at 10 hours of age. Hemolytic disease of newborn due to Rh-incompatibility was diagnosed. 2 hours later the infant has indirect serum bilirubin level

increasing 14 mmol/L. What is most appropriate for treatment hyperbilirubinemia in this infant? A. *exchange blood transfusion B. phototherapy C. phenobarbital D. intestinal sorbents E. infusion therapy

434. 7 -year-old boy with chronic cinusitis and reccurent pulmonary infections has chest a x-ray demonstrating a right-sided cardiac silhouette. What is the most likely diagnosis? A. *Kartagener syndrome B. cystic fibrosis C. bronhiolitis obliterans D. laryngotracheomalacia E. antitrypsin deficiency

435. Parents of 3-year-old girl complain of rectal prolapse and failure to gain weight in spite of a good appetite. Patient has a history of recurrent prolonged respiratory infections and frequent, bulky, greasy stools. Sweet chloride is 126 mmol/l. What is the diagnosis? A. *cystic fibrosis B. antitrypsin deficiency C. Kartagener syndrome D. celiac disease E. Hirschprung’s disease

436. 2-year-old previously healthy boy had eaten peanuts and suddenly presents with an acute onset of cough, choking, and respiratory distress. Physical examination reveals a RR of 45 and wheezing, body temperature is normal. There is no history of asthma or allergic reactions, and no one at home is ill. What is the most likely diagnosis? A. *foreign body aspiration B. acute bronchiolitis C. attack of asthma D. acute laryngitis E. angioedema

437. A 6-year-old girl has had a dry cough without sputum for 2 months. The cough is getting worse after exercises and at night. Family history revealed that the parents have eczema. On physical examination, you hear a wheeze in both lung fields. She has none of the signs of chronic lung disease. What is the most likely diagnosis? A. *bronchial asthma B. pertussis C. foreign body aspiration D. bronchiectasis E. interstitial pneumonia

438. A 3 month child has vomiting, poor feeding, fever up to 38°C for the 2 days. His stool is frequent, with slime. Infant has lost 250 g of weight. He is pale, mucous are dry, fontanel is “fallen”, turgor is reduced, the abdomen is moderately inflated. The diagnosis of enterocolitis was made, dehydratation of 2nd degree. What solution is necessary to prescribe for rehydratation? A. *Rehydron B. 5 % glucose solution C. Boiled water D. Tea E. Broth of a camomile (medical)

439. The girl is 3 years old. She has admitted to hospital with the parents’ complaint on poorly walking. The excessive development of shoulder muscles is noticed, the lower extremities are poor developed, muscle hypotonia. The borders of relative heart dullness are extended to the left on 2 cm. There is systolic murmur in 2nd intercostal interspace on the right side of the sternum. Blood pressure on arms – 100/70, on legs – 40/20. Which diagnosis could be suspected?

A. *Coarctation of aorta B. Ventricle septal defect C. Atrium septal defect D. Tetralogy of Fallot E. myocarditis

440. A mother consults her 3 years old daughter by pediatrician. The child complains of hoarse voice, barking cough, laboured breathing with difficult inspiration. The infectious croup due to acute laryngitis is diagnosed. What anatomical feature predisposes a child to laryngeal stridor? A. *Narrow vocal slit B. Wide laryngeal lumen C. Watering-pot form of the larynx D. Poor vascularization of the mucous membrane E. Diaphragmatic type of respiration

441. The child is 3 months old. He has admitted to hospital to diagnose the reason for cardiac murmur. Complaints of parents: low weight gain, attacks of dyspnoea and cyanosis which amplify at a physical load. Systolic murmur in 3rd intercostal interspace on the left side, systolic murmur under 2nd intercostals interspace on the right side of the sternum, hypertrophia of right ventricle. What diagnosis should be suspected? A. *Tetralogy of Fallot B. infectious endocarditis C. ventricular septal defect D. hypertrophyc cardiomyopathy E. atrial septal defect

442. The child aged 12 yr complains of slight fever up to 38.00C, knee joints pain, a day before ankle pain, tenderness in active and passive movements, common weakness and cardiac pain. It is known he was ill with acute tonsillitis two weeks ago. Physical examination reveals left heart border external shift, non frequent cardiac premature bites. What disease should be suspected? A. *Rheumatic fever B. Now-rheumatic carditis C. Rheumatoid arthritis. D. Systemic lupus erythematosus E. Reactive arthritis

443. 8 years old girl had had a rheumatic fever manifested with chorea and carditis 6 months ago. She was treated in-patient department within 1,5 months. Now she is under long term observation by the rheumocardiologist in children out-patient department. The prevention therapy of rheumatic fever relapse foresees: A. *The administration of bicillin-5 every month during 5 years B. The administration of bicillin-5 every month during a year C. The administration of bicillin-5 every month during 3 years D. The administration of bicillin-3 every month during a year E. The administration of bicillin-3 every month during 3 years

444. 2 years old child has simple dyspepsia with nausea and vomiting. There are not signs of dehydration. he prevention of pathological fluid losses consists in the following administration: A. *Oral rehydration solution (ORS) B. Intravenous infusion 5\% glucose in water solution C. Starvation during 12 hours D. Antibacterial therapy E. Gastric lavage

445. The child aged 12 yr complains of cramping pain in the right hypochondrium which is easily controlled with antyspasmotic preparations. During attacks nausea and less often vomiting occur. Palpation of the abdomen reveals tenderness in the gallbladder projection point. The liver is not enlarged. What additional method of clinical investigation is the most informative in this case?

A. *Ultrasound examination B. Esophagogastroduedenoendoscopy C. CBC (complete blood count) D. X-Ray upper Gastrointestinal (GI) series E. Coprologic examination

446. A 11 month female has poor appetite, stool with plenty of faeces, periodically vomiting after introduction of solid food during last months. Temperature is normal. Bodyweight is 7 kg. On physical exam there are pale, oedema on legs, enlargement of abdomen. Coprogram shows a lot of greasy acids. The diagnosis of celiac disease was made, the aglutenic diet was prescribed. What is necessary to exclude from child diet? A. *Cereals B. Milk products C. Meat D. Egg E. Easy absorbed carbohydrates

447. A 2 year boy has subfebrile temperature, dry, persistent, prolonged, attacked cough, frequent breathing with hindered exhalation. The breathing under auscultation is harsh, there are diffuse dry sibilant rales. X-ray lung examination shows increased transparency. There is leukopenia in blood. What diagnosis is the most probable? A. *Obstructive bronchitis B. Pertussis C. Pneumonia D. Rhinitis E. Bronchiolitis

448. A 1,5 year old boy has non-productive cough with purulent sputum, dyspnea, retardation in physical development, polyfecalia, increasing of sweat chloride up to 150 mEq/l. The cystic fibrosis was diagnosed. What treatment will you prescribe? A. *Enzymes + antibiotics B. Choleretics + adaptogens C. Н2-histaminic blockaders + hepatoprotectors D. Vitamins + antibiotics E. Vitamins + mucolytics

449. Boy is 3 years old. Complaints: dyspnoe, fast tiredness, frequent episodes of respiratory diseases in history. Borders of relative heart dullness are extended to the left, strengthening of the 2nd heart sound in the 2nd intercostal interspace on the left side, hard systolo-dyastolic murmur in the second intercostal interspace on the left side and above the clavicle (“machine noise”), which is conducted on interscapular interspace. What is the most probable diagnosis? A. *patent arterial ductB. aortal stenosis C. atrial septal defect D. ventricle septal defect E. Isolated stenosis of arteria pulmonalis

450. The infant aged 2 months complaints of restlessness, subcutaneous fat wasting and underweight. The deficiency calculated from the ratio between the actual and average expected weight for his age is 14\%. The constipation and little amount of stools with undigested bits are noted. The child is in the breast feeding. The total day breast milk volume is not known. There is not evidence of infection in this case. What is the most probable diagnosis: A. *Hyponutrition in development B. Mucoviscidosis, intestinal form C. Acute gastroenteritis D. Chronic gastroduodenitis E. Dehydration

451. What kind of breathing can be heard above the lungs in healthy children aged 7 years?

A. *Vesicular B. Puerile C. Weakened vesicular D. Coarse E. Grunting

452. The girl is 5 years old. She has been hospitalised with the complaints on generalized oedema (face, abdomen, lower extremities), reduce of daily urine volume up to 300 ml, loss of appetite. In urinalysis – protein – 3 g/l, RBC – 1-2, WBC – 6-7, specific gravity – 1027. CBC – Hb – 110 g/l, WBC – 10 G/l. What diagnosis is most probable? A. *Glomerulonephritis with nephrotic syndrome B. Interstitial nephritis C. Pyelonephritis D. Polycystic kidney disease E. Glomerulonephritis with nephritic syndrome

453. The girl is 11 years old. She complains on pain in abdomen and lumbar region, headache, increase of temperature up to 39 C, vomiting, reduce of appetite. Skin is pale, hot, respiration is normal, heart rate – 100 per minute, positive Pasternatsky symptom. CBC: RBC – 3,9 T/l, WBC - 18 G/l, ESR-34 mm/hour. Urinalysis - protein 0,066 g/l, WBC – 30-40, RBC – 1-2, bacteria - a lot of. Most probable diagnosis is? A. *Acute pyelonephritis B. Acute glomerulonephritis C. Cystitis D. paranephritis E. Polycystic kidneys disease

454. The girl of 12 years old is suffering from acute glomerulonephritis. Daily diuresis is 700 ml, heart rate – 100 per minute, blood pressure – 130/95, temperature – 37,3 C. In urinalysis – RBC – 30-40, WBC – 5-6, protein – 0,099 g/l. Which medicines should be prescribed in this case? A. *Antibiotic, hypotensive, diuretic, vitamin C B. Antibiotics + prednisone C. Antihistaminic + vitamins D. Antiaggregants + diuretics E. Sulfanilamids + cytostatics

455. The girl is 12 years old. Yesterday she was overcooled. Now she complains on pain in suprapubic area, frequent painful urination by small portions, temperature is 37,8 C. Pasternatsky symptom is negative. Urinalysis – protein - 0,033 g/l, WBC – 20-25, RBC – 1-2. What diagnosis is most probable? A. *Acute cystitis B. Dysmetabolic nephropathy C. Acute glomerulonephritis D. Acute pyelonephritis E. Urolithiasis

456. The polycystic kidney disease was diagnosed at the boy of 3 years old. Mother complaints that the boy has growth retardation, poor appetite, vomiting. Skin is pale, turgor of soft tissues is reduced, heart rate - 120 per minute, harsh breathing at auscultation, abdomen is enlarged, soft. Biochemical tests – urea – 14 mmol/l, creatinine – 0,130 mmol/l, protein – 58 g/l. Which condition has been developed? A. *Chronic renal failure B. Acute renal failure C. Encephalopathy D. Interstitial nephritis E. Pyelonephritis

457. The boy of 9 years old. He is ill for 3 days. He has complaints on pain and restriction of movements in right knee and left elbow joints, dyspnoea. He was suffered from acute tonsillitis 2

weeks ago. There are fever (38,5 C), oedema of joints, extension of the borders of cordial dullness on 2 cm left, HR - 110 per 1 min, weakness of 1st sound, "soft" systolic murmur on an apex. What diagnosis should be suspected? A. *Acute rheumatic fever B. systemic lupus erythematodes C. juvenile rheumatoid arthritis D. Reiter’s disease E. reactive arthritis

458. The 7 years old boy developed an asphyxic attack, expiratory wheezing and cough. In past history the child has had relapsing rhinitis and red eyes syndrome. His sister suffers with atopic dermatitis. The correct diagnosis is: A. *bronchial asthma; B. acute bronchitis; C. viral croup; D. acute obstructive bronchitis; E. pneumonia.

Tests for pictures (тести до малюнків)

1. All of the following conditions are cause of glomerulonephritis (Fig. 89), except:A. *AllergiesB. Bacterial infectionC. ViralD. Fungal infectionE. Vaccination

2. Excretory urography (Fig. 87) allows evaluating:A. *Anatomical state of urinary tract and urodinamics B. State of nephronsC. Functional state of urinary tract D. State of urinary bladderE. Functional state of urodinamics

3. Excretory urography (Fig. 87) DOES NOT allow evaluating:A. Position, sizes, form of kidneysB. State of nephronsC. Functional state of urinary tract D. *Anomaly of kidney vesselsE. State of urinary bladder

4. What etiology of glomerulonephritis (Fig. 89) is the most often seen?A. *StreptococcusB. StaphylococcusC. E. cоlіD. Virus of hepatitis AE. Virus of flu

5. What is the main pathogenetical mechanism of development of glomerulonephritis (Fig. 89)?A. Bacterial inflammationB. Allergy reactionC. *Immunocomplex damaging D. Violation of passage of urineE. Reflux

6. Which sign does give information about the damaging of function of kidneys (Fig. 89)?A. HypoproteinemiaB. ProteinuriaC. *Asotemia

D. HyperlipidemiaE. Hyperglycemia

7. Secondary tubulopathy is observed at (Fig. 89):A. Phosphate-diabetesB. Tubulyar renal acidosisC. Disease de Toni-Debrae-FanconiD. Hereditary nephritisE. *Galaktozemia

8. Hereditary nephritis (Fig. 89) is inherited by:A. *Dominant type, connected with the X chromosomeB. Recessive type, connected with the X chromosomeC. Dominant type, connected with Y chromosomeD. Recessive type, connected with Y chromosomeE. Autosomal recessive type

9. Hereditary nephritis (Fig. 89) with deafness is called:A. Disease-de Toni- Debrae- FanconiB. Berger's diseaseC. *Alport syndromeD. Harnupa diseaseE. Leu syndrome

10. What is the etiologic factor of primary tubulopathy (Fig. 89)?A. MicrobsB. *Violation of membrane substances inrenal tubulesC. Reflux D. Anatomic anomaly of kidneyE. Insufficiency of podotcytes

11. At sick girl 7 years old impairment of urine filtration was found. The illness was manifested by high ever, sweating, aching pain in lumbar area, a discomfort in urination and frequent urination. What epartment of nephrone (Fig. 89) filtration of urine is performed in?A. Proximal ductuleB. Interstitium of kineyC. GlomerulusD. *Henle loopE. Distal ductule

12. A 14-year-old girl is seriously ill. She has chronic pyelonephritis with frequent exacerbations. xcretory urography (Fig. 87) should be prescribed to evaluate:A. Anatomic state of urinary ways and urine dynamicsB. State of the kidney’s pelvis systemC. Functional ability of urinary waysD. Sizes of kidneysE. *All is correct

13. The child of 8 years ill acute: temperature 39.5 C, pain in back, and the pain during urination, ymptom of Pasternatskiy positive from both sides, more in left. (Fig. 88). What disease is present?A. * Acute pielonephritisB. Acute viral infection. C. Acute cystitisD. Acute glomerulonephritisE. Kidney colic

14. ?When does opened ductus arteriosus will diagnose as congenital heart disease fig.40?A. If it not closed after 1 weak of life of the newbornB. If it not closed after 2weak of life of the newbornC. If it not closed after 3weak of life of the newborn

D. If it not closed after 4 weak of life of the newbornE. *If it not closed after 2 month of life of the newborn

15. What clinical symptom is the most typical to the patent ductus arteriosus fig 40?A. Systolic murmurB. Diastolic murmurC. *Systolic-diastolic murmurD. Presystolic murmurE. Prediastolic murmur

16. What pathological sine does appear from the beginning of patent ductus arteriosus fig 40?A. Hypertrophy of left ventricleB. Hypertrophy of right ventricleC. Hypertrophy of ventriclesD. *Lung’s hypertensionE. right ventricle failure

17. What kind of therapy of patent ductus arteriosus is the most effective fig 40?A. cardiac glycosidesB. Hypotensive medicinesC. prostaglandinsD. lock of b-receptorsE. *surgeon correction

18. What does appear due to decomposition of patent ductus arteriosus fig 40?A. left ventricle failureB. edema of lungC. cardiac asthmaD. *all that is mentionedE. nothing that is mentioned

19. Hypertrophy of what ventricle does appear from the beginning of ventricle septal defect fig. 41?A. Right ventricle B. *Left ventricle C. Right atrium D. Left atrium E. Aorta

20. Hypertrophy of what heart cavity does appear from the beginning of atrial septal defect fig. 41?A. Right ventricle B. Left ventricle C. *Right atrium D. Left atrium E. Aorta

21. Hypertrophy of what ventricle does appear due to patent ductus arteriosus fig. 41?A. *Right ventricle B. Left ventricle C. Right atrium D. Left atrium E. Aorta

22. Hypertrophy of what heart cavity does appear due to tetralogy of Fallot fig. 41?A. *Right ventricle B. Left ventricle C. Right atrium D. Left atrium E. Aorta

23. Hypertrophy of what heart cavity does appear from the beginning of transposition of grate vessels fig. 5?A. *Right ventricle

B. Left ventricle C. Right atrium D. Left atrium E. Aorta

24. Hypertrophy of what heart cavity does appear from the beginning of pulmonary arteria stenosis fig. 5?A. *Right ventricle B. Left ventricle C. Right atrium D. Left atrium E. Aorta

25. Hypertrophy of what heart cavity does appear from the beginning of aortal stenosis fig. 45?A. Right ventricle B. *Left ventricle C. Right atrium D. Left atrium E. Aorta

26. This pathology (Fig. 85) refers to:A. kidney multicystosisB. horseshoe kidneyC. anomalies of kidney location and formD. additional kidneyE. *aplasia, renal dysplasia

27. Hypertrophy of what heart cavity does appear from the beginning of coartation of aorta fig. 45?A. Right ventricle B. *Left ventricle C. Right atrium D. Left atrium E. Aorta

28. In what point of auscultation does systolic murmur due to coarctation of aorta have maximum sound ig. 7?A. Second intercostals space on a right side from sternumB. Second intercostals space on a left side from sternumC. Heart apexD. *Intrascapular spaceE. On a sternum

29. Due to what caned of congenital heart disease appear hypertrophy of left ventricle most quickly fig. 3? A. pulmonary arteria stenosis B. *aortal stenosis C. Тransposition of grate vesselsD. coarctation of aorta E. ventricle septal defect

30. Name the most typical clinical symptom of tetralogy of Fallot fig. 45?A. Cyanosis of skin and mucosal membrane B. tachycardiaC. Dispneae D. *Fit of cyanosis and dispneae E. edema

31. Leg in physical development of the child is most typical to…fig. 7A. ventricle septal defect B. tetralogy of Fallot C. aortal stenosis

D. *all that is mentionedE. nothing that is mentioned

32. In case of pulmonary stenosis we can see most probable… fig. 43 A. Left ventricle hypertrophy B. Isolated right atrium hypertrophyC. Both left and right ventricles hypertrophyD. *Right ventricle hypertrophy E. Isolated left atrium hypertrophy

33. What are the blood compensatory mechanisms in cyanotic defects fig. 58?A. *Polycythemia, hyperhemoglobinemia, high blood viscosityB. Leucocytosis with left shift, elevation of ESRC. Hypohemoglobinemia, low blood viscosity, erythrocytopeniaD. Thrombocytosis, high blood viscosityE. Leucocytopenia, Hypohemoglobinemia

34. What changes of EKG shows to us absolute necessity of surgery operation in aortic stenosis fig. 60?A. High R wave in V1, V2B. *Depression of S-T segment in V5, V6C. High R wave in V5, V6D. Depression of S-T segment in V1, V2E. High P wave in V1, V2

35. What examination is the most important for the patients with masked aortic stenosis fig 60?A. X-ray examinationB. ElectrocardiographyC. EchocardioscopyD. PhonocardiographyE. *Cardiac catheterization

36. In the case of aortic stenosis we can see most probably … fig. 42?A. Both left and right ventricles hypertrophyB. Isolated right atrium hypertrophyC. *Left ventricle hypertrophyD. Right ventricle hypertrophy E. Isolated left atrium hypertrophy

37. How can you characterize the patent ductus arteriosus fig. 58?A. *Cyanotic heart defect with right-to-left shuntB. Acquired heart diseaseC. Acyanotic heart defect with left-to-right shuntD. Complication of the congenital heart diseaseE. Acyanotic heart defect without shunt

38. At what congenital defect can we see round, apple-shaped heart during X-ray examination fig.43?A. tetralogy of FallotB. Pulmonary stenosisC. Atrial septal defectD. Aortic stenosisE. *Tricuspid atresia

39. Choose the most right assertion fig. 63:A. During indirect heart massage the frequency of sternum compression is 40-60 times per 1

minute, fter 1 inspiration we make 4 compressions B. During indirect heart massage te frequency of sternum compression is 60-100 times per 1

minute, after 1 inspiration we make 2 compresions C. *During indirect heart massage the frequency of sternum compression is 60-100 times per

1 minute, fter 1 inspiration we make 4 compressions D. During indirect heart massage the frequency of sternum compression is 60-100 times per 1

minute, fter 1 inspiration we make 1 compressions E. During indirect heart massage the frequency of sternum compression is 40-60 times per 1

minute, fter 1 inspiration we make 2 compressions 40. Give a conclusion about the reslts of ECG fig. 46.

A. *Ventricle tachycardiaB. Weakness of ventricle with disorded retractive ability C. Complete AV blockadeD. Ventricular extrasystoleE. Supraventricular tachycardia

41. Give a conclusion about the results of ECG fig. 47.A. Widespread QRS tachycardia B. Weakness of ventricle with disordered retractive ability C. *Complete AV blockadeD. Ventricle tachycardiaE. Supraventricular tachycardia

42. Give a conclusion about the results of ECG fig. 48.A. Disordered atrio-ventricular conduction 2:1B. Disordered atrio-ventricular conduction 3:1C. *Disordered atrio-ventricular conduction 4:1D. Cardiac flutterE. Sinus tachycardia

43. Give a conclusion about the results of ECG fig. 49A. Disordered atrio-ventricular conduction 2:1B. Disordered atrio-ventricular conduction 3:1C. Disordered atrio-ventricular conduction 4:1D. *Disordered atrio-ventricular conduction 1:1E. Cardiac fibrillation

44. Give a conclusion about the results of ECG fig. 50A. Disordered atrio-ventricular conduction 3:1B. Disordered atrio-ventricular conduction 4:1C. Sinus tachycardia D. Cardiac fibrillation E. *Wolff- Parkinson-White-syndrome

45. Give a conclusion about the results of ECG fig. 51A. Disordered atrio-ventricular conduction 4:1B. *Supraventricular tachycardiaC. Ventricle tachycardiaD. Sinus tachycardia E. Cardiac fibrillation

46. What indicate medicine of the first chose for treatment of this pathology fig. 51A. LidocainumB. NovokainomidC. AjmalineD. KordaronE. *Isoptinum

47. Give a conclusion about the results of ECG fig. 52.A. Disordered atrio-ventricular conduction 4:1B. Supraventricular tachycardiaC. Ventricle tachycardiaD. *Sinus tachycardia E. Cardiac fibrillation

48. Give a conclusion about the results of ECG fig. 53A. Disordered atrio-ventricular conduction 4:1

B. Supraventricular tachycardiaC. Ventricle tachycardiaD. *Sinus tachycardia E. Cardiac fibrillation

49. What medical manipulations will be adequate in this case fig. 53? A. *Removal (sublation) of the reasonB. Sedatives C. Antiarrhythmical medicinesD. Cardiac glycosidesE. potassium contained medicines

50. Give a conclusion about the results of ECG fig. 54A. Disordered atrio-ventricular conduction 4:1B. Supraventricular tachycardiaC. *Ventricle tachycardiaD. Sinus tachycardia E. Cardiac fibrillation

51. WhatIndicate medicine of the first chose for treatment of this pathology fig. 54A. *LidocainumB. NovokainomidC. AjmalineD. KordaronE. Isoptinum

52. Give a conclusion about the results of ECG fig. 55.A. *Cardiac fibrillation B. Supraventricular tachycardiaC. Ventricle tachycardiaD. Sinus tachycardia E. Supraventricular tachycardia

53. What congenital heart defect is following with frequent infections of lower respiratory tract fig. 62?A. Coarctation of the aortaB. *Patent ductus arteriosusC. Tetralogy of Fallot D. Pulmonary valve stenosis E. Persistent truncus arteriosus

54. Which disorder is characterized by systolic ejection murmur in the pulmonic area and mid-diastolic urmur in the lower right sternal area fig. 7?A. Ventrical septal defectB. Patent ductus arteriosusC. Tetralogy of FallotD. Atrial septal defectE. *Pulmonary valve stenosis

55. What symptom is occurred in children with tetralogy of Fallot fig. 7?A. The murmur is harsh and is best heard at the midsternal or lower left sternal boder.B. A systolic and diastolic ejection murmur is heard in the pulmonic area.C. *A systolic ejection murmur is in the pulmonic areaD. A harsh systolic ejection murmur is heard along the sternal boderE. A diastolic ejection murmur is in the pulmonic area

56. What features is occurred on the chest x-ray in children with patent ductus arteriosus fig. 62?A. The heart and main pulmonary artery segment are enlargedB. The chest radiograph may be normalC. *Cardiomegaly, increased pulmonary vascularity and enlargement of the left atrium and

left ventricle re seen

D. The heart size is normal. The apex is uptilted, and a concavity is noted in the pulmonary segment, hich gives the heart the appearance of a boot

E. Notching of the fourth through eighth ribs57. What congenital heart defect is following with frequent infections of lower respiratory tract fig.

62?A. Persistent truncus arteriosusB. Tetralogy of FallotC. *Atrial septal defectD. Aortic valve and discrete subaortic stenosisE. Coarctation of the aorta

58. What symptom is occurred in children with atrial septal defect fig. 7?A. The murmur is harsh and is best heard at the midsternal or lower left sternal boderB. *A systolic and diastolic ejection murmur is heard in the pulmonic areaC. A harsh systolic ejection murmur is heard along the sternal boderD. A systolic ejection murmur is in the pulmonic areaE. A diastolic ejection murmur is in the pulmonic area

59. What feature is occurred on the chest x-ray in children with tetralogy of Fallot fig. 9?A. the heart and main pulmonary artery segment are enlargedB. the chest radiograph may be normal C. cardiomegaly, increased pulmonary vascularity and enlargement of the left atrium and left

ventricle re seen.D. *the heart size is normal; the apex is uptilted, and a concavity is noted in the pulmonary

segment, hich gives the heart appearance of the bootE. Notching of the fourth throug eighth ribs

60. Which of the lesion are characterized of tetralogy of Fallot fig. 84?A. *pulmonary atresia with ventricular septal defectB. left ventricular hypertrophyC. coarctation of the aortaD. ventricular septal defectE. atrial septal defect

61. What congenital heart defect is following with frequent infections of lower respiratory tract fig. 62?A. Tetralogy of FallotB. Pulmonary valves stenosisC. Coarctation of the aortaD. *Ventricular septal defectE. Transposition of the great arteries

62. What symptom is occurred in children with patent ductus arteriosus fig.7?A. A systolic and diastolic ejection murmur is heard in the pulmonic areaB. *The murmur is harsh and is best heard at the midsternal or lower left sternal borderC. A systolic ejection murmur is in the pulmonic areaD. A harsh systolic ejection murmur is heard along the sternal borderE. A diastolic ejection murmur along the sternal border

63. What feature is occurred on the chest X-ray in children with atrial septal defect fig. 9?A. *cardiomegaly, increased pulmonary blood flow and enlargement of the left atrium and

left ventricle re seenB. the heart and main pulmonary artery segment are enlargedC. the chest radiograph may be normal D. the heart size is normal; the apex is uptilted, and a concavity is noted in the pulmonary

segment, hich gives the heart appearance of the boot.E. notching of the fourth througheighth ribs

64. What lesion of tetralogy of Fallot is physiologically important fig. 84?A. * variable right ventricular outflow tract obstruction

B. dextraposition of the aortaC. ventricular septal defectD. right ventricular hypertrophyE. Mixed blood flow throw aorta

65. During which congenital structural disorders does pulmonary blood flow increased fig. 84?A. Coarctation of the aortaB. patent ductus arteriosusC. Tetralogy of FallotD. Pulmonary valves stenosisE. *Persistent truncus arteriosus

66. What congenital heart defect is following with systolic ejection murmur in pulmonary area fig. 7? A. Ventricular septal defectB. *patent ductus arteriosusC. Coartation of the aortaD. Tetralogy of FallotE. Pulmonary valves stenosis

67. What symptom is occurred in children with ventricular septal defect fig. 7?A. *The murmur is harsh and is best heard at the midsternal or lower left sternal borderB. A systolic ejection murmur is in the pulmonary areaC. A systolic and diastolic ejection murmur is heard in the pulmonary areaD. A harsh systolic ejection murmur is heard along the sternal borderE. A systolic ejection murmur is in the aortal area

68. What feature is occurred on the chest X-ray in children with coarctation of the aorta fig. 9?A. the heart and main pulmonary artery segment are enlargedB. the chest radiograph may be normalC. cardiomegaly, increased pulmonary blood flow and enlargement of the left atrium and left

ventricle re seenD. the heart size is normal; the apex is up tilted, and a concavity is noted in the pulmonary

segment, hich gives the heart appearance of the boot.E. * notching of the fourth througheighth ribs

69. During which congenital structural disorders is pulmonary blood flow increased fig.84?A. Tetralogy of FallotB. Coarctation of the aortaC. *Ventricular septal defect D. Pulmonary valves stenosisE. Transposition of the great arteries

70. ?Which of the following assertion is wrong (fig. 80)?A. The most intense pancreas is increasing in the first 3 yearsB. Glandular apparatus of the pancreas is not enough differentiated after birthC. The development the liver parenchyma ends to 8 yearsD. *Endocrine function of the pancreas is less developed, than exocrineE. Children under 1 year of the high content of taurocholic bile acid

71. What infectious disease does most frequently cause inflammatory changes in the tissue of the ancreas (fig. 80)?A. *MmpsB. ChickenpoxC. Viral hepatitisD. Streptococcal infectionE. Measles

72. Signs of hypotonic form of bile ducts dyskinesia at the duodenal probing are (fig. 80):A. Increase of portion A volumeB. *Increase of portion B volumeC. Increase of portion С volume

D. Decrease of portion B volume E. Decrease of portion A volume

73. Signs of hypertonic form of bile ducts dyskinesia at the duodenal probing are (fig. 80):A. Increase of portion A volumeB. Increase of portion B volumeC. Increase of portion С volumeD. *Decrease of portion B volume E. Decrease of portion A volume

74. Up to what age in healthy children in the supine position the lower edge of the liver can be palpated fig.82)? A. Up to 3 years B. Up to 5 years C. *Up to 7 years D. Up to 10 years E. Up to 12 years

75. In children of the first three years lower edge of liver (on a mid clavicle line) (fig. 82):A. Does not extends from the rib archB. Extends from the rib arch up to 0,5-1 cmC. Extends from the rib arch up to 1-2 сm D. *Extends from the rib arch up to 2-3 сmE. Extends from the rib arch up to 3-5 сm

76. At the healthy children of what age does not palpated lower edge of liver (on a mid clavicle line) (fig. 2)?A. From 1 yearB. From 3 yearsC. From 5 yearsD. *From 7th yearsE. From 10 years

77. In neonates the liver has the following features (fig.82): A. *Weight is 4-6% of body weight B. Parenchyma is little differentiated C. Sanguineous D. A distinct lobed structure E. All transferred

78. For dyspepsia syndrome in case of bile ducts pathology is typically (fig.83):A. Diarrhea after the use of milkB. Diarrhea is frequent, profuseC. Stabile constipation from the birthD. *Periodical constipationE. Diarrhea before the use of milk

79. In case of hypotonic biliary dyskinesia is typically (fig.83):A. *Discomfort in right subcostal areaB. Hungry, nightly stomach-acheC. Intensive, paroxysmal pain in right subcostal areaD. Distension pain in the mesogastriumE. Discomfort in left subcostal area

80. In case of hypertonic biliary dyskinesia is typically (fig.83):A. Discomfort in right subcostal areaB. Hungry, nightly stomach-acheC. *Intensive, paroxysmal pain in right subcostal areaD. Distension pain in the mesogastriumE. Discomfort in left subcostal area

81. For chronic pancreatitis are typical (fig. 79):

A. Hepatomegaly, jaundice, periodic constipationB. Splenomegaly, jaundice, diarrheaC. Local pain in a pylorodyodenal area, anemia, constipationD. *Girdling abdominal pain, anemia, diarrheaE. Discomfort in left subcostal area

82. What complex of investigations is needed for patients with the diseases of hepatobiliary system (fig. 0)?A. РH-metry, ultrasonography, duodenal probingB. РH-metry, EFGDS, ultrasonographyC. EFGDS, ultrasonography, laparoscopyD. *Laparoscopy, ultrasonography, duodenal probingE. EFGDS, laparoscopy

83. For the depression of the external secretion function of the pancreas is better to give (fig.79):A. TramadolB. TavegilC. *AtropineD. No-spaE. Heparin

84. For liquidation of pain in the patient with pancreatitis is prescribed (fig. 79)::A. *BaralginB. AspirinC. IndomethacinD. CaffeineE. Papaverin

85. For treatment of the acute pancreatitis attack is prescribed (fig. 79):A. PenicillinB. HeparinC. *ContricalD. VicasolE. Voltaren

86. For medicinal suppression of the pancreas is used (fig. 79):A. *RanitidinB. AlmagelC. CreonD. LinexE. VicasolF. t in the first days in case of the pancreatitis attack is (fig. 79):G. HungerH. Diet № 1I. Diet № 5J. Diet № 7K. Diet № 8

87. Which of the following assertion is wrong (fig. 79)?A. The most intense pancreas is increasing in the first 3 yearsB. Glandular apparatus of the pancreas is not enough differentiated after birthC. The development the liver parenchyma ends to 8 yearsD. *Endocrine function of the pancreas is less developed, than exocrineE. Children under 1 year of the high content of taurocholic bile acid

88. The pancreas of the newborn has the following features (fig. 79):A. Enough differentiated parenchymaB. Much vascularizedC. Has a poor connective tissueD. *The head of pancreas is most developed

E. Detoxification organ89. In neonates the liver has the following features (fig. 82):

A. *Weight is 4-6% of body weight B. Parenchyma is little differentiated C. Sanguineous D. A distinct lobed structure E. All transferred

90. What type of biliary dyskinesia does occur with the predominance of sympatic tone of the nervous ystem of patients (fig. 83)?A. HypertonicB. NormotonicC. *HypotonicD. PrimaryE. Secondary

91. Pain syndrome, dyspeptic syndrome, hepatomegaly, positive gallbladder symptoms are characterized or (fig. 83):A. HypertonicB. NormotonicC. *HypotonicD. PrimaryE. Secondary

92. The first phase of fractional duodenal intubation is (fig. 80):A. The selection of hepatic bile B. A time of closed sphincter of OddiC. *Choledochic D. A time of the opening of the sphincter of OddiE. A period of emptying of the gall bladder

93. The second phase of fractional duodenal intubation is (fig. 80):A. The selection of hepatic bile B. *A time of closed sphincter of OddiC. Choledochic D. A time of the opening of the sphincter of OddiE. A period of emptying of the gall bladder

94. The third phase of fractional duodenal intubation is (fig. 80):A. The selection of hepatic bile B. A time of closed sphincter of OddiC. Choledochic D. *A time of the opening of the sphincter of OddiE. A period of emptying of the gall bladder

95. The fourth phase of fractional duodenal intubation is (fig. 80):A. *A period of emptying of the gall bladderB. The selection of hepatic bile C. A time of closed sphincter of OddiD. Choledochic E. A time of the opening of the sphincter of Oddi

96. The fifth phase of fractional duodenal intubation is (fig. 80):A. A period of emptying of the gall bladderB. *The selection of hepatic bile C. A time of closed sphincter of OddiD. Choledochic E. A time of the opening of the sphincter of Oddi

97. Cholangiography is used for diagnose of (fig. 83):A. Parenchimatous jaundice

B. Hemolytic jaundiceC. *Obstructive jaundiceD. Conjugation jaundiceE. All types of jaundice

98. What type of cholangiography is absent (fig. 83)?A. Percutaneous transhepatic cholangiographyB. Intraoperative cholangiographyC. Endoscopic retrograde cholangiopancreatographyD. Magnetic resonance cholangiopancreatographyE. *Contrast cholangiography

99. What is not used for diagnostic of cholecystitis (fig. 83)?A. UltrasoundB. CholescintigraphyC. CT ScanD. CholangiographyE. *All are used

100. ?Which pathology is depicted on the Fig. 85?A. *Left kidney aplasiaB. Left kidney hypoplasiaC. Doubling right kidneyD. Left kidney dystopiaE. Right kidney dystopia

101. Which the main complication of this pathology (Fig. 86) do you know?A. polycystic kidneyB. glomerulonephritisC. primary pyelonephritisD. *hydronephrosisE. phosphate diabetes

102. Which pathology is represented on the Fig. 86?A. Left kidney aplasiaB. Left kidney hypoplasiaC. Doubling right kidneyD. Left kidney dystopia E. *Horseshoe kidney

103. Which investigation is the most effectiveness for diagnostic this pathology (Fig. 86)?A. Ultrasound examinationB. *Excretory urographyC. CystoscopyD. CystografiaE. Thermography

104. For diagnostic of this pathology (Fig. 85) you can use all methods, exceptA. Excretory urographyB. Ultrasound examinationC. Computer tomographyD. *CystoscopyE. MR tomography

105. This examination (Fig. 87) gives information aboutA. anatomical status of urinary tract and urodynamicB. state of pelvis of the kidney C. functional ability of urinary tractD. kidney’s sizeE. *all right

106. What is the name of this examination (Fig. 87)?

A. *Excretory urographyB. Ultrasound examinationC. Computer tomographyD. CystoscopyE. MR tomography

107. What is the main contraindication for this examination (Fig. 87)?A. PyelonephritisB. Persistent isolated urinary syndromeC. HypertensionD. *Terminal chronic renal failureE. Tumor in the abdominal cavity or retroperitoneal space

108. What is the name of this symptom (Fig. 88)?A. BlyooberhB. VoskresenskyC. BabinskiD. OrtnerE. *Pasternatsky.

109. For which disease is positive this symptom (Fig. 88)?A. Kidney aplasia B. *PyelonephritisC. CystitisD. Kidney dystopiaE. Enuresis

110. The structure of this organ (Fig. 89) you can define usingA. CystographyB. CystoscopyC. Nechiporenko’s testD. Endogenous creatinine clearance E. *Ultrasound examination

111. Which structures of this organ (Fig. 89) are damaged by pyelonephritis?A. glomerulusB. *channels of kidneyC. mucous membrane of the ureterD. kidney capsuleE. glomerular capillaries.

112. How long is continues antibacterial therapy of inflammation process of this organ (Fig. 89)?A. 7-14 daysB. 14-21 daysC. *before the complete clinical and laboratory remissionD. within 6 monthsE. before the partial clinical-laboratorial remission

113. Put diagnosis in patient according to chest X-ray on fig. 1:A. *Destructive pneumoniaB. BronchopneumoniaC. Interstitial pneumoniaD. Lobar pneumonia E. Segmental pneumonia

114. Choose the most possible etiological agent of pneumonia on fig. 1:A. Esherichia coliB. *St. aureusC. Klebsiella pneumoniaeD. Strep. Pneumoniae E. Hemophylus influenzae

115. Choose dosage of Penicillin group antibiotic for treatment of pneumonia on fig. 1.A. 80-100 000 IU/kg/ dayB. 100-120 000 IU/kg/ day C. 120-150 000 IU/kg/ dayD. 150-200 000 IU/kg/ dayE. *200-300 000 IU/kg/ day

116. Put diagnosis in patient according to chest X-ray on fig.2:A. Destructive pneumoniaB. BronchopneumoniaC. Pneumothorax D. *Pleuritis E. Abscess

117. What complication of pneumonia is shown on fig. 2:A. Pneumothorax B. AbscessC. *Pleuritis D. Infective endocarditisE. Infectious-toxic shock

118. Put diagnosis in patient according to chest X-ray on fig. 3:A. Destructive pneumonia B. *Lower lobar pneumoniaC. Interstitial pneumoniaD. Pneumothorax E. Pleuritis

119. Put diagnosis in patient according to chest X-ray on fig. 3:A. Pleuritis B. Empyema C. *Lower lobar pneumoniaD. Parapneumonic effusions E. Pneumothorax

120. Choose dosage of Penicillin group antibiotic for treatment of pneumonia on fig.3:A. 80-100 000 IU/kg/ dayB. 50-90 000 IU/kg/ dayC. *120-150 000 IU/kg/ dayD. 500-1000 IU/kg/ dayE. 250-300 000 IU/kg/ day

121. What clinical signs are typical for pneumonia on fig.3:A. Scattered coarse ralesB. Scattered high-pitched wheezingC. Barking cough D. *Tachypnea E. Hoarse voice

122. Put diagnosis in patient according to chest X-ray on fig. 4:A. *EmpyemaB. Destructive pneumoniaC. Lower lobar pneumoniaD. Interstitial pneumoniaE. Pneumothorax

123. What complication of pneumonia is shown on fig. 4:A. AbscessB. *PleuritisC. Infective endocarditisD. Infectious-toxic shock

E. Pneumothorax 124. Choose the most possible etiological agent of complicated pneumonia in children on fig. 4.

A. *Strep. Pneumoniae B. St. aureusC. Esherichia coliD. Klebsiella pneumoniaeE. Hemophylus influenzae

125. What clinical signs are typical for pathological condition on fig. 4:A. Scattered coarse ralesB. Scattered high-pitched wheezingC. Inspiratory and expiratory wheezingD. Tympanic on percussionE. *Dullness on percussion

126. Put diagnosis in patient according to chest X-ray on fig.5:A. Destructive pneumoniaB. EmpyemaC. Polysegmental pneumoniaD. Interstitial pneumoniaE. Pneumothorax

127. Choose the most possible etiological agent of pneumonia in preschool children on fig.5. A. *Strep. PneumoniaeB. St. aureusC. Esherichia coliD. Klebsiella pneumoniaeE. St. epidermitis

128. Choose the most possible etiological agent of pneumonia in children 1-6 months on fig.5: A. Strep. PneumoniaeB. *St. aureusC. Esherichia coliD. Klebsiella pneumoniaeE. Mycoplasma pneumoniae

129. Choose the most possible etiological agent of nosocomial pneumonia in children on fig.5: A. *Ps. aeruginosaB. Strep. PneumoniaeC. St. aureusD. Esherichia coliE. Mycoplasma pneumoniae

130. What pathology is revealed on chest X-ray on fig. 6: A. Right lobar pneumoniaB. Left side pyothoraxC. Right side pneumothoraxD. Left hemorrhagic pleuritisE. Right side abscess

131. What complication of pneumonia is shown on fig. 6:A. AbscessB. *Pneumothorax C. Infective endocarditisD. PleuritisE. Empyema

132. Put diagnosis in patient according to chest X-ray on fig.6:A. Right side pleuritis B. Lower lobar pneumonia on the left side C. Parapneumonic effusions on the left side

D. *Pneumothorax on the right sideE. Abscess on the left side

133. What clinical signs are typical for pneumonia on fig. 7A. Inspiratory and expiratory wheezingB. Scattered high-pitched wheezingC. Diminished breath sound D. Scattered dry ralesE. Tympanic on percussion

134. What clinical signs are typical for acute bronchitis on fig. 7 A. FeverB. TachypneaC. Backy cough D. *Coarse breath sound E. Prolongation of the expiratory of breathing

135. What clinical signs are typical for acute bronchiolitis on fig. 7A. Runny noseB. StridorC. Backy cough D. DroolingE. *Prolonged expiratory

136. What clinical signs are typical for bronchial asthma on fig.7A. Stridor B. HoarsenessC. Inspiratory and expiratory wheezingD. Diminished breath sound E. Barky cough

137. Put diagnosis in patient according to chest X-ray on fig. 8:A. Aspiration pneumoniaB. *Lobar pneumoniaC. BronchopneumoniaD. Interstitial pneumoniaE. Segmental pneumonia

138. What is the possible etiological agent of lobar pneumonia on fig. 8:A. Ps. aeruginosaB. Strep. PneumoniaeC. Mycoplasma pneumoniae D. Esherichia coliE. *St. aureus

139. What are clinical signs of pathology on fig 8:A. Expiratory whistling sound, wheezingB. Stridor, barky coughC. Runny nose, droolingD. Muffled voice, sore throatE. *Diminished breath, tachypnea

140. Choose the most possible diagnosis on fig.8: A. Abscess B. Interstitial pneumoniaC. PneumothoraxD. *Lobar pneumoniaE. Pleuritis

141. What examination is shown on fig.9?A. Chest X-rayB. MRI examination

C. Ultrasound examination of lungsD. CT of lungsE. *UVB-therapy

142. What is the purpose of such method on fig.9?A. To diagnose pneumoniaB. To diagnose pneumonia complicationsC. To treat pneumonia in flare-up period D. *To treat pneumonia in recovery periodE. To treat complications of pneumonia

143. Choose the right diagnosis on fig.10:A. *Right side emphysema B. Right lobar pneumoniaC. Right side pneumothoraxD. Left side pleuritisE. Left lobar pneumonia

144. What objective changes can be revealed on pathology on fig 10?A. *Tympanic sound on the right side on percutionB. Dullness on the right side on percutionC. Ban-box sound on left side on percutionD. Clear lung sound on the left side on auscultationE. Scattered high-pitched wheezing on auscultation

145. What objective changes can be revealed on pathology on fig. 10?A. Ban-box sound on the right side on percutionB. Diminished breath sound on the right sideC. Decreased relative dullness of heartD. Low diaphragm dispositionE. *Everything correct

146. What measures such tool on fig. 11?A. *Speed and volume of breath outB. Concentration of О2 during breathingC. Concentration of СО2 during expirationD. Supplement with oxygen E. All correct

147. What is shown on fig.11?A. Inhalation therapyB. *SpirometryC. Оxygen-therapyD. UVB-therapyE. Pulse oximetry

148. At what age can be performed such examination on fig.11?A. First month B. First yearC. *after 5 yearsD. after 4 yearsE. after fourteen years

149. Choose typical sign of pneumonia in newborn on fig.12:A. Rhythm breathing disorder B. Decreased activity C. TachypneaD. Nostril flaringE. *All correct

150. Choose the most common sign of congenital pneumonia on fig.12:A. Tachypnea

B. CyanosisC. Retraction D. GruntingE. *All correct

151. Choose the most possible etiological agent of congenital pneumonia on fig. 12:A. Chlamydia pneumoniaB. St. aureusC. Group B StreptococciD. Strep. Pneumoniae E. Hemophylus influenzae

152. Put the diagnosis according the chest X-ray (Fig. 27):A. BronchoectasisB. *Left lung hypoplasiaC. Polycystic lung diseaseD. Chronic lung diseaseE. Lung tumor

153. Put the diagnosis according the tomogram (Fig. 28):A. Lung abscessB. Lung tumorC. Lung hypoplasiaD. *Bronchogenic cystE. Lung sequestration

154. Put the diagnosis according the chest X-ray (Fig. 29):A. Lung abscessB. Lung tumorC. Lung hypoplasiaD. Lung sequestrationE. *Congenital cyst

155. Name is the method of examination (Fig. 30):A. BronchographyB. *AortographyC. AngiographyD. Contrast tomographyE. Cystocholangiography

156. This method is used for the diagnostics of (Fig. 30):A. *Lung sequestrationB. BronchoectasisC. Liver diseasesD. PneumoniaE. Chronic lungs diseases

157. What disease is schematically pictured in the Fig. 31?A. Hamman-Rich syndromeB. Williams-Campbell syndromeC. *Mounier-Kuhn syndromeD. Kartagener syndromeE. Cystic fibrosis

158. What is the main method of the treatment of disease which is pictured in Fig. 31?A. Surgical resectionB. Bronchus drainageC. PhysiotherapyD. *Complex conservative treatmentE. There is no treatment

159. Bronchoscopy and bronchography (Fig. 32) isn’t allow to diagnose:

A. Tracheo-esophageal fistulaB. Williams-Campbell syndromeC. Mounier-Kuhn syndromeD. Kartagener syndromeE. *Hamman-Rich syndrome

160. Bronchoscopy (Fig. 32) is used in children:A. After 1 month of life onlyB. After 1 year of life onlyC. After 5 years of life onlyD. After 10 years of life onlyE. *In any age

161. What disease is schematically pictured in the Fig. 33?A. Hamman-Rich syndromeB. *Williams-Campbell syndromeC. Mounier-Kuhn syndromeD. Kartagener syndromeE. Cystic fibrosis

162. What examination is the most useful for the diagnosis of this disease (Fig. 33)?A. *BronchographyB. BronchoscopyC. Chest X-rayD. SpirographyE. CT-scan

163. The localization of the bronchoectasises in the lower parts of lungs (Fig. 34) is typical for:A. Hamman-Rich syndromeB. *Williams-Campbell syndromeC. Mounier-Kuhn syndromeD. Kartagener syndromeE. Cystic fibrosis

164. Put the diagnosis according the chest X-ray (Fig. 35):A. Hamman-Rich syndromeB. Williams-Campbell syndromeC. Mounier-Kuhn syndromeD. *Kartagener syndromeE. Cystic fibrosis

165. What congenital chronic lung disease is pictured in the Fig. 35?A. Hamman-Rich syndromeB. Williams-Campbell syndromeC. Mounier-Kuhn syndromeD. *Kartagener syndromeE. Cystic fibrosis

166. The diffuse lungs tissue abnormalities presented in the Fig. 36 are typical for:A. *Hamman-Rich syndromeB. Williams-Campbell syndromeC. Mounier-Kuhn syndromeD. Kartagener syndromeE. Cystic fibrosis

167. The diffuse pathological changes in the bronchial glands presented in the Fig. 37 are typical for:A. Hamman-Rich syndromeB. Williams-Campbell syndromeC. Mounier-Kuhn syndromeD. Kartagener syndromeE. *Cystic fibrosis

168. In the treatment of cystic fibrosis (Fig. 37) the most important is:A. AntibioticsB. MucolyticsC. *EnzymesD. MassageE. Immunomodulators

169. The pathological changes of the hands presented in the Fig. 38 are typical for:A. Rheumatoid artritisB. Rheumatic feverC. *Chronic hypoxiaD. DermatomyositisE. Nephrotic syndrome

170. What diseases can cause the pathological changes of the hands presented in the Fig. 38?A. *Chronic lungs diseasesB. Systemic connective tissue diseasesC. Kidneys diseasesD. Chromosomal diseasesE. All mentioned above

171. The chest deformation presented in the Fig. 39 is typical for:A. Chronic bronchitisB. Congenital lungs diseaseC. Cystic fibrosisD. Chronic acquired lungs diseaseE. *All mentioned above

172. The chest deformation presented in the Fig. 39 is NOT typical for:A. Chronic bronchitisB. Congenital lungs diseaseC. Cystic fibrosisD. *RicketsE. Chronic acquired lungs disease

173. In the Fig. 64 are presented skin changes on the face which are typical for the:A. Systemic sclerodermaB. DermatomyositisC. *Systemic lupus erhythematosusD. Rheumatoid artritisE. Systemic vasculitis

174. Besides of typical skin changes on the face (Fig. 64) for the systemic lupus erhythematosus is haracteristic:A. *Polyserositis and nephritisB. Artritis with joints destructionC. Myositis and nephritisD. Myositis and artritisE. Hepato- and splenomegaly and intoxication

175. Such appearance of the patient (Fig. 65) the most probably is the result of:A. HeredityB. Chromosomal diseaseC. ObesityD. *Prolonged glucocorticoids intakeE. Nephrotic syndrome

176. What syndrome is presented in the Fig. 65?A. NephroticB. ObesityC. Prader – Willey syndrome

D. Hypothalamic syndromeE. *Cushing syndrome

177. What is reason of the appearance of the patient in the Fig. 65?A. OvereatingB. Constitutional peculiaritiesC. *HyperadrenocorticismD. Congenital metabolic diseaseE. Lack of physical activity

178. In the Fig. 66 are presented face changes (periorbital edema and heliotrope rush) which are typical or:A. Systemic sclerodermaB. *DermatomyositisC. Systemic lupus erhythematosusD. Rheumatoid artritisE. Systemic vasculitis

179. What is the name of symptom presented in the Fig. 66?A. Gottron patchesB. “Butterfly” symptomC. Annular erythemaD. *Heliotropic rashE. Livedo reticularis

180. The subcutaneous nodulas presented on the Fig. 67 are typical for the:A. Systemic sclerodermaB. DermatomyositisC. Systemic lupus erhythematosusD. *Rheumatoid artritisE. Systemic vasculitis

181. In the rheumatoid nodules (Fig. 67) besides of RF can be founded: A. Lymphocytes accumulationB. CollagenC. *Immune complexesD. CalcificatesE. All mentioned above

182. The symmetrical proximal muscles weakness which is indicated in the Fig. 69 is typical for:A. Systemic sclerodermaB. *DermatomyositisC. Systemic lupus erhythematosusD. Rheumatoid artritisE. Systemic vasculitis

183. Patient with the symmetrical proximal muscles weakness in dermatomyositis (Fig. 69) should be ndergone a: A. Magnetic resonance imaging (MRI) B. Biochemical blood testC. EMG D. Muscle biopsyE. *All mentioned above

184. Deformations of the wrist joints are the most typical for (Fig. 70):A. Systemic sclerodermaB. DermatomyositisC. Systemic lupus erhythematosusD. *Rheumatoid artritisE. Systemic vasculitis

185. Uveitis and iritis (Fig. 70) is the typical for:

A. Systemic sclerodermaB. DermatomyositisC. Systemic lupus erhythematosusD. *Rheumatoid artritisE. Systemic vasculitis

186. Ophtalmologist consultation for this patient (Fig. 70) is obligatory in:A. Systemic sclerodermaB. DermatomyositisC. Systemic lupus erhythematosusD. *Rheumatoid artritisE. Systemic vasculitis

187. Gottron’s patches on the skin (Fig. 72) are typical for the A. Systemic sclerodermaB. *DermatomyositisC. Systemic lupus erhythematosusD. Rheumatoid artritisE. Systemic vasculitis

188. The prolonged glucocorticoids intake can cause (Fig. 65):A. Adrenogenital syndromeB. Waterhouse-Friderichsen syndromeC. Prader – Willey syndromeD. Hypothalamic syndromeE. *Cushing syndrome

189. Swelling of the fingers, hands with thickening, hardening, and discoloration of the skin (“nonliving, rtificial” skin) (Fig. 68) is typical for the:A. *Systemic sclerodermaB. DermatomyositisC. Systemic lupus erhythematosusD. Rheumatoid artritisE. Systemic vasculitis

190. What ayes complications are typical for the JRA (fig. 71):A. ConjunctivitisB. BlepharitisC. *UveitisD. GlaucomaE. All mentioned above

191. The presence of ayes complications in JRA (Fig. 71) is indication for the beginning of the therapy ith:A. MethothrexateB. PenicillaminC. NSARD’sD. *GlucocorticoidsE. Gold compounds

192. Enter the basic mechanism of bronchial obstruction in newborn infants fig.13:A. Foreign body in bronchusB. *Mucus inside the bronchusC. Reduction of bronchial muscleD. Swelling of the bronchial mucosaE. Compression of the bronchus from the outside

193. Specify the basic mechanism of bronchial obstruction in infants on fig.13:A. Foreign body in bronchusB. Mucus inside the bronchusC. Reduction of bronchial muscle

D. *Swelling of the bronchial mucosaE. Compression of the bronchus from the outside

194. Specify the basic mechanism of bronchial obstruction in children 1-5 years on fig. 13:A. *Foreign body in bronchusB. Mucus inside the bronchusC. Reduction of bronchial muscleD. Swelling of the bronchial mucosaE. Compression of the bronchus from the outside

195. Call procedure in the picture on fig. 14:A. InhalationB. Nasal instillationC. Taking the material for plantingD. Nasal insufflationE. *Cleansing the nasal passages

196. Specify the basic mechanism of bronchial obstruction in school children with bronchial asthma on ig. 13:A. Foreign body in bronchusB. Mucus inside the bronchusC. *Reduction of bronchial muscleD. Swelling of the bronchial mucosaE. Compression of the bronchus from the outside

197. Specify the basic mechanism of bronchial obstruction in preschool children with bronchial asthma on ig. 13:A. Foreign body in bronchusB. Mucus inside the bronchusC. Reduction of bronchial muscleD. *Swelling of the bronchial mucosaE. Compression of the bronchus from the outside

198. At what degree of respiratory insufficiency used this method (on fig. 15) of oxygen therapy?A. 0B. 1C. *2D. 3E. 4

199. Specify the purpose of this procedure on fig. 15.A. Cleansing the nasal passages B. Control of O2 concentration on breathing C. Control of CO2 during respiration D. InhalationE. *Oxygen therapy

200. What procedure is shown in the picture on fig.16?A. BronchoscopyB. *Laryngoscopy C. Pharyngoscope D. Endophotografiya pharynxE. Endophotografiya larynx

201. Specify the indications for laryngoscopy in children on fig.16:A. Identifying the causes of changes voteB. Identifying causes of pain in the throat or in the earC. Identifying the causes difficulty swallowing, foreign body sensation in the throat or the

presence of lood expectorationD. Detection of laryngeal lesions, narrowing his or violations of the airwayE. *All of the above

202. Enter the preparations for laryngoscopy on fig.16:A. Irrigation throat by lidokainB. General anesthesiaC. *Refrain from taking food and fluidD. Conduct allergy testingE. Chest X-ray

203. Specify the conditions when the laryngoscopy (on fig. 16) is not done of children?A. *Obstructive bronchitisB. Foreign body upper airwayC. Differential diagnosis of false and true croupD. Difficulty swallowingE. Sudden attacks of cough

204. What is depicted on the fig. 17?A. UV sterilizerB. *Oxygen tentC. Protective mask D. Mask for apparatus AmbuE. Incubator

205. This method oxygen therapy is most effective (on fig. 17)?A. *During 1-st year B. For 2-7 yearsC. For school childrenD. All agesE. Newborns

206. Children of any age using this method oxygen therapy on fig. 18:A. During 1-st year B. For 2-7 yearsC. For school childrenD. *All agesE. Newborns

207. When using this method of oxygen therapy on fig. 18?A. *RI І-ІІ st.B. RI ІІІ st.C. Asthmatic statusD. Hypoxic comaE. Stenosis of the larynx ІV st.

208. What is depicted on the fig. 19?A. Cleansing the respiratory tract B. Control of O2 concentration on breathing C. Control of CO2 during respiration D. *InhalationE. Oxygen therapy

209. What used to inhalation in stenosing laryngotracheitis on fig. 19?A. Expectorants herbal concoctionsB. b-blockers C. Anesthetics D. Soda-saline E. *Decongestants mixture

210. What used to inhalation in obstructive bronchitis on fig. 19?A. *Expectorants herbal concoctionsB. b-blockers C. Anesthetics D. Soda-saline

E. Decongestants mixture211. What used to inhalation of bronchial asthma on fig. 19?

A. Expectorants herbal concoctionsB. *b2-agonistsC. Anesthetics D. Soda-saline E. Decongestants mixture

212. What used to inhalation of acute tracheitis on fig. 19?A. Expectorants herbal concoctionsB. b2-agonistsC. Anesthetics D. *Soda-saline E. Decongestants mixture

213. What used to inhalation for angina on fig. 19?A. Expectorants herbal concoctionsB. b2-agonistsC. *Anesthetics D. Soda-saline E. Decongestants mixture

214. What is depicted on the fig.20?A. Ultrasonic nebulizer B. Cardio-Respiratory Monitor C. *Apparatus for the artificial lungs ventilationD. Infusion dosingE. Apparatus Ambu

215. This form of the chest is called on fig. 21:A. *Barrel chestB. HyperstenicC. HyposthenicD. AsthenicE. Normal

216. This form of chest indicates on fig.21:A. *EmphysemaB. Attack of asthmaC. Obstructive bronchitisD. False croupE. True croup

217. Put the diagnosis of patient with chest X-ray on fig. 22.A. Acute pneumoniaB. *Acute obstructive bronchitisC. Acute bronchiolitisD. Foreign bodyE. Congenital heart disease

218. Describe the changes on the radiograph on fig.22.A. Changing the shape of the heartB. Hilar infiltration C. Enhancement bronho-vascular pattern D. *Diminished of the vascular pattern on the periphery and increased in the central partsE. Deformation of the bronchial pattern

219. What is depicted on the fig 23?A. EpistaxisB. CoryzaC. Stridor nasal

D. Symptom of "the rabbit's nose"E. *Allergic rhinitis

220. What indicates this symptom on fig 23? A. Acute respiratory infectionB. EpistaxisC. Sridor nasalD. Allergic rhinitisE. *The beginning of an asthmatic attack

221. What period of asthma conducted allergic skin tests (on fig. 24)A. InitialB. AcuteC. Asthmatic status D. Healing E. * Remission

222. What are the allergens most often the cause of asthma in children (on fig. 24)?A. Occupational irritants B. Tobacco smoke C. CockroachesD. *Domestic dust mitesE. Pollen

223. What is depicted on figure 25?A. MDI - Metered dose inhalerB. *Peak flow meterC. Nasal sprayD. Spray for the throatE. Spray the mouth

224. What is depicted on figure 26?A. *MDI - Metered dose inhalerB. Peak flow meterC. Nasal sprayD. Spray for the throatE. Spray the mouth

225. For any disease using this method on fig. 26?A. *Bronchial asthmaB. Obstructive bronchitisC. BronchiolitisD. Stenosis of the larynxE. All these

226. The child of 10 years complains of high temperature, pain in abdomen, and pain during urination. In rinalysis: protein – 0.33 g/l, leucocytes – in all visual field, erythrocytes 5-10 in visual field. What nvestigation is it necessary to do to know the level of urinary system damaging (Fig. 87)?A. *UrographyaB. To take urine by catheterC. Bacteriological test of urineD. Ultrasound investigationE. Zimnitskiy test

227. The child of 9 years become ill acutely: temperature is 39 C, pain in lower back, and pain during rination. Pasternatskiy symptom is positive bilaterally, more in the left side (Fig. 88). What is the most robable diagnosis?A. *Acute pyelonephritisB. Acute viral infectionC. Acute cystitisD. Acute glomerulonephritis

E. Kidney colic228. The child is 10 years old. The complaints are: increase of temperature, pain in abdomen, and pain

uring urination. In analysis of urine: protein – 0,33 g/l, leucocytes – in all field of view, erythrocytes 5-10 in ield of view. What investigation it is necessary to prescribe to know the level of damaging of urinary system Fig. 87)?A. *UrographyB. To take urine by catheterC. Bacteriological test of urine D. Ultrasound investigation E. Zimnitskiy test